Nurs1500 LSN unit (test#2)

Pataasin ang iyong marka sa homework at exams ngayon gamit ang Quizwiz!

A nurse is preparing to administer lactated Ringer's (LR) IV 100ml over 15min. The nurse should set the infusion pump to deliver how many mL/hr? (Round the answer to the nearest whole number. Do not use a trailing zero)

400

Active Learning Scenario: Chapter 51 Discuss medication considerations with vascular insufficiency

Vascular insufficiency prevents distribution of a medication to affected tissue. The nurse should document and monitor for the medication's effectiveness and reports concerns to the provider.

Box 37-1 Warning signs of colon cancer

-Change in bowel elimination pattern -Blood in the stools -Rectal or abdominal pain -Change in character of the stool -Sensation of incomplete emptying after bowel movement

A nurse is preparing to administer methylprednisolone 10mg by IV bolus. The amount available is methylprednisolone injection 40mg/mL. How many mL should the nurse administer? (Round the answer to the nearest tenth. Do not use a trailing zero)`

0.3

Table 35-2 Sources, Functions, and Significance of Carbohydrates, Protein, and Fat

Carbohydrates: Simple sugars and starch- Fruits, Vegetables, Grains (rice, pasta, breads, cereals), Dried peas and beans, milk (lactose), Sugars (white and brown sugar, honey, molasses, syrup); Provide energy, spare protein so it can be used for other functions, Prevent ketosis from inefficient fat metabolism; An adequate intake for total fiber is 25g/day (women) and 38g/day (men), maximum level of 25% of total calories or less from added sugars, low carbohydrate intake can cause ketosis, high simple sugar intake increases the risk for dental caries Cellulose and other water-insoluble fibers: Whole wheat flour and wheat bran, vegetables (cabbage, peas, green beans, wax beans, broccoli, brussel sprouts, cucumber skins, peppers, carrots), Apples; absorb water to increase fecal bulk, decrease intestinal transit time; Are nondigestable, therefore are excreted, help relieve constipation, North Americans are urged to eat more of all types of fiber, excess intake can cause gas, distention, and diarrhea Water soluble fibers: Oat bran and oatmeal, dried peas and beans, vegetables, prunes, pears, apples, bananas, oranges; slow gastric emptying, lower serum cholesterol level, delay glucose absorption; help improve glucose intolerance in diabetics Protein: Milk and milk products, meat, poultry, fish, eggs, dried peas and beans, nuts; tissue growth and repair, component of body framework: bones, muscles, tendons, blood vessels, skin, hair, and nails; component of body fluids: hormones, enzymes, plasma proteins, neurotransmitters, mucus; Helps regulate fluid balance through oncotic pressure, helps regulate acid base balance, detoxifies harmful substances, forms antibodies, transports fat and other substances through the blood, provides energy when carbohydrate intake is inadequate; Experts recommend that we eat less animal protein and more vegetable protein. Protein deficiency is characterized by edema, retarded growth and maturation, muscle wasting, changes in the hair and skin, permanent damage to physical and mental development (in children), diarrhea, malabsorption, numerous secondary nutrient deficiencies, fatty infiltration of the liver, increased risk for infections, and high mortality; Protein malnutrition occurs secondary to chronic diseases, such as cancer, AIDS, COPD, it may also result from acute critical illnesses such as trauma and sepsis. It may also be seen in the homeless, the elderly, fad dieters, adults addicted to drugs or alcohol, people with eating disorders. Fat: Butter, oils, margarine, lard, salt pork, salad dressings, mayonnaise, bacon; Whole milk and whole milk products; High fat meats, Nuts; Provides energy, provides structure, insulates the body, cushions internal organs, necessary for the absorption of fat soluble vitamins; High fat diets increase the risk for heart disease and obesity and are correlated with an increased risk for colon and breast cancers

Table 28-5 Intramuscular Injection Needle length

Site/Age--- Needle length: Vastus lateralis- 5/8" to 1" Deltoid (children)- 5/8" to 1 1/4" Deltoid (adult)- 5/8" to 1 1/2" Ventrogluteal (adults)- 1 1/2"

Prochlorperazine (Compazine), 7.5mg IM, is ordered. Prochlorperazine is available as 5mg/mL. How much would the nurse administer?

dose desired x quantity on hand = X(desired quantity) --------------- Dose on hand 7.5mg x 1mL= X ------- 5mg (1.5)(1)= X X= 1.5mL

Ondansetron (Zofran) 0.15mg/kg is ordered. Ondansetron is available as 2mg/mL.The patient weighs 30kg. How much would the nurse administer?

0.15mg = X ------- ------- 1kg . 30kg 1x= 4.5mg Dose on hand = . Dose desired --------------- --------------- Quantity on hand . Quantity desired 2mg = 4.5mg ----- ------- 1mL . X 2X= 4.5 X= 2.25mL

A nurse is preparing to administer acetaminophen 320mg PO every 4 hr PRN for pain. The amount available is acetaminophen liquid 160mg/5 mL. How many mL should the nurse administer per dose? (Round the answer to the nearest tenth. use a leading zero if it applies. Do not use a trailing zero)

10

Active Learning scenario: Chapter 46 List four areas of the body where biotransformation takes place

Biotransformation (metabolism) changes medication into less active forms or inactive forms by the action of enzymes. This occurs primarily in the liver, but also takes place in the kidneys, lungs, intestines, and blood

Active Learning Scenario: Chapter 51 Discuss medication considerations with impaired kidney function.

Impaired kidney function prevents or delays medication excretion, which increases the risk for toxicity. Decreased kidney function is a major cause of medication accumulation leading to toxicity

The nurse is assessing patients for postoperative complications. What is the most commonly assessed postanesthesia recovery emergency? A. Respiratory obstruction B. Cardiac distress C. Wound infection D. Dehydration

A. Respiratory obstruction

A nurse is preparing to administer a 0900 medication to a client. Which of the following are acceptable times for this medication? (Select all that apply) A. 0905 B. 0825 C. 1000 D. 0840 E. 0935

A.D. 0905/0840 The nurse should administer medications within 30minutes of the time it is due

Table 37-3 Classification of Anitdiarrheal medications

Action on gastrointestinal smooth muscle -Opium (Paregoric); Increases smooth muscle tone, decreases GI motility, Diminishes GI secretions (action); effective (advantages); May be addictive due to morphine content, may cause drowsiness and lightheadedness, should be discontinued as soon as diarrhea has diminished (cautions) -Dipheoxylate and Atropine (Lomotil); slows gastric motility through local effect on the gastrointestinal wall (action); Effective (advantages); is chemically related to morphine, atropine added to prevent addiction but in high doses can become addictive (Cautions) -Loperamide (Imodium); inhibits peristalsis via direct effect on gastrointestinal wall muscles (action); Not addictive, longer duration than Lomotil (advantages); May cause drowsiness, Must be discontinued if no improvement in 48 hours with acute cases (cautions) Absorbent -Kaolin-pectin (Kaopectate); Absorbs and softens (action), no drowsiness (advantages); may interfere with absorption of other oral medications, may interfere with absorption of nutrients with prolonged use (caution) Antisecretory/antimicrobial Bismuth subsalicylate (Pepto Bismol);Decreases gastrointestinal tract secretion, has antimicrobial action against bacterial and viral pathogens (Action); no drowsiness (advantage); Contains salicylates, check with physician before giving to children or administering with aspirin, may decrease absorption of some medications (cautions)

Active Learning Scenario: Chapter 43 List at least 8 factors that affect bowel elimination, along with a brief example of description of each

Age -Infants: Breast milk stools, Watery and yellow brown; Formula stools, Pasty and brown -Toddlers: Bowel control at 2-3 years old -Older adults: Decreased peristalsis, relaxation of sphincters Diet: -Fiber requirement: 25-30g per day -Lactose Intolerance: Difficulty digesting milk products -Fluid requirement: 2000-3000 mL/day from fluid and food sources Physical Activity: Stimulates intestinal activity Psychosocial factors: -Emotional distress increasing peristalsis and exacerbating chronic conditions (colitis, Crohn's disease, ulcers, irritable bowel disease) -Depression decreasing peristalsis Personal habits: Use of public toilets, false perceptions of the need for one-a-day bowel movements, lack of privacy when hospitalized Positioning: -Normal: squatting -Immobilized client: difficulty defecating Pain: -Discomfort leading to suppression of the urge to defecate -Opioid use contributing to constipation Pregnancy: -Growing fetus compromising intestinal space -Slower peristalsis -Straining increasing the risk of hemorrhoids Surgery and anesthesia: -Temporary slowing of intestinal activity -Paralytic ileus Medications: -Laxatives: to soften stool; overuse leads to chronic constipation -Cathartics: to promote peristalsis

Table 28-4 Intramuscular site selection Criteria/Recommended site

Age of patient: Infants- vastus lateralis Toddlers and children- vastus lateralis or deltoid Adults- Ventrogluteal or deltoid Medication type: Biologicals (infants and young children)- Vastus lateralis BIologicals (older children and adults)- Deltoid Hepatitis B/Rabbies- Deltoid Medications that are known to be irritating, viscous, or oily solutions- Ventrogluteal

Active Learning Scenario: Chapter 42 List at least four different types of therapies with examples of each

Alternative medical philosophy: traditional Chinese medicine, Ayurvedic medicine, homeopathy Biological and botanical therapies: diets, vitamins, minerals, herbal preparations Body manipulation: massage, touch, chiropractic therapy Mind-body therapies: Biofeedback, art therapy, mediation, yoga, psychotherapy, tai chi Energy therapies: Reiki, therapeutic touch

Active Learning Scenario: Chapter 39 List two common eating disorders and their characteristics

Anorexia Nervosa: Significantly low body weight for gender, age, developmental level, and physical health. Fears being fat Self-perception of being fat Consistent restriction of food intake or repeated behavior that prevents weight gain Bulimia Nervosa: Cycle of binge eating followed by purging (vomiting, using diuretics or laxatives, exercise, fasting) Lack of control during binges Average of at least one cycle of binge eating and purging per week for at least 3 months Binge eating disorder: Repeated episodes of binge eating Feels a loss of control when binge eating, followed by an emotional response such as guilt, shame, or depression Does not use compensatory behaviors such as purging Binge eating episodes can range from 1 more than 14 times per week Clients are often overweight or obese

Table 35- Preventing Complications of Enteral Feeding

Aspiration: use appropriate measures to check tube placement; elevate the head of the bed at least 30-45 degrees during feeding and for 1 hour afterward; give small, frequent feedings; avoid oversedation of patient; check residual volume per policy Clogged tube: flush tube before and after feeding, every 4 hours during continuous feeding, and after withdrawing aspirate; Instill 30mL of warm water with 50mL or 60mL syringe to attempt to unclog tube Nasal erosion with nasogastric or nasointestinal tubes: Check nostrils every shift for signs of pressure; Clean and moisten nares every 4 to 8 hours; Start feeding at a slow rate Diarrhea: Prevent contamination in both open and closed systems; Change delivery set every 12 to 24 hours according to agency policy; Refrigerate opened cans of formula and discard after 24 hours; Limit hang time to 8 hours when using open system; Use aseptic technique for patients who are immunosuppressed or acutely ill; Assess for fecal impaction. Other GI symptoms (nausea, vomiting, distention): Check residual prior to intermittent feedings and every 4 hours during continuous feedings; Avoid oversedating client (Delays gastric emptying); Administer GI motility medications (metoclopramide), as ordered Unplanned extubation: Anchor tube adequately with commercial device, elastic net, or tape; Check on patient frequently; Measure external length of tubing at regular intervals Stoma infection: Clean skin every shift with soap and warm water. Dry thoroughly; Use topical antibiotics and/or antifungals, as ordered; assess for signs of infection; Request consult with wound care specialist, as needed

The nurse is assessing patients for BMR. Which patient would the nurse suspect would have an increased BMR? A. An elderly patient B. A patient who has a fever C. A patient who is fasting D. A patient who is asleep

B. A patient who has a fever

The nurse is caring for a 48 year old Native American man hospitalized following am MI who asks to see his medicine man. What would be the nurse's best response to this patient? A. Inform the patient that he may visit his medicine man when he is discharged B. Arrange for a visit with the medicine man in the hospital C. State that while in the hospital, it is necessary to follow traditional medicine D. Ask the patient to concentrate on getting better and following his care plan

B. Arrange for a visit with the medicine man in the hospital

A nurse forms the following nursing diagnosis for a patient: Impaired Urinary Elimination related to maturational enuresis. Based on this diagnosis, for which patient is the nurse caring? A. An adult older than 65 years of age who is incontinent B. A child older than 4 years of age who has involuntary urination C. A 12-month-old child who has involuntary urination D. A patient with neurologic damage resulting in bladder dysfunction

B. Maturational enuresis is involuntary urination after an age when continence should be present. A 12 month old child is not expected to be continent, and incontinence and neurologic damage are not maturational problems

To convert 0.8 grams into milligrams, the nurse should do which of the following? A. Move the decimal point two places to the right B. Move the decimal point three places to the right C. Move the decimal point two places to the left D. Move the decimal point three places to the left

B. Move the decimal point three places to the right

A nurse caring for patients in a PACU assesses a patient who is displaying signs and symptoms of shock. What is the priority nursing intervention for this patient? A. Remove extra coverings on the patient to keep temperature down B. Place the patient in a flat position with legs elevated 45 degrees C. Do not administer any further medication D. Place the patient in the prone position

B. Place the patient in a flat position with legs elevated 45 degrees

A 9 month old baby is scheduled for heart surgery. When preparing this patient for surgery, the nurse should consider which surgical risk associated with infants? A. Prolonged wound healing B. Potential for hypothermia or hyperthermia C. Congestive heart failure D. Gastrointestinal upset

B. Potential for hypothermia or hyperthermia

Older adults often have reduced vital capacity as a result of normal physiologic changes. Which nursing intervention would be most important for the postoperative care of an older surgical patient specific to this change? A. Take and record vital signs every shift. B. Turn, cough, and deep breathe every 4 hours. C. Encourage increased intake of oral fluids. D. Assess bowel sounds daily.

B. Reduced vital capacity in older adults increases the risk for respiratory complications, including pneumonia and atelectasis. Having the patient turn, cough, and deep breathe every 4 hours maintains respiratory function and helps to prevent complications

Table 27-2 Integrative care of the common cold

Complementary and Alternative therapies: Acupuncture to appropriate areas to reduce sinus congestion Rest and fluids Herbs, vitamin C to stimulate the immune system Allopathic approaches: Decongestant over the counter medications Rest and fluids Vitamin C to stimulate the immune system

The nurse is administering a large volume cleansing enema to a patient who complains of severe cramping upon introduction of the enema solution. What would be the nurse's next action? A. Place the patient on bedpan in the supine position while receiving enema B. Remove the tube and check for any fecal contents C. Modify the amount and length of administration D. Lower solution container and check temperature and flow rate

D. Lower the solution container and check temperature and flow rate

A nurse is preparing to administer digoxin to a client who states, " I don't want to take that medication. I do not want to take on more pill." Which of the following responses should the nurse make? A. Your physician prescribed it for you, so you really should take it B. Well, lets just get it over quickly then C. Okay, I'll give you your other medications D. Tell me your concerns about taking this medication

D. Tell me your concerns about taking this medication. Although client's have the right to refuse a medication, the nurse is correct in determining the reason for refusal by asking the client his concerns. Then the nurse can provide information about the risk of refusal and facilitate an informed decision. At that point, if the client still exercises his right to refuse a medication, the nurse should notify and the provider and document the refusal and the actions the nurse took The response dismisses the client's concerns The nurse is dismissing the client's concerns about taking the medication by continuing with medication administration Although clients have the right to refuse a medication, the nurse should provide information about the risk of refusal instead of proceeding with medication administration.

The doctor has ordered the collection of a fresh urine sample for a particular examination. Which urine sample would the nurse discard? A. The sample collected immediately after lunch B. The bedtime voiding C. The voiding collected at 4pm D. The first voiding of the day

D. The first voiding of the day

Ciprofloxacin (Cipro), 500mg PO, is ordered. Ciprofloxacin is available as 250mg tablets. How many tablets would the nurse administer

Dose desired x quantity on hand = X ---------------- Dose on hand 500mg x 1 tablet= X ------- 250mg (2)(1)= 2 tablets

Clonidine (Catapres), 0.1mg PO, is ordered. Clonidine is available as 0.2mg tablets. How many tablets would the nurse administer?

Dose desired x quantity on hand = X ---------------- Dose on hand 0.1mg x 1 tablet= X ------- 0.2mg (1/2)(1)= X X= 1/2 tablet

Furosemide (Lasix), 20mg PO, is ordered. Furosemide is available as 40mg tablets. How many tablets would the nurse administer?

Dose desired x quantity on hand = X ---------------- Dose on hand 20mg x 1 tablet= X ------ 40mg (1/2)(1)= 1/2 tablets

Nalbuphine (Nubain), 1.5mg IM, is ordered. Nalbuphine is available as 1mg/mL. How much would the nurse administer?

Dose desired x quantity on hand= X(desired quantity) --------------- Dose on hand 1.5mg x 1mL= X ------- 1mg (1.5)(1)= X X= 1.5mL

Active Learning Scenario: Chapter 44 List at least 4 types of the six types of urinary incontinence, along with a brief example or description of each.

Stress: The loss of small amounts of urine from increased abdominal pressure without bladder muscle contraction with laughing, sneezing, or lifting Urge: The inability to stop urine flow long enough to reach the bathroom due to an overactive detrusor muscle with increased bladder pressure Overflow: Urinary retention from bladder over-distention and frequent loss of small amounts of urine due to obstruction of the urinary outlet or an impaired detrusor muscle Reflex: The involuntary loss of a moderate amount of urine usually without warning due to hyperreflexia of the detrusor muscle, usually from spinal cord dysfunction Functional: The loss of urine due to factors that interfere with responding to the need to urinate, such as cognitive, mobility, and environmental barriers Total: The unpredictable, involuntary loss of urine that does not generally respond to treatment

Box 29-1 Preoperative Information for Outpatient/Same Day Surgery

Using simple language the patient can understand, instruct the patient (verbally and in writing) as follows: -List medications routinely taken, and ask the physician which should be taken or omitted the morning of surgery. -Notify the surgeon's office if a cold or infection develops before surgery -List all allergies, and be sure the OR staff is aware of them. -Follow all instructions from your surgeon regarding bathing or showering with a special soap solution -Remove nail polish and do not wear makeup on the day of the procedure -Leave all jewelry and valuables at home -Wear clothing that buttons in front; short sleeved garment are better or surgery on the hands -Have someone available for transportation home after recovery from anesthesia Inform patient of: -Limitations on eating and drinking before surgery, with a specific time to begin the limitations -When and where to arrive or the procedure, as well as the estimated time when the procedure will be performed

Box 29-7 Nursing Interventions to Facilitate Postoperative Coping and Adaptation

-Accept each patient as a unique individual -Identify through verbal and nonverbal cues patients who are at risk for alteration in self concept. The risk is increased if the patient has little support from others, a visible alteration, or an alteration that will seriously affect functional ability -Allow time for patients and families to verbalize their feelings about the alteration, and do not assume that all patients will have problems -Identify and support strengths and effective coping mechanisms -Encourage the patient and family to be part of goal setting and decision making throughout the surgical experience -Provide teaching and honest information to the patient and family about all aspects of care -Work collaboratively with other members of the health care team to provide referrals and resources as necessary to meet physical, psychological, and spiritual needs

Box 35-4 Communicating Effectively About Nutrition with culturally diverse patients

-Acquire basic information about health beliefs and practices of various cultural groups in your health care setting. This provides a basis for assessing patients beliefs and practices. Recognize, however, that within all cultures and ethnic groups, there are members who do not hold all the values of the group -Ask specifically about the use of folk or home remedies prescribed by a nontraditional healer -Determine the patients language preferences for spoken and written communication -Utilize printed or audiovisual information that is in the language spoken by your parents -Promote healthy food choices by identifying healthy traditional food practices and encourage their use -Support healthy new food practices -Discourage the adoption of less healthy American dietary practices - Encourage cultural sensitivity in healthcare workers in your particular setting -Recognize that diversity exists within cultural groups. For example, the hispanic population includes Mexicans, Cubans, Puerto Ricans, and other Latino groups -Emphasize threads or messages in health teaching that are common to all cultures (concern about family, faith, and home) - Help culturally diverse patients to value and understand the importance of communicating concerns and asking questions about prescribed dietary practices

Box 29-4 Nursing Interventions to Prevent of Monitor Postoperative Cardiovascular Complications

-Assess and document vital signs as ordered and as the patient's status dictates, using preoperative assessments as a baseline -Provide covers, forced warm air, or other warming device or techniques as necessary to prevent shivering and hypothermia -Maintain fluid balance -Maintain accurate intake and output - Monitor rate, type, and assess site of intravenous fluids - Assess skin turgor and hydration of mucous membranes -Monitor amount, color, and consistency or wound drainage (dressings and drains or tubes) -Implement leg exercises and turning in bed every 2 hours -Assist with ambulation. Ambulation usually begins the evening of surgery and increases as tolerated; blood pressure and pulse and respiratory rates are used to monitor tolerance -Apply and follow protocols for graduated compression stockings or pneumatic compression devices, if ordered -Administer anticoagulant medications, if prescribed -Measure bilateral calf and thigh circumference daily -Assess for leg swelling, tenderness or palpable venous cord -Avoid positioning that impedes venous return (ex. Do not mechanically raise the knee portion of the bed or place pillows under the knees)

Box 35-7 Special Considerations and Interventions for Feeding Patients with Dementia or Other Alterations in Cognition

-Change the environment in which meals occur -Assess the area where meals are served. Create a homelike environment by preparing food close to the place where it will be served to stimulate senses. -Observe as many former rituals as possible, such as handwashing and saying a blessing -Avoid clutter and distractions -Maintain a pleasant, well lighted room. Play calming music -Keep food as close to its original form as possible -Serve meals in the same place at the same time -Closely supervise mealtime -Check food temperatures to prevent accidental mouth burns -Assist as needed. Be alert for cues from the patient. Turning away may signal that the patient has had enough to eat or that the patient needs to slow down. Leaning forward with an open mouth usually means the patient is ready for more food. -Stroking the underside of the chin may help promote swallowing -Provide one food item at a time. Offer small, frequent eating opportunities. A whole tray of food may be overwhelming -Ensure that the patient's glasses and hearing aid are working properly -Demonstrate what you want the patient to do. State the goal clearly, and then mimic the action with exaggerated motions. -Provide between meal snacks that are easy to consume using the hands -Use adaptive feeding equipment as needed such as weighted utensils, large handled cups, and larger or smaller silverware than standard -Promote family involvement to encourage eating.

Box 29-2 Nursing Interventions to meed psychological needs of patients having surgery

-Establish and maintain a therapeutic relationship, allowing the patient to verbalize fears and concerns -Use active listening skills to identify and validate verbal and nonverbal messages revealing anxiety and fear -Use touch, as appropriate, to demonstrate genuine empathy and caring -Be prepared to respond to common patient questions about surgery, including: ---Will I lose control of body functions while I'm having surgery? ---How long will I be in the operating room and PACU? ---Where will my family be? ---Will I have pain when I wake up? ---Will the anesthetic make me sick? ---Will I need a blood transfusion? ---How long will it be before I can eat? ---What kind of scar will I have? ---When will I be able to be sexually active? ---When can I go back to work?

Box 35-2 Method of Calculating Caloric Requirements

-Estimate the basal metabolic rate (BMR) or the amount of calories necessary to maintain the body at rest. Because men usually have a greater muscle mass than women, their caloric requirements are slightly higher. A rule of thumb guideline for calculating BMR is to multiply healthy weight (in pounds) by 10 for women and 11 for men. For a person who is overweight, multiply by the average weight within the healthy weight range. For example: Male: 130lb x 11cal/lb= 1430 cal/day Female: 130lb x 10cal/lb= 1300 cal/day -Estimate the total number of calories according to usual activity level. Choose the category that describes usual activities -20% Sedentary: mostly sitting, driving, sleeping, standing, reading, typing, other low intensity activities -30% Light Activity: light exercise, such as walking not more than 2 hours per day -40% Moderate activity: moderate exercise such as heavy housework, gardening, and very little sitting -50% High activity: active in physical sports or a labor intensive occupation, such as construction work -Multiply the BMR by the percentage associated with the activity level. For example, the average female's BMR= 1300 calories per day. She engages light activity 1300 x 30%= 390 1300+390= 1690 total daily calories

Box 36-4 Patient Intermittent Self Catheterization

-Explain the reason for self catheterization and corresponding health issues related to the need for catheterization -Explain the consequences of not doing intermittent self catheterization, such as upper urinary tract problems, urinary tract infections, and incontinence -Explain potential complications, such as bleeding and the risk of urinary tract infections and what to do if they occur. -Ensure privacy and dignity -Include discussion regarding the frequency of intermittent catheterization and how to incorporate into usual daily routine. -Explain the anatomy of the urinary tract, hygiene, and preparation of the catheter -Demonstrate how to open, hold, and use the catheter -Explain catheterization process, demonstrate process, and observe return demonstration by patient -Explores process to obtain supplies, cleaning of reusable catheters, and storage -Provide information in an appropriate format (such as written materials or video, in appropriate language) to reinforce instruction -Allow the patient adequate time to ask questions -Provide information about how to recognize a urinary tract infection and other signs/symptoms to report to primary health care provider -Aids are available to help meet the challenges of intermittent self catheterization for patients with poor eyesight, reduced mobility, and/or reduced manual dexterity

Box 35-8 Special considerations and Interventions for Feeding Patients With Dysphagia

-Provide at least a 30 minute rest period prior to mealtime. A rested person will likely have less difficulty swallowing. -Sit the patient upright, preferably in a chair. If bedrest is mandatory, elevate the head of the bed to a 90 degree angle -Provide mouth care immediately before meals to enhance the sense of taste. -Avoid rushed or forced feeding. Adjust the rate of feeding and size of bites to the patient's tolerance -Collaborate to obtain a speech therapy consult for swallowing evaluation -Initiate a nutrition consult for appropriate diet modification such as chopping, mincing, or pureeing of foods and liquid consistency (thin, nectar thick, honey like, spoon thick) -Keep in mind that some patients may find thickened liquids unpalatable and thus drink insufficient fluids. -Reduce or eliminate distractions at mealtime so that the patient can focus attention on swallowing. -Alternate solids and liquids -Assess for signs of aspiration during eating: sudden appearance of severe coughing; choking, cyanosis; voice change; hoarseness; and/or gurgling after swallowing; frequent throat clearing after meals; or regurgitation through the nose and mouth -Inspect oral cavity for retained food -Avoid or minimize the use of sedatives and hypnotics since these agents may impair the cough reflex and swallowing

Box 27-3 When healing touch can be used

-To stimulate wound healing -To relieve pain -To reduce anxiety and stress -To energize the auric field -To promote health -To assist patients before and after medical treatments and procedures such as surgery -To support the dying process

A nurse is preparing to administer ketorolac 0.5mg/kg IV bolus every 6 hr to a school aged child who weighs 66 lb. The amount available is ketorolac injection 30 mg/ml. How many mL should the nurse administer per dose? (Round to the nearest tenth. Use a leading zero if it applies. Do not use a trailing zero)

0.5

A nurse is preparing to administer dextrose 5% in water (D5W) 1000 mL IV to infuse over 10 hr. The nurse should set the IV infusion pump to delver how many mL/hr? (Round the answer to the nearest whole number. Do not use a trailing zero)

100

A nurse is preparing to administer metoprolol 200mg PO daily. The amount available is metoprolol 100mg/tablet. How many tablets should the nurse administer (Round to the nearest whole number. Do not use a trailing zero)

2

A nurse is preparing to administer dextrose 5% in lactated Ringer's (D5LR) 1000 mL to infuse over 6 hr. The drop factor of the manual IV tubing is 15gtt/mL. The nurse should adjust the manual IV infusion to deliver how many gtt/min? (Round to the nearest whole number. Do not use a trailing zero)

42

A nurse is preparing to administer 0.9% sodium chloride (0.9% NaCl) 250ml IV to infuse over 30 min. The drop factor of the manual IV tubing is 10 gtt/mL. The nurse should adjust the manual IV infusion to deliver how man gtt/min? (Round to the nearest whole number. Do not use a trailing zero.)

83

An oral medication has been ordered for a patient who has a nasogastric tube in place. Which nursing activity would increase the safety of medication administration? A. Check the tube placement before administration B. Have Mr. Moran swallow the pills around the tube C. Flush the tube with 30 to 40mL saline before medication administration D. Bring the liquids to room temperature before administration

A Check the tube placement before administration

A nurse works in an office that follows the allopathic philosophy of medicine. For which patient would this type of medicine be most effective? A. A patient whose spinal cord was severed in a motor vehicle accident B. A patient diagnosed with juvenile diabetes C. A patient who has COPD D. A patient who has rheumatoid arthritis

A. A patient whose spinal cord was severed in a motor vehicle accident

A patient had a surgical procedure that necessitated a thoracic incision. The nurse anticipates that he will have a higher risk for postoperative complications involving which body system? A. Respiratory system B. Circulatory system C. Digestive system D. Nervous system

A. A thoracic incision makes it more painful for the patient to take deep breaths or cough. Shallow respirations and ineffective coughing increase the risk for respiratory complications

When checking the placement of a gastrostomy or jejunostomy tube, the nurse must make regular comparisons of: A. Tube length B. Gastric fluid C. pH D. Air pressure

A. Tube length

A nurse is caring for a patient who has crippling rheumatoid arthritis. Which nursing intervention best represents the use of integrative care? A. The nurse administers Naproxen (Aleve) and uses guided imagery to take the patient's mind off the pain. B. The nurse prepares the patient's physician approved herbal tea and uses meditation to relax the patient prior to bed. C. The nurse administers Naproxen (Aleve) and performs prescribed range of motion exercises D. The nurse arranges for acupuncture for the patient and designs a menu high in Omega 3 fatty acids

A. Adding guided imagery (CAT) to the administration of pain medications (allopathy) is an example of integrative care. A person who uses integrative care uses some combination of allopathic medicine and CAT

A nurse is caring for a postpartum patient who has stitches in the perineum from an episotomy (surgically planned incision to prevent vaginal tears). Which medication would the nurse most likely administer to this patient? A. A stool softener (Colace) B. An osmotic laxative (Miralax) C. A bulk forming laxative (Metamucil) D. An emollient laxative

A. Although all the choices are laxatives that would soften the stool and make it easier to expel, a stool softener, such as Colace is the one recommended for a patient who must avoid straining. In this case, it would help to prevent disturbing the stitches in the perineum

A nurse caring for patients with bowel alterations formulates a nursing diagnosis for a patient with a new ileostomy. Which diagnosis is most appropriate? A. Disturbed Body Image B. Constipation C. Delayed Growth and Development D. Excess Fluid Volume

A. An ileostomy may cause disturbed body image due to the invasive nature of the procedure and the presence of the stoma. Constipation does not normally occur with an ileostomy because the drainage is liquid. Growth and Development are not generally affected by the formation of an ileostomy. Excess fluid volume is unlikely to occur because the drainage is liquid and probably continual

A 72 year old woman who is scheduled for a hip replacement is taking several medications on a regular basis. Which drug category might create a surgical risk for this patient? A. Anticoagulants B. Antacids C. Laxatives D. Sedatives

A. Anticoagulant drug therapy would increase the risk for hemorrhage during surgery

A postsurgical patient is experiencing decreased lung sounds, dyspnea, cyanosis, crackles, restlessness, and apprehension. Which condition would the nurse suspect? A. Atelectasis B. Pneumonia C. Pulmonary embolus D. Thrombophelbitis

A. Atelectasis

A primary healthcare provider prescribes nose drops to be administered twice a day. Which should the nurse do when instilling the nose drops? (Select all that apply.) A. Tell the patient not to sniff the medication once administered. B. Place the patient in the supine position with the head tilted backward. C. Pinch the nares of the nose together briefly after the drops are instilled. D. Instruct the patient to blow the nose 5 minutes after the drops are instilled. E. Insert the drop applicator ½ inch into the nose toward the base of the nasal cavity.

A. Avoiding sniffing the nose drops after administration allows the medication to reach the desired areas (ethmoid and sphenoid sinuses) via gravity.

The physician has ordered an indwelling catheter inserted in a hospitalized male patient. What consideration would the nurse keep in mind when performing this procedure? A. The male urethra is more vulnerable to injury during insertion B. In the hospital, a clean technique is used for catheter insertion C. The catheter is inserted 2" to 3" into the meatus D. Since it uses a closed system, the risk for urinary tract infection is absent

A. Because of its length, the male urethra is more prone to injury and requires that the catheter be inserted 6" to 8". This procedure requires surgical asepsis to prevent introducing bacteria into the urinary tract. The presence of an indwelling catheter places the patient at risk for a UTI

A nurse is planning a high energy diet for a patient. What nutrient provides energy to the body and should be increased in the diet? A. Carbohydrates B. Vitamins C. Minerals D. Water

A. Carbohydrates

A nurse is assessing a client as part of an admission history. The client reports drinking an herbal tea every afternoon at work to relieve stress. The nurse should suspect the tea includes which of the following ingredients? A. Chamomile B. Ginseng C. Ginger D. Echinacea

A. Chamomile. The nurse should suspect the tea might contain chamomile, which produces a calming effect, or valerian, which reduces anxiety. The nurse should attempt to gain further information to confirm the ingredients of any herbal or natural products the client may use. The nurse should expect a client to use ginseng tea to improve physical endurance. The nurse should expect a client to use ginger tea to prevent or relieve nausea. The nurse should expect a client to use Echinacea tea to boost the immune system

A nurse assessing an elderly patient finds that the patient has had four urinary tract infections in the past year. Which physiologic change of aging would the nurse suspect is the cause? A. Decreased bladder contractility B. Diminished ability to concentrate urine C. Decreased bladder muscle tone D. Neurologic weakness

A. Decreased bladder contractility

The nurse is assessing an obese patient scheduled for heart surgery. Which surgical risk related to obesity should the nurse monitor? A. Delayed wound healing and wound infection B. Alterations in fluid and electrolyte imbalance C. Respiratory distress D. Hemorrhage

A. Delayed wound healing and wound infection

A nurse manager who works in a hospital setting is researching the use of energy healing to use as an integrative care practice. Which patient would be the best candidate for this type of CAT? A. A patient who is anxious about residual pain from cervical spinal injury B. A patient who is experiencing abdominal discomfort C. A patient who has chronic pain from diabetes D. A patient who has frequent cluster headaches

A. Energy healing is focused on pain that lingers after an injury heals, as well as pain complicated by trauma, anxiety, or depression. Nutritional and herbal remedies treat all chronic pain, but especially abdominal discomfort, headaches, and inflammatory conditions, such as rheumatoid arthritis

A patient who has pneumonia has had a fever for 3 days. What characteristics would the nurse anticipate related to the patient's urine output? A. Decreased and highly concentrated B. Decreased and highly dilute C. Increased and concentrated D. Increased and dilute

A. Fever and diaphoresis cause the kidneys to conserve body fluids. Thus, the urine is concentrated and decreased in amount

A nurse is teaching a client how to administer medication through a jejunostomy tube. Which of the following instructions should the nurse include? A. Flush the tube before and after each medication B. Mix your medications with your enteral feeding C. Push tablets through the tube slowly D. Mix all the crushed medications prior to dissolving them in water

A. Flush the tube before and after each medication.- The client should flush the tubing before and after each medication with 15 to 30ml water to prevent clogging the tube. To maximize therapeutic effect of a medication, the client should not mix medications with enteral formula. In addition, if the client does not receive the entire feeding, he does not receive the entire medication. This can also delay the client receiving the medication The client should not administer tablets or undissolved medications through a jejunostomy tube because they can clog the tube. The client should self administer each medication separately

A nurse in an outpatient clinic is caring for a client who has a new prescription for an antihypertensive medication. Which of the following instructions should the nurse give the client? A. Get up and change positions slowly B. Avoid eating aged cheese and smoked meat C. Report any unusual bruising or bleeding to the doctor immediately D. Eat the same amount of foods that contain vitamin K every day

A. Get up and change positions slowly. Antihypertensive medications can cause orthostatic hypotension. The nurse should instruct the client to change positions slowly and to sit or lie down when feeling dizzy or lightheaded to prevent injury Consuming foods that contain tyramine (avocados, figs, aged cheese, yeast extracts, beer, smoked meats) while taking monoamine oxidase inhibitors, not antihypertensives, can lead to hypertensive crisis. Clients taking an anticoagulant, not an antihypertensive, should report bruising, discolored urine or stool, petechiae, bleeding gums, and any other manifestations of bleeding to the provider immediately. Clients taking anticoagulants, not antihypertensives, should maintain a consistent intake of dietary vitamin K to avoid sudden fluctuations that could affect the action of the anticoagulant

A nurse is teaching a patient meditation techniques to provide mental calmness and physical relaxation. Which nursing intervention facilitates this process? A. Helping the patient to assume a specific, comfortable posture B. Providing a stimulating environment in which to conduct the meditation C. Teaching the patient to have multiple focal points D. Promoting a closed attitude to avoid judgements and distractions

A. Helping the patient to assume a specific, comfortable posture

In which position would the surgical nurse place a patient undergoing minimally invasive surgery of the lower abdomen or pelvis? A. Tredelenburg position B. Sim's position C. Lithotomy position D. Prone position

A. Trendelenburg position

The nurse is administering a subcutaneous injection of insulin to a patient. Which action would the nurse take after choosing the appropriate administration site? A. Identify the appropriate landmarks for the site chosen B. Cleanse the area around the injection site with alcohol C. Use a firm, bak and forth motion to cleanse the site D. Remove the needle cap with the dominant hand pulling it straight off

A. Identify the appropriate landmarks for the site chosen

A nurse is administering an oral medication to a patient via a gastric tube. The nurse observes the medication enter the tube, and then the tube becomes clogged. What would be the appropriate initial action of the nurse in this situation? A. Attempt to dislodge the medication with a 10ml syringe B. Notify the primary care provider C. Remove the tube and replace it with another tube D. Flush the tube with 60mL of water

A. If medication becomes clogged in a gastric tube, the nurse should attach a 10mL syringe on the end of the tube and pull back and lightly apply pressure to the plunger in a repetitive motion to attempt to dislodge the medication. If the medication does not move through the tube, the nurse should notify the primary care provider, who may request the tube be replaced.

A nurse is administering a tube feeding for a patient who is post bowel surgery. When attempting to aspirate the contents, the nurse notes that the tube is clogged. What would be the nurse's next action following this assessment? A. Use warm water and gentle pressure to remove the clog B. Use a stylet to unclog the tubes C. Administer cola to remove the clog D. Replace the tube with a new one

A. In order to remove a clog in a feeding tube, the nurse should try using warm water and gentle pressure to unclog it. A stylet should never be used to unclog a tube, and cola and meat tenderizers have not been shown effective in removing clogs. The nurse should attempt to remove the clog, and if unsuccessful, the tube should be replaced

The nurse is irrigating a nasogastric tube attached to suction and finds that the flush solution is meeting a lot of force when the plunger is pushed. What would be the nurse's first intervention in this situation? A. Inject 20-30mL of free air into the abdomen in an attempt to reposition the tube and enable flushing the tube B. Check the suction canister to ensure that the suction is working appropriately C. Assess the abdomen for distention and ask the patient if he or she is experiencing any nausea or abdominal discomfort D. Attempt to flush the tube to ensure its patency

A. Inject 20 to 30mL of free air into the abdomen in attempt to reposition the tube and enable flushing the tube

A nurse is ordered to administer epinephrine to a child who was stung by a bee and is allergic to insect bites. Which means of drug administration would the nurse use to achieve rapid absorption and quicker results in this emergency situation? A. Injection B. Oral C. Patch D. Inhalation

A. Injection

A nurse administers regional anesthesia to a patient being prepared for facial surgery. What type of regional anesthesia would be the appropriate choice for this patient? A. Nerve block B. Subdural block C. Surface anesthesia D. Local infiltration with lidocaine

A. Nerve block

A nurse is caring for a client scheduled for abdominal surgery. The client reports being worried. Which of the following actions should the nurse take? A. Offer information on a relaxation technique and ask the client if he is interested in trying it B. Request a social worker see the client to discuss meditation C. Attempt to use biofeedback techniques with the client D. Tell the client many people feel the same way before surgery and to think of something else

A. Offer information on a relaxation technique and ask the client if he is interested in trying it. It is appropriate for the nurse to recommend a noninvasive technique to facilitate coping, and to allow the client to make an informed decision about participating Meditation does not require specialized training . The nurse can use this therapy and does not need to request a social worker consult. The nurse should recognize that biofeedback requires specialized training and licensing or certification. It is not appropriate for the nurse to attempt to use these techniques. This response by the nurse is nontherapeutic because it uses stereotyping and dismisses the client's feeling. The nurse should use therapeutic communication techniques to allow the client to further verbalize fears

A nurse prepares an injection of morphine to administer to a client who reports pain. Prior to administering the medication, the nurse assists another client onto a bedpan. She asks a second nurse to give the injection. Which of the following actions should the second nurse take? A. Offer to assist the client who needs the bedpan B. Administer the injection that the other nurse prepared C. Prepare another syringe and administer the injection D.Tell the client who needs the bedpan she will have to wait for her nurse

A. Offer to assist the client who needs the bedpan.- The second nurse should offer to assist the client who needs the bedpan. This will allow the nurse who prepared the injection to administer it. A nurse should only administer medications that she prepared Preparing another syringe will delay the administration fo the pain medication Telling the client to wait is not an acceptable option for a client who needs a bedpan

A nurse is feeding an elderly patient who has dementia. Which intervention should the nurse perform to facilitate this process? A. Stroke the underside of the patient's chin to promote swallowing. B. Serve meals in different places and at different times. C. Offer a whole tray of various foods to choose from. D. Avoid between-meal snacks to ensure hunger at mealtime.

A. To feed a patient with dementia, the nurse should stroke the underside of the patient's chin to promote swallowing, serve meals in the same place and at the same time, provide one food item at a time since a whole tray may be overwhelming, and provide between meal snacks that are easy to consume using the hands

A nurse is performing an admission assessment for an older adult client. After gathering the assessment data and performing the review of systems, which of the following actions is a priority for the nurse? A. Orient the client to his room B. Conduct a client care conference C. Review medical prescriptions D. Develop a plan of care

A. Orient the client to his room..The greatest risk to this client is injury from unfamiliar surroundings. Therefore, the priority action is to orient the client to his room. Before the nurse leaves the room, the client should know how to use the call light and other equipment at the bedside The nurse should conduct a client care conference, review medical prescriptions, and develop a plan of care. However, orienting the client to his room is the priority

A nurse is preparing a hospitalized patient for a colonoscopy. Which nursing action is the recommended preparation for this test? A. Have the patient follow a clear liquid diet 24-48 hours before the test B. Have the patient take Dulcolax and ingest a gallon of bowel cleaner on day one C. Prepare the patient for the use of general anesthesia during the test D. Explain that barium contrast mixture will be given to drink before the test

A. Preparation for a colonoscopy includes a clear liquid diet 24-48 hours before the test along with a 2 day bowel prep of a strong cathartic and Dulcolax on day 1 and an enema on day 2 of the test, or a 1 day bowel prep that consists of ingestion of a gallon of bowel cleanser in a short period of time. Conscious sedation, not general anesthesia, will be given for the colonoscopy. A chalky tasting barium contrast mixture is given to drink before an upper gastrointestinal and small bowel series of tests

A nurse is caring for a patient who is taking phenazopyridine (Pyridium, a urinary tract analgesic). The patient questions the nurse: "My urine was bright orangish-red today; is there something wrong with me?" What would be the nurse's best response? A. This is a normal finding when taking phenazopyridine. B. This may be a sign of blood in the urine C. This may be the result of an injury to your bladder D. This is a sign that you are allergic to the medication and must stop it

A. Pyridium is noted for turning the urine orange-red; the patient needs to be aware of this

The nurse is preparing to catheterize a patient who is incontinent of urine following bladder surgery. What fact should the nurse keep in mind when performing catheterization? A. The bladder normally is a sterile cavity B. The external opening to the urethra should always be sterilized C. Pathogens introduced into the bladder remain in the bladder D. A normal bladder is as susceptible to infection as an injured one

A. The bladder normally is a sterile cavity

A nurse is caring for a male patient who had a condom catheter applied following hip surgery. What action would be a priority when caring for this patient? A. Preventing the tubing from kinking to maintain free urinary drainage B. Not removing the catheter for any reason C. Fastening the condom tightly to prevent the possibility of leakage D. Maintaining bedrest at all times to prevent the catheter from slipping off

A. The catheter should be allowed to drain freely through tubing that is not kinked. It also should be removed daily to prevent skin excoriation and should not be fastened too tightly or restriction of blood vessels in the area is likely. Confining a patient to bedrest increases the risk for other hazards related to immobility

A nurse is feeding a patient who states that she is feeling nauseated and can't eat what is being offered. What would be the most appropriate initial action of the nurse in this situation? A. Remove the tray from the room B. Administer an antiemetic and encourage the patient to take small amounts C. Explore with the patient why she does not want to eat her food D. Offer high calorie snacks such as pudding and ice cream

A. The first action of the nurse when a patient has nausea is to remove the tray from the room. The nurse may then offer small amounts of foods and liquids such as crackers or ginger ale. The nurse may also administer a prescribed antiemetic and try small amounts of food when it takes effect.

A 62 year old male patient has been admitted to the the alcoholic referral unit in the local hospital. Based on an understanding of the effects of alcohol on the GI tract, which is the priority concern related to nutrition? A. Vitamin B malnutrition B. Obesity C. Dehydration D. Vitamin C deficiency

A. The need for B vitamins is increased in alcoholics because these nutrients are used to metabolize alcohol, thus depleting their supply. Alcohol abuse specifically affects the B vitamins. Obesity, dehydration, and vitamin C deficiency may be present, but these are not directly related to the effect of alcohol on the GI tract

A nurse is administering a pain medication to a patient. In addition to checking his identification bracelet, the nurse correctly verifies his identity by: A. Asking the patient his name B. Reading the patient's name on the sign over the bed C. Asking the patient's roommate to verify his name D. Asking, "Are you Mr. Brown?"

A. The nurse should ask the patient to state his name. A sign over the patient's bed may not always be current. The roommate is an unsafe source of information. The patient may not hear his name but may reply in the affirmative anyway (ex. a person with a hearing deficit)

A nurse is assessing the abdomen of a patient who is experiencing frequent bouts of diarrhea. The nurse first observes the contour of the abdomen, noting any masses, scars, or areas of distention. What action would the nurse perform next? A. Auscultate the abdomen using an orderly clockwise approach in all abdominal quadrants. B. Percuss all quadrants of the abdomen in a systematic clockwise manner to identify masses, fluid, or air in the abdomen. C. Lightly palpate over the abdominal quadrants; first checking for any areas of pain or discomfort D. Deeply palpate over the abdominal quadrants, noting muscular resistance, tenderness, organ enlargement, or masses

A. The sequence for abdominal assessment proceeds from inspection, auscultation, percussion, then palpation. Inspection and auscultation are performed before palpation because palpation may disturb normal peristalsis and bowel motility. Percussion and deep palpation are usually performed by advance practice professionals

A home care nurse is teaching a patient with diabetes how to self- administer insulin. Which teaching point would the nurse include in the teaching plan? A. Use the same area of the body at the same time every day. B. Use the same site on the body for each injection. C. Reuse syringes and needles up to three times D. Store needles and syringes in a glass container

A. Use the same area of the body at the same time every day

A nurse is caring for a newly placed gastrostomy tube of a postoperative patient. Which nursing action is performed correctly? A. The nurse dips a cotton tipped applicator into sterile saline solution and gently cleans around the insertion site. B. The nurse wets a washcloth and washes the area around the tube with soap and water. C. The nurse adjusts the external disk every 3 hours to avoid crusting around the tube. D. The nurse tapes a gauze dressing over the site after cleansing it

A. When caring for a new gastrostomy tube, the nurse would use a cotton tipped applicator dipped in sterile saline to gently cleanse the area, removing any crust or drainage. The nurse would not use a washcloth with soap and water on a new gastrostomy tube, but may use this method if the site is healed. Also, once the sutures are removed, the nurse should rotate the external bumper 90 degrees once a day. The nurse should leave the site open to air unless there is drainage. If there is drainage, one thickness of precut gauze should be placed under the external n=bumper and changed as needed to keep the area dry.

The nurse is assessing a female patient who states that she notices an involuntary loss or urine following a coughing episode. What would be the nurse's best reply? A. You are experiencing stress incontinence. Do you know how to do Kegel exercises? B. Your are experiencing a reflex incontinence. Have you had a spinal cord injury in the past? C. You are experiencing total incontinence. Have you had any surgeries or trauma that may be causing this? D. You are experiencing transient incontinence. Have you been administered diuretics or IV fluids lately?

A. You are experiencing stress incontinence. Do you know how to do Kegel exercise?

A patient is scheduled for surgery and the nurse is teaching the patient about the importance of vitamin C in wound healing. Which sources of vitamin C should the nurse include in the teaching plan? (Select all that apply.) A. Potatoes B. Papayas C. Yogurt D. Beans E. Milk

A.B. Potatoes and papayas are both excellent sources of vitamin C (ascorbic acid). A medium potato contains approximately 42 mg and 1 cup of papaya contains 87 mg.

A public health nurse is planning a health class about herbal remedies for a group of older adults at the community center. Which information about herbal remedies should the nurse include in the class? (Select all that apply.) A. Can cause serious herbal-to-drug interactions with prescribed medications B. Required to be labeled with information about their structure C. Approved by the FDA D. Natural because they are botanical in origin E. Safe because they are organic

A.B. Some herbal supplements can cause dangerous herbal-to-drug interactions or nontherapeutic responses when taken concurrently with OTC or prescribed medications. The Dietary Supplemental Health and Education Act of 1994 stipulated that herbs must be labeled with information about their effects on the structure and function of the body. Herbal substances officially are considered food. The FDA does not regulate herbal remedies.

A patient is receiving a diuretic that contributes to the loss of potassium, and the nurse provides dietary teaching. Which foods selected by the patient indicate an understanding of excellent sources of potassium? (Select all that apply.) A. Cooked spinach B. Baked potato C. Green beans D. Bran flakes E. Lean meat

A.B. Spinach and baked potatoes are excellent sources of potassium

A scrub nurse is assisting a surgeon with a kidney transplant. What are the patient responsibilities of this surgical team member? (Select all that apply) A. Maintaining a sterile technique B. Draping and handling instruments and supplies C. Identifying and assessing the patient on admission D. Integrating case management E. Preparing the skin at the surgical site F. Providing exposure of the operative area

A.B. The scrub nurse is a member of the sterile team sho maintains sterile technique while draping and handling instruments and supplies. Two duties of the circulating nurse are to identify and assess the patient on admission to the operating room and prepare the skin at the surgical site. The RNFA actively assists the surgeon by providing exposure of the operative area. The APRN coordinate care activities, collaborates with physicians and nurses in all phases of perioperative and postanesthesia care, and integrates case management, critical paths, and research into care of the surgical patient

A primary healthcare provider prescribes a medication that must maintain effective blood concentrations of the drug to be effective. Which information is essential for the nurse to know about this medication to ensure its effectiveness? (Select all that apply.) A. Therapeutic range B. Trough level C. Peak level D. Half-life E. Onset

A.B.C. Knowing a drug's therapeutic range is essential. It indicates the lowest blood concentration level that is effective and the highest blood concentration level that is effective without causing toxicity. A trough level reflects the lowest plasma concentration of a drug in the patient's body. The nurse must determine if a trough level is within the drug's therapeutic range. A trough plasma level is determined by a blood test that assesses the level of the drug administration of a prescribed dose of the medication. A peak level reflects the highest plasma concentration of a drug in the patient's body. The nurse must determine if the peak level is high enough to be within the therapeutic range yet not too high to be toxic. A peak plasma level is determined by a blood test that assesses the level of the drug in the patient's body 30 minutes to hour after administration of the medication.

A nurse in a senior center is counseling a group of older adults about their nutritional needs and considerations. Which of the following information should the nurse include? (Select all that apply) A. Older adults are more prone to dehydration than younger adults are B. Older adults need the same amount of most vitamins and minerals as younger adults do C. Many older men and women need calcium supplementation D. Older adults need more calories that they did when they were younger. E. Older adults should consume a diet low in carbohydrates

A.B.C. Older adults are more prone to dehydration than younger adults are. Sensations of thirst diminish with age, leaving older adults more prone to dehydration. Older adults need the same amount of most vitamins and minerals as younger adults do. These requirement do not change from middle adulthood to older adulthood. Many older men and women need calcium supplementation. If older adults ingest insufficient calcium in the diet, they need supplements to help prevent bone demineralization (osteoporosis) Older adults have a slower metabolic rate, so they require less energy (unless they are very active), and therefore need fewer calories. Many older adults need more carbohydrates for the fiber and bulk they contain. They should, however, reduce their intake of fats and of empty calories, such as pastries and soda pop

The nurse is administering medication to a patient through a drug infusion lock using a saline flush. During the process, the patient complains of pain at the site. Which interventions are appropriate in this situation? (Select all that apply) A. Stop the medication and assess the site for signs of infiltration and phlebitis B. Flush the medication lock with normal saline again to recheck patency C. If site is within normal limits, resume medication administration at a slower rate. D. Immediately stop the medication, remove lock, and restart at new site. E. Notify the primary care provider that the site has been infiltrated F. Finish administering medication and then change the medication lock

A.B.C. Stop the medication and assess the site for signs of infiltration and phlebitis Flush the medication lock with normal saline again to recheck patency. If site is within normal limits, resume medication administration at a slower rate

A nurse is preparing to administer a cleansing enema to an adult client in preparation for a diagnostic procedure. Which of the following steps should a nurse take? (Select all that apply) A. Warm the enema solution prior to instillation B. Position the client on the left side with the right leg flexed forward C. Lubricate the rectal tube or nozzle D. Slowly insert the rectal tube about 5cm (2inches) E. Hang the enema container 61cm (24inches) above the client's anus

A.B.C. The nurse should warm the enema solution because cold fluid can cause abdominal cramping, and hot fluid can injure the intestinal mucosa. The nurse should place the client in this position to promote a downward flow of solution by gravity along the natural anatomical curve of the sigmoid colon. The nurse should lubricate the tubing to prevent trauma or irritation to the rectal mucosa. The correct length of insertion for a child is 5cm. For an adult client, the nurse should insert the tube 7.6-10cm (3-4inches). The maximum recommended height is 46cm (18inch). The height of the fluid container affects the speed of instillation. The nurse should hang the container within the recommended height range to prevent rapid instillation and possibly painful distention of the colon.

In today's environment, nurses must be prepared to use CAT in their nursing practices. Which statements accurately represent the role of CAT in nursing today? (Select all that apply) A. The nursing profession is expanding its knowledge base to include information that explains selected CAT. B. Certification is available for nurses wishing to practice holistic nursing C. Graduate level specialization in holistic nursing is available at some universities. D. The development of CAT is market and patient driven E. It is expected that CAT will eventually replace traditional nursing in many health care facilities F. Practitioners of CAT are strictly regulated by the government

A.B.C.D. The nursing professing is expanding its knowledge base to include information that explains selected CAT. Certification is available for nurses wishing to practice holistic nursing. Graduate level specialization in holistic nursing is available at some universities. The development of CAT is market and patient driven

A nurse is admitting a client who has acute cholecystitis to a medical surgical unit. Which of the following actions are essential steps of the admission procedure? (Select all that apply) A. Explain the roles of the other care delivery staff B. Begin discharge planning C. Inform the client that advance directives are required for hospital admission D. Document the client's wishes about organ donation E. Introduce the client to his roommate

A.B.C.D.E. The client's hospitalization is likely to be more positive if the client understands who can perform which care activities. Unless the client is entering a long term care facility, discharge planning should begin on admission.The Patient Self-Determination Act does not require that clients have advance directives prior to hospital admission. The act requires asking clients if they have advance directives. Upon hospital admission, required request laws direct providers to ask clients older than 18 years if they are organ or tissue donors. Any action that can reduce the stress of hospitalization is therapeutic. Introductions to other clients and staff can encourage communication and psychological comfort

An older adult asks the nurse, "I want to make sure I get enough vitamin A to keep my eyes healthy. Which fruits can I eat because I am not fond of vegetables? Which fruits should the nurse explain are excellent sources of vitamin A? (Select all that apply.) A. Cantaloupe B. Apricots C. Peaches D. Raisins E. Prunes

A.B.C.E. Cantaloupe, apricots, peaches, and prunes are excellent sources of vitamin A.

A nurse uses healing touch to care for patients on a hospital ward. For which patients would this practice be most appropriate? (Select all that apply) A. A patient with a surgical wound B. A patient who has unrelenting pain C. A patient whose energy field is unbalanced D. A patient who is overweight E. A patient who leaves the hospital against medical advice. F. A patient who is being prepared for a surgical procedure

A.B.C.F. A patient with a surgical wound. A patient who has unrelenting pain. A patient whose energy field is unbalanced. A patient who is being prepared for a surgical procedure

Active Learning Scenario: Chapter 47 List at least three acceptable identifiers to use to verify the clients identity.

Acceptable identifiers include the clients name, an assigned identification number, telephone number, birth date, or other person specific identifier The nurse can use bar code scanners to identify clients

A nurse cares for patients in a chiropractic office. What patient education might this nurse perform? (Select all that apply) A. Applying heat or ice to an extremity B. Explaining the use of electrical stimulation C. Teaching a patient relaxation techniques D. Teaching a patient about a prescription E. Explaining an invasive procedure to a patient F. Teaching about dietary supplements

A.B.C.F. Chiropractors may combine the use of spinal adjustments and other manual therapies with several other treatments and approaches including heat and ice, electrical stimulation, relaxation techniques, rehabilitative and general exercise, counseling about weight, and using dietary supplements. Chiropractors do not prescribe medications or perform invasive procedures

What are components of a medication order? (Select all that apply) A. The full name of the patient B. The date and sometimes the time when the order is written C. Preferably the brand name of the drug to be administered D. The dosage of the drug stated in either the apothecary or metric system E. The route by which the drug is to be administered, only if there is more than one route possible F. The signature of the nurse carrying out the order

A.B.D. The full name of the patient The date and sometimes the time when the order is written The dosage of the drug stated in either the apothecary or metric system.

A patient has a prescription for a vaginal suppository. Which actions should the nurse perform when administering this medication? (Select all that apply.) A. Lubricate the suppository and the index finger of a gloved hand before insertion of the suppository B. Instruct the patient to remain flat in the bed for 20 minutes after insertion of the suppository C. Irrigate the vagina with normal saline before inserting the suppository D. Place the patient in the dorsal recumbent position for the procedure E. Advance the suppository along the posterior vaginal wall F. Insert the suppository while wearing clean gloves

A.B.D.E. A-facilitates insertion and limits trauma to vaginal mucous membranes. B-facilitates absorption. D-facilitates insertion. E-facilitates placement just outside of the cervical os so that when it melts it will eventually disperse through the entire vaginal wall.

Which factors would the nurse consider when assessing surgical patients following surgery? (Select all that apply) A. Infants are at a greater risk from surgery than are middle aged adults. B. Infants experience a slower metabolism of drugs that require renal biotransformation. C.Muscle relaxants and narcotics have a shorter duration of action in infants D. Older adults have decreased renal blood flow and a reduced bladder capacity, necessitating careful monitoring of fluid and electrolyte status and input and output E. Older adults have an increased gastric pH and require monitoring of nutritional status during the perioperative period F. Older adults have an increased hepatic blood flow, liver mass, and enzyme function that prolongs the duration of medication effects

A.B.D.E. Infants are at a greater risk from surgery than are middle aged adults. Infants experience a slower metabolism of drugs that require renal biotransformation. Older adults have decreased renal blood flow and a reduced bladder capacity, necessitating careful monitoring of fluid and electrolyte status. Older adult have an increased gastric pH and require monitoring of nutritional status during the perioperative period

The nurse is assessing a patient's bladder volume using an ultrasound bladder scanner. Which nursing actions are performed correctly?(Select all that apply) A. The nurse gently palpates the patient's symphysis pubis. B. The nurse places a generous amount of ultrasound gel or gel pad midline on the patient's abdomen, about 1 to 1.5 inches above the symphysis pubis C. The nurse places the scanner head on the gel or gel pad, with the directional icon on the scanner head pointed away from the patient's head D. The nurse aims the scanner head toward the bladder (points the scanner head downward toward the coccyx). E. The nurse adjusts the scanner head to center the bladder image on the crossbars F. The nurse presses and holds the END button until it beeps three times and then read the volume measurement on the screen

A.B.D.E. The nurse gently palpate the patient's symphysis pubis. The nurse places a generous amount of gel or gel pad midline on the patient's abdomen, about 1 to 1.5inches above the symphysis pubis. The nurse aims the scanner head toward the bladder (points the scanner head slightly downward toward the coccyx) The nurse adjust the scanner head to center the bladder image on the crossbars

A nurse is caring for a client who had a stroke is scheduled for transfer to a rehabilitation center. Which of the following tasks are the responsibility of the nurse at the transferring facility? (Select all that apply) A. Ensure that the client has possession of his valuables B. Confirm that the rehabilitation center has a room available at the time of transfer C. Assess how the client tolerates the transfer D. Give a verbal transfer report via telephone E. Complete a transfer form for the receiving facility

A.B.D.E. The nurse should account for all of the client's valuables at the time of transfer. On the day of the transfer, the nurse should confirm that the receiving facility is expecting the client and that the room is available. The nurse should provide the nurse at the receiving facility with a verbal transfer report in person or via telephone. The nurse should complete any documentation for the transfer, including the transfer form and the client's medical records. It is the responsibility of the nurse at the receiving facility to assess the client upon arrival to determine how he tolerated the transfer

A nurse is evaluating patients to determine their need for total parenteral nutrition (TPN) Which patients would be the best candidates for this type of nutritional support? (Select all that apply) A. A patient with irritable bowel syndrome who has intractable diarrhea B. A patient with celiac disease not absorbing nutrients from the GI tract C. A patient who is underweight and needs short term nutritional support D. A patient who is comatose and needs long term nutritional support E. A patient who has anorexia and refuses to take foods via the oral route F. A patient with burns who has not been able to eat adequately for 5 days

A.B.F. Assessment criteria used to determine the need for TPN include inability to achieve or maintain enteral access; motility disorders; intractable diarrhea; impaired absorption of nutrients from the GI tract (patient with celiac disease) and when oral intake has been or is expected to be inadequate over a 7 to 14 day period. TPN promotes tissue healing and is a good choice for a patient with burns who has an inadequate diet. Oral intake is the best method of feeding; the second best method is via the enteral route. For short term use (less than 4 weeks), a nasogastric or nasointestinal route is usually selected. A gastrostomy (enteral feeding) is the preferred route to deliver enteral nutrition in the patient who is comatose because the gastroesophageal sphincter remains intact, making regurgitation and aspiration less likely than with NG tube feedings. Patients who refuse to take food should not be force fed nutrients against their will.

A nurse is preparing medications for a patients in the ICU. The nurse is aware that there are patient variables that may affect the absorption of these medications. Which statements accurately describe these variables? (Select all that apply) A. Patients in certain ethnic groups obtain therapeutic responses at lower doses or higher doses than those usually prescribed B. Some people experience the same response with a placebo as with the active drug used in studies C. People with liver disease metabolize drugs more quickly than people with normal liver functioning D. A patient who receives a pain medication in a noisy environment may not receive full benefit from the medication's effects E. Oral medications should not be given with food as the food may delay the absorption of the medications F. Circadian rhythms and cycles may influence drug action.

A.B.D.F. Nurses need to know about medications that may produce varied responses in patients from different ethnic groups. The patient's expectations of the medication may affect the response to the medication, for example, when a placebo is given and a patient has a therapeutic effect. The patient's environment may also influence the patient's response to medications, for example, sensory deprivation and overload may affect drug responses. Circadian rhythms and cycles may also influence drug action. The liver is the primary organ for drug breakdown, thus pathologic conditions that involve the liver may slow metabolism and alter the dosage of the drug needed to reach a therapeutic level. The presence of food in the stomach can delay the absorption of orally administered medications. Alternately, some medications should be given with food to prevent gastric irritation, and the nurse should consider this when establishing a patient's medication schedule. Other medications may have enhanced absorption if take with certain foods.

Which examples accurately describe significant abnormal findings related to presurgical screening tests that the nurse should report to the surgeon? A. An elevated white blood count, indicating an infection B. Decreased hematocrit and hemoglobin level, indicating bleeding or anemia C. Increased hyperkalemia or hypokalemia, indicating possible renal failure D. Elevated blood urea nitrogen or creatinine levels, indicating an increased risk for cardiac problems E. Abnormal urine constituents, including infection or fluid imbalances F. Increased hemoglobin level, indicating infection

A.B.E. An elevated white blood count, indicating an infection. Decreased hematocrit and hemoglobin level, indicating bleeding or anemia. Abnormal urine constituents, including infection or fluid imbalances.

A nurse educator is teaching a module about safe medication administration to newly licensed nurses. Which of the following statements should the nurse identify as an indication that one of the group understands how to implement medication therapy? (Select all that apply) A. I will observe for side effects B. I will monitor for therapeutic effects C. I will prescribe the appropriate dose D. I will change the dose if adverse effects occur E. I will refuse to give a medication if I believe it is unsafe

A.B.E. I will observe for side effects- The nurse is responsible for observing for side effects.This is within a nurse's scope of practice. The nurse is responsible for monitoring therapeutic effects. This is within a nurse's scope of practice. The nurse is responsible for identifying when a medication could harm a client. It is within the nurse's scope of practice to refuse to administer the medication and contact the provider. The provider is responsible for prescribing the appropriate dose. This is outside the nurse's scope of practice. The provider is responsible for changing the dose if adverse effects occur. This is outside the nurse's scope of practice

A nurse is teaching a client who has a new prescription for oxybutynin about managing the medication's anticholinergic effects. Which of the following instructions should the nurse include? (Select all that apply) A. Take sips of water frequently B. Wear sunglasses when outdoor in sunlight C. Use a soft toothbrush when brushing teeth D. Take the medication with an antacid E. Urinate prior to taking the medication

A.B.E. Taking sips of water frequently will help relieve the anticholinergic effect of dry mouth. Wearing sunglasses will help relieve the anticholinergic effect of photophobia. Urinating prior to taking the medication will help relieve the anticholinergic effect of urinary retention. Anticholinergic effects do not increase the client's risk for bleeding. Constipation is an example of an anticholinergic effect. Taking the medication with an antacid will not decrease the anticholinergic effects.

A nurse mentor is teaching a new nurse about the underlying beliefs of CAT vs allopathic therapies. Which statements by the new nurse indicate that teaching was effective? Select all that apply. A.CAT proponents believe the mind, body, and spirit are integrated and together influence health and illness. B. CAT proponents believe that health is a balance of body systems: mental, social, and spiritual, as well as physical C. Allopathy proponents believe that the main cause of illness is an imbalance or disharmony in the body systems. D. Curing according to CAT proponents seeks to destroy invading organism or repair the affected part E. The emphasis is on disease for allopathic proponents and drugs, surgery, and radiation are key tools for curing F. According to CAT proponents, health is the absence of disease

A.B.E. With CAT, mind, body, and spirit are integrated and together influence health and illness, and illness is a manifestation of imbalance or disharmony. Allopathic beliefs include: the main causes of illness are considered to be pathogens (bacteria or viruses) or biochemical imbalances, curing seeks to destroy the invading organism or repair the affected part, and emphasis is on disease and high technology. Drugs, surgery, and radiation are among the key tools for dealing with medical problems. According to allopathic beliefs, health is the absence of disease.

A nurse who has incorporated complementary and alternative therapies (CAT) into nursing practice is caring for a patient in a short term care facility. Which examples of nursing interventions are based on CAT? (Select all that apply) A. The nurse investigates herbs that may stimulate the patient's immune system. B. The nurse encourages the patient to join a yoga class C. The nurse administers pain medication prescribed by the primary care provider D. The nurse schedules diagnostic tests for the patient E. The nurse teaches the patient how to meditate F. The nurse uses guided imagery to relieve patient anxiety

A.B.E.F. The nurse investigates herbs that may stimulate the patient's immune system. The nurse encourages the patient to join a yoga class. The nurse teaches the patient how to meditate, The nurse uses guided imagery to relieve patient anxiety

A nurse working in a hospital includes abdominal assessment as part of a patient assessment. In which patients would a nurse expect to find decreased or absent bowel sounds after listening for five minutes? (Select all that apply) A. A patient diagnosed with peritonitis B. A patient who is on prolonged bedrest C. A patient who has diarrhea D. A patient who has gastroenteritis E. A patient who has an early bowel obstruction F. A patient who has paralytic ileus caused by surgery

A.B.F. Decreased or absent bowel sounds- evidenced only after listening for 5 minutes- signify the absence of bowel motility, commonly associated with peritonitis, paralytic ileus, and/or prolonged immobility. Hyperactive bowel sounds indicate increased bowel motility, commonly caused by diarrhea, gastroenteritis, or early bowel obstruction.

Which routes are unrelated to the parental administration of medications? (Select all that apply.) A. Buccal B. Z-track C. Sublingual D. Intravenous E. Intradermal

A.C. A parental route is outside the GI tract. A medication administered by the buccal route dissolves between the cheeks and gums, where it acts on the oral mucous membranes or is swallowed with saliva. Most troches are used for their local effect. The mucosal route of administration includes the nasal mucosa, buccal mucosa, sublingually, and the bronchioles. The sublingual route is a mucosal route of administration.

A nurse is to administer an eye irrigation to a patient's right eye. Which should the nurse do? (Select all that apply.) A. Direct the flow of solution from the inner to the outer canthus. B. Irrigate with a bulb syringe held several inches above the eye. C. Expose the conjunctival sac and hold open the upper lid. D. Don sterile gloves before beginning the procedure. E. Position the patient in a right lateral position.

A.C. Directing the flow of solution from inner to the outer canthus prevents secretions and fluid from entering and irritating the lacrimal ducts. Exposing the conjunctival sac and holding the upper lid provide access to the eye.

A primary healthcare provider prescribes a rectal suppository for an adult patient. Which actions should the nurse implement when administering the rectal suppository? (Select all that apply.) A. Lubricate the medication before insertion. B. Warm the medication equal to the body temperature. C. Instruct the patient to take deep breaths through the mouth. D. Insert the medication just inside the rectum's external sphincter. E. Place the patient in the prone position to administer the medication.

A.C. Lubrication eases insertion by reducing friction, which limits tissue trauma and discomfort. Taking deep breaths relaxes the rectal sphincters.

Which of the following commonly used enema solutions would the nurse administer to distend the intestine and increase peristalsis? (Select all that apply) A. Tap Water B. Soap C. Normal saline D. Mineral oil E. Hypertonic F. Olive oil

A.C. Tap water. Normal saline

What information must be provided to a patient to obtain informed consent? (Select all that apply) A. A description of the procedure or treatment, along with potential alternative therapies B. The name and qualifications of the nurse providing perioperative care C. The underlying disease process and its natural course D. Explanation of the risks involved and how often they occur E. Explanation that a signed consent form is binding and cannot be withdrawn F. Customary insurance coverage for the patient

A.C.D. A description of the procedure or treatment, along with potential alternative therapies. The underlying disease process and its natural course. Explanation of the risks involved and how often they occur

The nurse is assessing adequate nutrition for residents of a long term care facility. Which of strategies are recommended to address age related changes affecting nutrition? (Select all that apply) A. Avoid cold liquids with decreased peristalsis in the esophagus B.Serve a variety of foods at each meal for loss of sense of taste and smell. C. Avoid eating right before bedtime for gastroesophageal reflux D. Eat a high fiber diet for slowed intestinal peristalsis E. Eat more protein for lowered glucose tolerance F. Offer large meals at frequent intervals for reduction in appetite and thirst sensation

A.C.D. Avoid cold liquids with decreased peristalsis in the esophagus. Avoid eating right before bedtime for gastroesophageal reflux. Eat a high fiber diet for slowed intestinal peristalsis

A nurse is teaching an overweight patient the holistic approach to choosing foods. Which teaching points would the nurse include? (Select all that apply) A. Eat foods that are in season B. Avoid organically grown foods C. Reduce intake of refined and natural sugars D. Consider adopting a vegetarian diet. E. increase intake of dairy products F. Replace refined sugars with artificial sweeteners

A.C.D. Eat foods that are in season. Reduce the intake of refined and natural sugars. Consider adopting a vegetarian diet

Which nursing actions follow guidelines for preventing complications with enteral feedings? (Select all that apply) A. Elevate the head of the bed at least 30 degrees during the feeding and for at least 1 hour afterward B. Give large, infrequent feedings. C. Flush the tube before and after feeding D. Clean and moisten the nares every 4 to 8 hours E. Change the delivery set every other day according to agency policy F. Check the residual before intermittent feedings and every 8 hours during continuous feedings

A.C.D. Elevate the head of the bed at least 30 degrees during the feeding and for at least 1 hour afterward. Flush the tube before and after feeding. Clean and moisten the nares every 4 to 8 hours

A primary healthcare provider prescribes a liquid oral medication for a patient with a nasogastric tube on low continuous suction. Which actions should the nurse implement when administering this medication? (Select all that apply) A. Give each medication separately. B. Follow medication administration with 100 mL of free water. C. Crush crushable tablets into a fine powder and mix with 30 mL of warm water. D. Shut off nasogastric tube suctioning for 30 minutes after medication administration. E. Ensure nasogastric tube placement by instilling 30 mL of air while auscultating over the epigastric area for a "whoosing" sound.

A.C.D. If the tube used to administer a medication via a nasogastric (NG) tube becomes accidently disconnected during administration, the nurse can identify the approximate volume of the one medication that was lost when reporting the event to the primary healthcare provider. Crushing crushable tablets into a fine powder and missing it with 30 mL of warm water dissolve the medication and prevents clogging the enteral tube. Shutting off NG tube suctioning for 30 minutes after medication administration enhances medication absorption in the stomach.

Midazolam (Versed), 3mg IM, is ordered. Midazolam is available as 5mg/mL. How much would the nurse administer?

Dose on hand = . Dose desired --------------- --------------- Quantity on hand . Quantity desired 5mg = 3mg ------ ------ 1mL . X 5X= 3 X= 0.6mL

A nurse is teaching a group of nursing students on complementary and alternative therapies they can incorporate into their practice without the need for specialized licensing or certification. Which of the following should the nurse encourage the students to use? (Select all that apply) A. Guided imagery B. Massage therapy C. Meditation D. Music therapy E. Therapeutic touch

A.C.D. Nurses may use guided imagery with clients once they understand the general principles of this therapy. Nurses may use meditation with clients once they understand the general principles of this therapy. Nurses may use music therapy with clients once they understand the general principles. The nurse should inform the students that massage therapists undergo training as well as certification and/or licensure. The nurse should inform the students that therapeutic touch practitioners undergo specific training

A primary healthcare provider prescribes an oral medication for a patient. The nurse identifies that the patient is having some difficulty swallowing. What should the nurse plan to do? (Select all that apply.) A. Crush tablets that are crushable and mix a small amount of applesauce. B. Have the patient hyperextend the neck slightly when swallowing. C. Give water before, during, and after medication administration. D. Stroke under the chin over the larynx. E. Have the patient use a straw.

A.C.D. Reducing the size of a tablet and mixing it with a food the consistency of applesauce facilitate ingestion and minimize the risk of aspiration. The thickness of applesauce is easier to control in the mouth than water for a person who has difficulty swallowing. Giving fluid before, during, and after medication administration lubricates the oral cavity and facilitates movement of medication toward the esophagus and stomach. Stroking the chin over the larynx encourages laryngeal elevation, which facilitates swallowing.

A nurse is planning to apply a transdermal patch to a patient. Which actions should the nurse implement? (Select all that apply.) A. Use different sites each time to limit skin irritation and excoriation B. Rub the area to promote comfort and vasodilation before applying the patch C. Shave the area to facilitate adherence of the patch and medication absorption D. Wear clean gloves to protect one's self from absorbing the medication through the hands E. Remove the old patch an hour after applying the new patch to ensure a therapeutic blood level of the drug

A.C.D. Sites should be rotated to limit skin irritation and to allow time for the site to recover if irritated. A hairless site ensures there's effective contact with the skin. Clean gloves provide a barrier of protection from the nurse absorbing some of the medication.

Which actions would the nurse take when instilling eyedrops correctly? (Select all that apply) A. Wash hands and put on gloves B. Clean the eyelids and eyelashes of any drainage with cotton balls soaked in clean water C. Tilt the patient's head back slightly if sitting or place the head on a pillow if lying down D. Have the patient look up and focus on something on the ceiling E. Place the thumb near the margin of the lower eyelid and exert pressure upward over the bony prominence of the cheek F. Squeeze the container and allow the prescribed number of drops to fall into the cornea

A.C.D. Wash hands and put on gloves. Tilt the patient's head back slightly if sitting or place the head on a pillow of lying down Have the patient look up and focus on something on the ceiling

A nurse is administering intramuscular injections to patients on a hospital ward. What needle sizes has the nurse used correctly? (Select all that apply) A. 5/8 inch needle for the vastus lateralis site B. 5/8 inch needle for an adult in the ventrogluteal site C. 1 1/4 inch needle for a child in the deltoid site D. 1 1/2inch needle for an adult in the deltoid site E. 5/8 inch needle for a child in the deltoid site F. 5/8 inch needle for an adult in the ventrogluteal site

A.C.D.E. 5/8 inch needle for the vastus lateralis site 1 1/4 inch needle for a child in the deltoid site 1 1/2 inch needle for an adult in the deltoid site 5/8 inch needle for a child in the deltoid site

A nurse is collecting a stool specimen from a patient. Which measures are appropriate for this procedure? (Select all that apply) A. The patient should be asked to void first because the lab study may be inaccurate if the stool contains urine B. The patient should be asked to defecate into a clean bedpan or toilet bowl, depending on the nature of the study. C. The patient should be instructed not to place toilet tissue in the bedpan or specimen container D. Medical aseptic techniques are always followed E. Handwashing is performed before and after glove use when handling a stool specimen F. Generally, 2 inches of formed stool or 20 to 30mL of liquid stool is sufficient for a stool specimen

A.C.D.E. The patient should be asked to void first because the lab study may be inaccurate if the stool contains urine. The patient should be instructed not to place toilet tissue in the bedpan or specimen container. Medical aseptic techniques are always followed. Handwashing is performed before and after glove use when handling a stool specimen

A nurse is interviewing a newly admitted patient in the process of completing a nursing admission history and physical. Which information should be included in a medication reconciliation form? (Select all that apply.) A. Vitamins B. Drug allergies C. Food supplements D. Over-the-counter herbs E. Prescribed medications

A.C.D.E. Vitamins are a medication and should be included on a MAR form. An accurate list of all the drugs that a patient is taking should be reconciled on admission and during transitions (e.g., transfers between units, shift reports, when new MARs are implemented, and upon discharge). Food supplements are considered medications because they often contain ingredients that may interact with medicinal products. OTC herbs are considered medications because they contain ingredients that may unfavorably interact with medicinal products. Prescribed medications should be included on a medication reconciliation form.

The nurse is changing a stoma appliance on a ileal conduit. Which of the following nursing actions are recommended procedure? (Select all that apply) A. Gently remove the appliance, starting at the top and keeping the abdominal skin taut. B. Remove appliance faceplate by pulling appliance from skin rather than pushing C. Apply a silicone based adhesive remover by spraying or wiping as needed D. Clean skin around stoma with alcohol on a gauze pad. E. Make sure skin around stoma is thoroughly dry by patting it dry F. Apply faceplate by using firm, even pressure for approximately 60 seconds.

A.C.E. Gently remove the appliance, starting at the top and keeping the abdominal skin taut. Apply a silicone based adhesive remove by spraying or wiping as needed. Make sure skin around stoma is thoroughly dry by patting it dry.

A nurse is teaching an adolescent patient how to use a meter dosed inhaler to control his asthma. What are appropriate guidelines for this procedure? (Select all that apply) A. Remove the mouthpiece cover and shake the inhaler well. B. Take shallow breaths when breathing through the spacer. C. Depress the canister releasing one puff into the spacer and inhale slowly and deeply. D. After inhaling, exhale quickly through repressed lips E. Wait 1 to 5 minutes as prescribed before administering the next puff. F. Gargle and rinse with salt water after using the MDI

A.C.E. The correct procedure for using a meter dosed inhaler is: remove the mouthpiece cover and shake the inhaler well; breather normally through the spacer; depress the canister releasing one puff into the spacer and inhale slowly and deeply; after inhaling, hold breath for 5 to 10 seconds, or as long as possible, and then exhale slowly through pursed lips; wait 1 to 5 minures as prescribed before administering the next puff; and gargle and rinse with tap water after using the MDI

A nurse caring for patients in an extended care facility performs regular assessments of the patients' urinary functioning. Which patients would the nurse screen for urinary retention? (Select all that apply) A. A 78 year old male patient diagnosed with an enlarged prostate B. An 83 year old female patient who is on bedrest C. A 75 year old female patient who is diagnosed with vaginal prolapse D. An 89 year old male patient who has dementia E. A 73 year old female patient who is taking antihistamines to treat allergies F. A 90 year old male patient who has difficulty walking to the bathroom

A.C.E. Urinary retention occurs when urine is produced normally but it is not excreted completely from the bladder. Factors associated with urinary retention include medications such as antihistamines, an enlarged prostate, or vaginal prolapse. Being on bedrest, having dementia, and having difficulty walking to the bathroom may place patients at risk for urinary incontinence

A nurse is performing a physical assessment of a patient's urinary system. Which nursing actions are appropriate during this assessment? (Select all that apply) A. If using a bedside scanner, the nurse places the patient in a supine position B. The nurse measures the height of the edge of the bladder below the symphysis pubis C. The nurse inspects the urethral orifice for any signs of inflammation, discharge, or foul odor D. The nurse places male patients in the dorsal recumbent position for good visualization of the meatus. E. The nurse retracts the foreskin of an uncircumcised male patient to visualize meatus F. The nurse assesses the patient's urine for color, odor, clarity, and the presence of any sediment

A.C.E.F. If using a bedside scanner, the nurse places the patient in a supine position. The nurse inspects the urethral orifice for any signs of inflammation, discharge, or foul odor. The nurse retracts the foreskin of an uncircumcised male patient to visualize the meatus. The nurse assesses the patient's urine for color, odor, claritiy, and the presence of any sediment.

A nurse is caring for a patient who has a nasogastric tube in place for gastric decompression. Which nursing actions are appropriate when irrigating a nasogastric tube connected to suction? (Select all that apply) A. Draw up 30mL of saline solution into the syringe B. Unclamp the suction tubing near the connection site to instill solution C. Place the tip of the syringe in the tube to gently insert the saline solution D. Place syringe in the blue air vent of a Salem sump or double lumen tube. E. After instilling irrigant, hold the end of the NG tube over an irrigation tray F. Observe for return flow of NG drainage into an available container

A.C.E.F. The nurse irrigating a nasogastric tube connected to suction should draw up 30mL of saline solution (or amount indicated in the order or policy) into the syringe, clamp the suction tubing near the connection site to protect the patient from leakage of NG drainage, place the tip of they syringe in the tube to gently insert the saline solution, then place the syringe in the drainage port, not in the blue air vent of a Salem sump or double lumen tube (the blue air vent acts to decrease pressure built up in the stomach when the Salem sump is attached to suction). After instilling irrigant, hold the end of the NG tube over an irrigation tray or emesis basin, and observe for return flow of NG drainage into an available container

A nurse is reviewing complimentary and alternative therapies with a group of nursing students. The nurse should classify which of the following interventions as a mind body therapy? (Select all that apply) A. Art therapy B. Acupressure C. Yoga D. Therapeutic touch E Biofeedback

A.C.F. Art therapy- The nurse should classify art therapy as a mind body therapy because it allows the client to express unconscious emotions or concerns about their health. The nurse should classify yoga as a mind body therapy because it focuses on achieving well being through exercise, posture, breathing, and meditation. The nurse should classify biofeedback because it increases mental awareness of the body responses to stress. The nurse should classify acupressure is a body based therapy because it focuses on specifically on body structures and systems. The nurse should classify therapeutic touch as an energy therapy because it involves using the hands to balance energy fields.

A responsibility of the nurse is the administration of preoperative medications to patients. Which statements describe the action of these medications? (Select all that apply) A. Diazepam is give to alleviate anxiety B. Ranitidine is given to facilitate patient sedation C. Atropine is used to decrease oral secretions D. Morphine is given to decrease respiratory function E. Cimetidine is given to prevent laryngospasm F. Fentanyl citrate-droperidol is given to facilitate a sense of calm

A.C.F. Sedatives, such as diazepam (Valium), midazolam (Versed), or lorazepam (Ativan) are given to alleviate anxiety and decrease recall of events related to surgery. Anticholinergics, such as atropine and glycopyrrolate (Robinul) are given to decrease pulmonary and oral secretions and to prevent laryngospasm. Neuroleptanalgesic agents, such as fentanyl citrate-droperidol (Innovar) are given to cause a general state of calm and sleepiness. Histamine-2 receptor blockers, such as cimetidine (Tagamet) and ranitidine (Zantac) are given to decrease gastric acidity and volume. Narcotic analgesics, such as morphine, are given to facilitate patient sedation and relaxation and to decrease the amount of anesthetic agent needed.

A nurse is collecting a urine specimen for urinalysis. What factors should the nurse consider when performing this procedure? (Select all that apply) A. Sterile urine specimens may be obtained by catheterizing the patient's bladder. B. A sterile urine specimen is required for a routine urinalysis C. If a woman is menstruating, a urine specimen cannot be obtained for urinalysis D. Strict aseptic technique must be used when collecting and handling urine specimens. E. Urine should be left standing at room temperature for a 24-hour period before being sent to the laboratory

A.D.F. Sterile urine specimens may be obtained by catheterizing the patient's bladder. Strict aseptic technique must be used when collecting and handling urine specimens. A clean catch specimen or urine may be collected in midstream

A nurse is caring for a 56 year old male patient diagnosed with bladder cancer who has a urinary diversion. Which actions would the nurse take when caring for this patient? (Select all that apply) A. Measure the patient's fluid intake and output B. Keep the skin around the stoma moist C. Empty the appliance frequently D. Report any mucous in the urine to the primary care provider E. Encourage the patient to look away when changing the appliance F. Monitor the return of intestinal function and peristalsis

A.C.F. When caring for a patient with a urinary diversion, the nurse should measure the patient's fluid intake and output to monitor fluid balance, change the appliance frequently, monitor the return of intestinal function and peristalsis, keep the skin around the stoma dry, watch for mucous in the urine as a normal finding, and encourage the patient to participate in care and look at the stoma

A nurse us providing for the nutritional needs of several patients. Which problems increase patients' caloric requirements? (Select all that apply.) A. Burns B. Nausea C. Dysphagia D. Pneumonia E. Depression

A.D. Burns interrupt the integrity of the skin and as a result a primary defense against infection is disrupted. The body's metabolic rate increases dramatically in an attempt to repair the infection. Nutrients are required to provide the building blocks for skin cells, white blood cells, and immunoglobulins.

A nurse is administering a lozenge to a patient's buccal area of the mouth. Which should the nurse do? (Select all that apply.) A. Ensure the patient stays awake while the lozenge dissolves B. Instruct the patient to take occasional sips of water C. Place the medication under the patient's tongue D. Alternate the cheeks from one dose to another E. Administer the lozenge just before meals

A.D. If the patient falls asleep the patient may aspirate the lozenge, which can cause airway obstruction. Alternating cheeks when placing a lozenge will limit irritation to the mucous membranes in the buccal area.

Which examples of patients would the nurse expect to have an increase in BMR? (Select all that apply) A. A toddler who is having a growth spurt B. An elderly patient who is in a long term care facility C. A teenager who has been fasting to lose weight D. An adolescent who has a fever E. An adult who is going through an emotional time due to divorce F. An adult who has hypersomnia

A.D.E. A toddler who is having a growth spurt. An adolescent who has a fever. An adult who is going through an emotional time due to divorce

A primary healthcare provider orders a 2-gram sodium diet for a patient. Which fluids should the nurse teach are high in sodium? (Select all that apply.) A. Cocoa B. Seltzer C. Lemonade D. Low-fat milk E. Tomato juice

A.D.E. Cocoa powder, containing non-fat dry milk, contains approximately 173 mg of sodium when mixed with 6 ounces of water. One cup of low-fat milk contains 103 mg and 1 cup of tomato juice contains about 877 mg of sodium.

As a nurse is aspirating the contents during a tube feeding, the nurse finds that the tube is clogged. What would be appropriate nursing intervention in this situation? (Select all that apply) A. Use warm water and gentle pressure to remove the clog. B. Flush with a carbonated beverage such as a cola soft drink C. Use a stylet to unclog the tube D. If necessary, replace the tube E. Ensure that adequate flushing is completed after each feeding F. Administer an antiemetic to the patient

A.D.E. Use warm water and gentle pressure to remove the clog. If necessary, replace the tube. Ensure that adequate flushing is completed after each feeding.

A nurse is assessing patients in a PACU. Which nursing actions would the nurse perform in this phase of the perioperative period? (Select all that apply) A. Prepare the patient for home care B. Inform the patient that surgical intervention is necessary C. Transfer the patient to the recovery room D. Admit the patient to the recovery area E. Assess for complications as the patient emerges from anesthesia F. Arrange for a rehabilitative program for the patient

A.D.E.F. Prepare the patient for home care. Admit the patient to the recovery area. Assess for complications as the patient emerges from anesthesia. Arrange for a rehabilitative program for the patient

A nurse who is administering medications to patients in an acute care setting studies the pharmacokinetics of the drugs being administered. Which statements accurately describe these mechanisms of action? Select all that apply. A. Distribution occurs after a drug has been absorbed into the bloodstream and is made available to body fluids and tissues. B. Metabolism is the process by which a drug is transferred from its site of entry into the body to the bloodstream. C. Absorption is the change of a drug from its original form to a new form, usually occurring in the liver. D. During first-pass effect, drugs move from the intestinal lumen to the liver by way of the portal vein instead of going into the system's circulation. E. The gastrointestinal tract, as well as sweat, salivary, and mammary glands, are routes of drug absorption. F. Excretion is the process of removing a drug, or its metabolites (products of metabolism), from the body.

A.D.F. Distribution occurs after a drug has been absorbed into the blood stream and the drug is distributed throughout the body, becoming available to body fluids and body tissues. some drugs move from the intestinal lumen to the liver by way of the portal vein and do not go directly into the systemic circulation following oral absorption. This is called the firs pass effect, or hepatic first pass. Excretion is the process of removing a drug or its metabolites (products of metabolism) from the body. Absorption is the process by which a drug is transferred from its site of entry into the body to the bloodstream. Metabolism, or biotransformation, is the change of a drug from its original form to a new form. THe liver is the primary site for drug metabolism. The gastrointestinal tract, as well as sweat, salivary, and mammary glands, are routes of drug excretion.

Which clinical manifestation indicates that a further nursing assessment is necessary to determine if the patient is having difficulty swallowing? (Select all that apply.) A. Debris in the buccal cavity B. Abdominal cramping C. Epigastric pain D. Slurred speech E. Constipation F. Drooling

A.D.F. Retention of food in the oral cavity indicates that the patient is not swallowing ingested food completely. Food collects in the buccal cavity because the area between the teeth and cheek forms a pocket that traps food. Slurred speech reflects an inability of the tongue and muscles of the face to form words. Dysfunction of the muscles of the face and tongue will interfere with the ability to chew and swallow food. Drooling indicates that oral secretions are accumulating in the mouth. This may occur when a person has difficulty swallowing.

Active Learning scenario: Chapter 46 List at least three factors that influence the rate of biotransformation

Age: Infants have a limited medication-metabolizine capacity. The aging process also can influence medication metabolism, but varies with the individual. In general, hepatic medication metabolism tends to decline with age. Older adults require smaller doses of medications due to the possibility of accumulation in the body An increase in some medication-metabolizing enzymes: This can metabolize a particular medication sooner, requiring an increase in dosage of that medication to maintain a therapeutic level. It can also cause an increase in the metabolism of other concurrent use of medications First pass effect: The liver inactivates some medications on their first pass through the liver. Thus, they require a nonenteral route (sublingual, IV) because of their high first pass effect. Similar metabolic pathways: When the same pathway metabolizes two medications, it can alter the metabolism of one or both of them. In this way, the rate of metabolism can decrease for one or both of the medications, leading to medication accumulation. Nutritional Status: Clients who are malnourished can be deficient in the factors that are necessary to produce specific medication-metabolizing enzymes, thus impairing medication metabolism

Box 36-1 Additional Terms Used to Describe Urinary Problems

Anuria: 24 hour urine output that is less than 50mL: synonyms are complete kidney shutdown or renal failure Dysuria: Painful or difficult urination Frequency: Increased incidence of voiding Glycosuria: Presence of sugar in the urine Nocturia: Awakening at night to urinate Oliguria: Scanty or greatly diminished amount of urine voided in a given time; 24 hour urine output is less than 400mL Polyuria: Excessive output of urine (diuresis) Proteinuria: Protein in the urine; indication of kidney disease Pyuria: Pus in the urine; urine appears cloudy Suppression: Stoppage of urine production; normally the adult kidneys produce urine continuously at the rate of 60 to 120mL Urgency: Strong desire to void Urinary incontinence: Involuntary loss of urine

A nurse is calculating the body mass index (BMI) of a 35 year old male patient who is extremely obese. The patient's height is 5'6 and his current weight is 325 pounds. What would the nurse document as his BMI? A. 50.5 B. 52.4 C. 54,5 D. 55.2

B. BMI= weight in pounds (325) ----------------------------------------------x 703 height in inches (66) x height in inches (66) BMI = 52.4

A patient has an order for Chloromycetin, 500mg every 6 hours. The drug comes in 250mg capsules. What would the nurse administer? A. 1 tab B. 2 tabs C. 3 tabs D. 4 tabs

B. 2 tabs

A physician prescribes a PRN order for a post surgical patient. When would the nurse administer the medication? A. Every hour B. As needed C. One time only D. Immediately

B. As needed

A nurse is inserting a nasogastric tube ordered for a patient to monitor bleeding in his GI tract. When the tube is being passed through the pharynx, the patient begins to cough and show signs of respiratory distress. What would be the priority action of the nurse upon this assessment? A. Keep the tube in place and notify the primary care provider immediately. B. Stop advancing the tube and pull it back into the nasal area C. Ask the patient if he wants the nurse to stop the procedure. D. Call for help to perform CPR

B. As the tube is passing through the pharynx and the patient begins to cough and show respiratory distress, the nurse should stop advancing the tube and pull it back into the nasal area. The nurse should also support the patient as he regains normal breathing ability and composure and have him try again if he feels able to.

A nurse is planning to use healing intention with a client who is recovering from a lengthy illness. Which of the following is the priority action the nurse should take before attempting this particular mind body intervention? A. Tell the client the goal of the therapy is to promote healing B. Ask whether the client is comfortable with using prayer C. Encourage the client participate actively for best results D. Instruct the client to relax during the therapy

B. Ask whether the client is comfortable with using prayer. The first action the nurse should take using the nursing process is to assess or collect data from the client. Because people can have personal, cultural, or religious sensitivities or aversions to religious practices such as prayer, the nurse must first determine that the client is comfortable with a therapy that involves prayer. The nurse should tell the client the goals of therapy to provide information to the client. The nurse should encourage the client to participate to improve effectiveness of the therapy. The nurse should instruct the client to relax to promote the client's ability to focus during the therapy. However, there is another action the nurse should take first.

A nurse is administering an anti hypertensive drug to a hospitalized patient. What action should the nurse take to identify the patient prior to administration? A. Call the patient by name B. Check the patient's ID bracelet C. Check the patient's record D. Check the patient's name with family or significant others

B. Check the patient's ID bracelet

Which nursing action associated with successful tube feedings follows recommended guidelines? A. Check tube placement by adding food dye to the tube feed as a means of detecting aspirated fluid B. Check the residual before each feeding or every 4 to 8 hours during a continuous feeding C. Assess for bowel sounds at least four times per shift to ensure the presence of peristalsis and a functional intestinal tract D. Prevent contamination during enteral feedings by using an open system

B. Check the residual before each feeding or every 4 to 8 hours during a continuous feeding

A nurse is working with a newly licensed nurse who is administering medications to clients. Which of the following actions should the nurse identify as an indication that the newly hired nurse understands medication error prevention? A. Taking all medications out of the unit dose wrappers before entering the client's room B. Checking with the provider when a single dose requires administration of multiple tablets C. Administering a medication, then looking up the normal dose range D. Relying on another nurse to clarify a medication prescription

B. Checking with the provider when a single dose requires administration of multiple tablets- if a single dose requires multiple tablets, it is possible that an error has occurred in the prescription or transcription of the medication. This action could prevent a medication error. To prevent errors, the nurse should not take unit dose medications out of wrappers until at the bedside when performing the third check of medication administration. The nurse can encourage clients' involvement and provide teaching at this time. Reviewing the usual dosage range prior to administration can help the nurse identify an inaccurate dosage If the prescription is unclear, the nurse should contact the provider, not another nurse, for clarification

A nurse working in a long term care facility incorporates aromatherapy into her practice. For which patient would this nurse use the herb ginger? A. A patient who has insomnia B. A patient who has nausea C. A patient who has dementia D. A patient who has migraine headaches

B. Commonly used essential oils in a health care setting are ginger or peppermint for nausea and lavender or chamomile for insomnia

A nurse is administering a hepatitis B shot to an adult patient. Which site would the nurse choose for this injection? A. Vastus lateralis site B. Deltoid muscle site C. Ventrogluteal site D. Dorsogluteal site

B. Deltoid muscle site

A nurse is caring for an obese patient who has had surgery. The nurse monitors this patient for what postoperative complication? A. Hunger B. Impaired wound healing C. Hemorrhage D. Gas pains

B. Fatty tissue is less vascular and, therefore, less resistant to infection and more prone to delayed wound healing

The nurse is scheduling tests for a patient who is experiencing bowel alterations. What is the most logical sequence of tests to ensure an accurate diagnosis? A. Barium studies, endoscopic examinations, fecal occult blood test B. Fecal occult blood test, barium studies, endoscopic examination C. Barium studies, fecal occult blood test, endoscopic examination D.Endoscopic examination, barium studies, fecal occult blood test

B. Fecal occult blood test, barium studies, endoscopic examination

A 56 year old patient who has COPD is refusing to eat. Which intervention would be most helpful in stimulating his appetite? A. Administering pain medication after meals B. Encouraging food from home when possible C. Scheduling his respiratory therapy before each meal D. Reinforcing the importance of his eating exactly what is delivered to him.

B. Food from home that the patient enjoys may stimulate him to eat. Pain medication should be given before meals, respiratory therapy should be scheduled after meals, and telling the patient what he must eat is no guarantee that he will comply.

A nurse is talking with a client who reports constipation. When the nurse discusses dietary changes that can help prevent constipation, which of the following foods should the nurse recommend? A. Macaroni and cheese B. Fresh fruit and whole wheat toast C. Bread pudding and yogurt D. Roast chicken and white rice

B. Fresh fruit and whole wheat toast. A high fiber diet promotes normal bowel elimination. The nurse should recommend the client consume fresh fruits and vegetables with whole grain carbohydrates to provide the highest fiber option. The nurse should identify macaroni and cheese as a low residue option that could actually worsen constipation. The nurse should identify bread pudding and yogurt as low residue options that could actually worsen constipation. The nurse should identify roast chicken and white rice as low residue options that could actually worsen constipation

A nurse is teaching an adult client how to administer ear drops. Which of the following statements should the nurse identify as an indication that the client understands the proper techniques? A. I will straighten my ear canal by pulling my ear down and back B. I will gently apply pressure with my finger to the front part of my ear after putting in the drops C. I will insert the nozzle of the ear drop bottle snug into my ear before squeezing the drops in D. After the drops are in, I will place a cotton ball all the way into my ear canal

B. I will gently apply pressure with my finger to the front part of my ear after putting in the drops- The client should gently apply pressure with the finger to the tragus of the ear after administering the drops to help the drops go into the ear canal. The client should straighten his ear canal by pulling the auricle upward and outward to open up the ear canal and allow the medication to reach the eardrum. The client should never occlude the ear canal with the dropper when instilling ear drops because this can cause pressure that could injure the eardrum The client should not place a cotton ball past the outermost part of the ear canal because it could introduce bacteria to the inner or middle ear.

Which nursing action is performed according to guidelines for aspirating fluid from smallbore feeding tube? A. Use a small syringe and insert 10mL of air B. If fluid is obtained when aspirating, measure its volume and pH and flush the tube with water. C. Continue to instill air until fluid is aspirated D. Place the patient in the Trendelenburg position to facilitate the fluid aspiration process.

B. If fluid is obtained when aspirating, measure its volume and pH and flush the tube with water

A nurse is irrigating the colostomy of a patient and is unable to get the irrigation solution to flow. What would be the nurse's next action in this situation? A. Assist the patient to a prone position on a waterproof pad and try again. B. Check the clamp on the tubing to make sure that the tubing is open C. Quickly pull the cone from the stoma and check for bleeding D. Remove the equipment and call the primary care provider

B. If irrigation solution is not flowing, the nurse should first check the clamp on the tubing to make sure the tubing is open. Next, the nurse should gently manipulate the cone in the stoma and check for a blockage of stool. If there is a blockage, the nurse should remove the cone from the stoma, clean the area, and gently reinsert. Alternately, the nurse could assist the patient to a side lying or sitting position in bed, place a waterproof pad under the irrigation sleeve, and place the drainage end of the sleeve in a bedpan

After surgery, a patient is having difficulty voiding. Which nursing action would most likely lead to an increased difficulty with voiding? A. Pouring warm water over the patient's fingers B. Having the patient ignore the urge to void until her bladder is full C. Using a warm bedpan when the patient feels the urge to void D. Stroking the patient's leg or thigh

B. Ignoring the urge to void makes urination even more difficulty and should be avoided. The other activities are all recommended to promote voiding

A nurse is reviewing a client's medications. They include cimetidine and imipramine. Knowing that cimetidine decreases the metabolism of imipramine, the nurse should identify that this combination is likely to result in which of the following effects? A. Decreased therapeutic effects of cimetidine B. Increased risk of imipramine toxicity C. Decreased risk of adverse effects of cimetidine D. Increased therapeutic effects of imipramine

B. Increased risk of imipramine toxicity. A medication that decreases the metabolism of another medication increases the serum level of that medication, increasing the risk for toxicity. A medication that increases the metabolism of another medication can decrease the effectiveness of that medication. A medication that decreases the metabolism of another medication does not decrease the risk for adverse effects. A medication that decreases the metabolism of another medication does not increase that medication's therapeutic effects.

A young adult client in a provider's office tells the nurse that she uses fasting for several days each week to help control her weight. The client takes several medications for various chronic issues. The nurse should explain to the client that which of the following mechanisms that results from fasting puts her at risk for medication toxicity? A. Increasing the metabolism of medications over time B. Increasing the protein binding response C. Increasing the medications' transit time through the intestines D. Decreasing the excretions of medications

B. Increasing the protein binding response. Inadequate nutrition, such as starvation, can affect the protein binding response of medications. It increases their response and thus increases the risk for medication toxicity. Some medications, not fasting, cause metabolic tolerance as metabolism of the medication increases over time and the effectiveness of the medication declines. Disorders that cause diarrhea, not fasting, cause oral medications to pass through the gastrointestinal tract too quickly for adequate absorption. This mechanism does not cause toxicity. Kidney disease or failure, not fasting, prevents or delays medication excretion, which can cause toxicity

A patient tells the nurse she is having pain in her right lower leg. How does the nurse assess for the presence of thrombophlebitis? A. By palpating the skin over the tibia and fibula B. By documenting daily calf circumference measurements C. By recording vital signs obtained four times a day D. By noting difficulty with ambulation

B. Inflammation from thrombophlebitis increases the size of the affected extremity and can be assessed by measuring circumference on a regular basis

A nurse is caring for a client who is at high risk for aspiration. Which of the following actions should the nurse take? A. Give the client thin liquids B. Instruct the client to tuck her chin when swallowing C. Have the client use a straw D. Encourage the client to lie down and rest after meals

B. Instruct the client to tuck her chin when swallowing. Tucking the chin when swallowing allows food to pass down the esophagus more easily. Thin liquids increase the client's risk for aspiration. Using a straw increases the client's risk for aspiration. Sitting for an hour after meals helps prevent gastroesophageal reflux and possible aspiration

At a follow up visit, a patient recovering from a myocardial infarction tells the nurse: "I feel like my life is out of control ever since I had the heart attack. I would like to sign up for yoga, but I don't think I'm strong enough to hold poses for long." What would be the nurse's best response? A. Right now you should concentrate on relaxing and taking your blood pressure medicine regularly, instead of worrying about doing yoga. B. There is a slower paced yoga called Kripaula that focuses on coming into balance and relaxation that you could look into C. Ashtanga yoga is a gentlepaced yoga that would help with your breathing and blood pressure D. Yoga is contraindicated for patients who have had a heart attack

B. Krishpaula or gentle yoga focuses on relaxation and coming into balance. Astanga focuses on synchronizing breath with a fast paced series of postures. The nurse should not discourage the use of yoga in patients who are healthy enough to participate. Yoga is not contraindicated in patient with controlled high blood pressure.

A nurse administers anticholinergics to a patient as a postoperative medication. What condition does this medication help to prevent? A. Cardiovascular complications B. Laryngospasm C. Nausea D. Shock

B. Laryngospasm

A patient requires 40units of NPH insulin and 10 units of regular insulin daily subcutaneously. What is the correct sequence when mixing insulins? A. Inject air into the regular insulin vial and withdraw 10units; then, using the same syringe, inject air into the NPH vial and withdraw 40units of NPH insulin B. Inject air into the NPH insulin vial, being careful not to allow the solution to touch the needle; next, inject air into the regular insulin vial and withdraw 10units; then, withdraw 40units of NPH insulin C. Inject air into the regular insulin vial, being careful not to allow the solution to touch the needle; next, inject air into the NPH insulin vial and withdraw 40units ; then withdraw 10units of regular insulin. D. Inject air into the NPH insulin vial and withdraw 40units; then using the same syringe, inject air into the regular insulin vial and withdraw 10units of regular insulin

B. Regular or short acting insulin should never be contaminated with NPH or any other insulin modified with added protein. Placing air in the NPH vial first without allowing the needle to contact the solution ensures that the regular insulin will not be contaminated

A nurse is preparing to inject heparin subcutaneously for a client who is postoperative Which of the following actions should the nurse take? A. Use a 22 gauge needle B. Select a site on the clients abdomen C. Spread the skin with the thumb and index finger D. Observe for the bleb formation to confirm proper placement

B. Select a site on the client's abdomen.- For a subcutaneous injection, the nurse should select a site that has an adequate fat-pad size (abdomen, upper hips, lateral upper arms, thighs) For a subcutaneous injection, the nurse should use a 25-27gauge needle For a subcutaneous injection, the nurse should pinch the skin with her thumb and index finger Bleb formation confirms injection into the dermis, not subcutaneous tissue

When preparing a patient who has diabetes mellitus for surgery, the nurse should be aware of what surgical risk associated with this disease? A. Fluid and electrolyte imbalance B. Slow wound healing C. Respiratory depression from anesthesia D. Altered metabolism and excretion of drugs

B. Slow wound healing

Digoxin (Lanoxin), 0.0625mg PO, is ordered. Digoxin is available as 0.125,g tablets. How many tablets would the nurse administer?

Dose on hand = Dose required ---------------- ----------------- Quantity on hand X(quantity desired) 0.125mg = 0.0625mg --------- ------------ 1 tablet X 0.125x= 0.0625 x= 0.5 or 1/2 tablet

The nurse is inserting a rectal tube to administer a large volume enema. Which of nursing action is performed correctly in this procedure? A. Position the patient on his or her back and drape properly B. Slowly and gently insert the enema tube 3-4 inches (7-10cm) for an adult C. Introduce the solution quickly over a period of 3-5 minutes D. Encourage the patient to hold the solution for at least 20 minutes

B. Slowly and gently insert the enema tube 3-4inches (7-10cm) for an adult

Ms. Hall has an order for hydromorphone (Dilaudid), 2 mg, intravenously, q 4 hours PRN pain. The nurse notes that according to Ms. Hall's chart, she is allergic to Dilaudid. The order for medication was signed by Dr. Long. What would be the correct procedure for the nurse to follow in this situation? A. Administer the medication; the doctor is responsible for medication administration. B. Call Dr. Long and ask that she change the medication. C. Ask the supervisor to administer the medication. D. Ask the pharmacist to provide a medication to take the place of Dilaudid.

B. The nurse is responsible for any medications he or she gives and must contact the doctor to inform her of the patient's allergy to the drug. The nurse should not give the medication and might speak with supervisor only if uncomfortable with the physician's answer once she is notified. The nurse is legally unable to order replacement medication, as is the pharmacist

A nurse caring for a patient's hemodialysis access documents the following: "5/10/15 0930 Arteriovenous fistula patient in right upper arm. Area is warm to touch and edematous. patient denies pain and tenderness. Positive bruit and thrill noted." Which documented finding would the nurse report to the primary care provider? A.Positive bruit noted B. Area is warm to touch and edematous C. Patient denies pain and tenderness D. Positive thrill noted

B. The nurse would report a site that is warm and edematous as this could be a sign of a site infection. The thrill and bruit are normal findings caused by arterial blood flowing into the vein. If these are not present, the access may be cutting off. No report of a pain is a normal finding

A 70 year old male is scheduled for surgery. He says to the nurse "I am so frightened- what if I don't wake up?" What would be the nurse's best response? A. You have a wonderful doctor B. Let's talk about how you are feeling C. Everyone wakes up from surgery D. Don't worry, you will be just fine

B. This answer allows the patient to talk about his feelings and fears, and is therapeutic

The nurse measures a patient's residual urine by catheterization after the patient voids. What condition would this test verify? A. Urinary tract infection B. Urinary retention C. Urinary incontinence D. Urinary suppression

B. Urinary retention

A physician orders a pain medication for a postoperative patient that is a PRN order. When would the nurse administer this medication? A. A single dose during the postoperative period B. Doses administered as needed for pain relief C. One dose administered immediately D. Doses routinely administered as a standing order

B. When the prescriber writes a PRN order (as needed) for medication, the patient receives medication when it is requested or required. With a singles or one time order, the directive is carried out only once, at a time specified by the prescriber. A stat order is a single order carried out immediately. A standing order (routine order) is carried out as specified until it is canceled by another order

While injecting a needle into a patient for an intramuscular injection, the nurse hits the patient's bone. What would be the appropriate initial response of the nurse to this situation? A. Remove the needle and have another nurse stay with the patient while informing the primary care provider B. Withdraw the needle, apply a new needle to syringe and administer the injection in an alternate site C. Document the incident according to facility policy and then remove the needle and syringe and discard it. D. Remove the needle and discard the needle and syringe; call the primary care provider

B. Withdraw the needle, apply a new needle to syringe and administer the injection in an alternate site

The nurse is catheterizing a male urinary bladder, and urine leaks out of the meatus around the catheter. What actions would the nurse perform next? (Select all that apply) A. Increase the size of the indwelling catheter B. Make sure the smallest sized catheter with a 10mL balloon is used C. Consider an evaluation for urinary tract infection D. Ensure that the correct amount of solution was used to inflate the balloon E. If under fill is suspected, attempt to push the catheter further into the bladder F. Assess the patient for diarrhea

B.C.D Make sure the smallest sized catheter with a 10mL balloon is used. Consider an evaluation for urinary tract infection. Ensure that the correct amount of solution was used to inflate the balloon

A nurse is collecting data from a client who takes haloperidol to treat schizophrenia. Which of the following findings should the nurse document as extrapyramidal symptoms (EPSs)? (Select all that apply) A. Orthostatic hypotension B. Tremors C. Acute Dystonia D. Decreased level of consciousness E. Restlessness

B.C.E. Tremors are an EPS. Others are rigidity, drooling, agitation, and a shuffling gait. Acute dystonia is an EPS. It includes spastic movements of the back, neck, tongue, and face. Restlessness in an EPS. Orthostatic hypotension is an adverse effect, but is not an EPS. Decreased level of consciousness is an adverse effect, but not an EPS.

A nurse is caring for a patient who is receiving 2 types of analgesics. Which effects does the nurse understand occur when these medications are administered together? (Select all that apply.) A. Curative B. Palliative C. Synergistic D. Potentiation E. Antagonistic

B.C.D. Analgesics decrease the intensity of pain (palliative treatment). Palliative treatments minimize signs and symptoms or promote comfort; palliative treatments do not produce a cure. When 2 analgesics are given together they exert a synergistic effect. A synergistic effect occurs when the combined effect of two medications is greater than when the effects of each are added together. When 2 analgesics are given together they potentiate the action of each other. Potentiation occurs when drugs administered together increase the action of one or both drugs.

The nurse is attempting to insert an NG tube and, as the tube is passing through the pharynx, the patient begins to retch and gag. What nursing interventions are appropriate in this situation? (Select all that apply) A. Inspect the other nostril and attempt to pass the nasogastric tube down that nostril B. Ask the patient if he or she needs to pause before continuing insertion C. Continue to advance tube when the patient relates that he or she is ready D. Have the emesis basin nearby in case patient begins to vomit. E. Give small air boluses until gastric contents can be aspirated F. Insert a nasointestinal tube

B.C.D. Ask the patient if he or she needs to pause before continuing insertion. Continue to advance the tube when the patient relates that he or she is ready. Have the emesis basin nearby in case patient begins to vomit

A nurse is preparing medications for a preschooler. Which of the following factors should the nurse identify as altering how a medication affects children? (select all that apply) A. Increased gastric acid production B. Lower blood pressure C. Higher body water content D. Increased absorption of topical medications E. Increased gastric emptying time

B.C.D. Children have lower blood pressure. Children have a higher body water content. Children have increased absorption of topical medications Children have decreased gastric acid production. Children have a slower gastric emptying time.

A nurse is caring for a client who has had diarrhea for 4 days. When assessing the client, the nurse should expect which of the following findings? (Select all that apply) A. Bradychardia B. Hypotension C. Elevated temperature D. Poor skin turgor E. Peripheral edema

B.C.D. Hypotension-Prolonged diarrhea leads to dehydration. The nurse should expect the client to have a decrease in blood pressure, elevated temperature, and poor skin turgor. The nurse should expect the client who has prolonged diarrhea to have tachycardia. The nurse should expect the client who has prolonged diarrhea to possibly have weakened peripheral pulses. Peripheral edema results from a fluid overload

To promote adherence with medication self‑administration, a nurse is making recommendations for an older adult client. Which of the following instructions should the nurse include? (Select all that apply.) A. Adjust dosages according to daily weight. B. Place pills in daily pill holders. C. Ask for liquid forms if the client has difficulty swallowing pills. D. Ask a relative to assist periodically. E. Request child‑resistant caps on medication containers.

B.C.D. Organizing medications in daily pill holders promotes medication adherence. Providing a form of medication that is easier for the client to swallow promotes medication adherence. Including the client's support system promotes medication adherence The provider adjusts the client's dosages. Instructing the client to base dosages on daily weight increases the risk for error in medication self administration. Some older adult clients have difficulty opening child resistant caps. Request easy open containers from the pharmacy

A patient has a diagnosis of osteoporosis. Which nutrients should the nurse encourage this patient to eat? (Select all that apply.) A. Rice B. Milk C. Yogurt D. Sardines E. Almonds F. Tomatoes

B.C.D.E. Milk and milk products, yogurt, sardines (which contain soft edible bones), and almonds are all excellent sources of calcium.

Which patients would the nurse schedule for surgery based on purpose? (Select all that apply) A. A patient who has uncontrolled bleeding B. A patient undergoing a breast biopsy C. A patient needing a cleft palate repair. D. A patient needing a bowel resection E. A patient in respiratory distress who needs a tracheostomy F. A patient post mastectomy who decides to have breast reconstruction

B.C.D.F. A patient undergoing a breast biopsy. A patient needing a cleft palate repair. A patient needing a bowel resection. A patient post mastectomy who decides to have breast construction

A nurse is preparing the discharge summary for a client who has had knee arthroplasty and is going home. Which of the following information about the client should the nurse include in the discharge summary? (Select all that apply) A. Advance directive status B. Follow up care C. Instructions for diet and medications D. Most recent vital sign data E. Contact information for the home health care agency

B.C.E. It is essential to include the names and contact information of providers and community resources the client will need after he returns home. The client will need written information detailing his medication and dietary therapy at home. A client who has had knee arthroscopy typically requires analgesics, possibly anticoagulants, and dietary instructions for avoiding postoperative complications such as constipation. It is essential to include the names and contact information of providers and community resources the client will need after returning home. For example, a client had a knee arthroplasty might require physical therapy at home until he can travel to a physical therapy department or facility. Advance directive status is important in transfer documentation, when other care providers will take over a client's care. They are not an essential component of a discharge summary for a client who is returning to his home. Vital sign measurements are important in transfer documentation, when other care providers will take over a client's care. They are not an essential component of a discharge summary for a client who is returning home

Which factors should the nurse consider when assessing patients for postsurgical risks? (Select all that apply) A. Cardiovascular diseases increase the risk for dehydration after surgery. B. Patients with respiratory disease may experience alterations in acid base balance after surgery C. Kidney and liver diseases influence the patient's response to anesthesia D. Endocrine diseases increase the risk for hyperglycemia after surgery E. Endocrine diseases increase the risk for slow surgical wound healing F. Pulmonary disorders increase the risk or hemorrhage and hypovolemic shock after surgery

B.C.E. Patients with respiratory disease may experience alterations in acid base after surgery. Kidney and liver diseases influence the patient's response to anesthesia. Endocrine disease increase the risk for slow surgical wound healing

The nurse is preparing a patient for an intravenous pyelogram. Which nursing actions are performed correctly? (Select all that apply) A. Tell the patient not to void before the test B. Withhold or limit foods before testing C. Give an enema the day of the examination D. Restrict fluids and foods immediately after the examination E. Obtain patient's allergy history F. Give a laxative the evening before the examination

B.C.E.F. Withhold or limit foods before testing. Give an enema the day of the examination. Obtain patient's allergy history. Give a laxative the evening before the examination

A nurse is assessing the bowel elimination of patients in a med surg unit. What developmental factors affecting elimination should the nurse consider? (Select all that apply) A. Voluntary control of defecation occurs between the ages of 12 and 18 months B. The number of stools that infants pass varies greatly C. Some children have bowel movements only every 2 or 3 days D. A child who has not had a bowel movement daily is most likely constipated E. In an infant, a liquid stool signifies diarrhea F. Constipation is often a chronic problem for older adults

B.C.F. The number of stools that infants pass varies greatly. Some children have bowel movements only every 2 or 3 days. Constipation is often a chronic problem for older adults

Which interventions are uniquely related to the administration of an intradermal injection? (Select all that apply.) A. Using the air-bubble technique B. Circling the injection site with a pen C. Pinching the skin during needle insertion D. Inserting the needle with the bevel upward E. Massaging the area after fluid is instilled

B.D. Circling the injection site with a pen indicates the area that must be evaluated; generally the site is assessed 72 hours after the ID injection. When medication is injected with the bevel up, a small wheal will form under the skin. This technique is used only with ID injections.

A nurse teaches a postoperative patient about foods high in protein that will promote wound healing. Which food selection by the patient indicates that the teaching was effective? (Select all that apply.) A. Milk B. Meat C. Bread D. Cheese E. Vegetables

B.D. Food from animal sources provide complete proteins and therefore is the best source of protein. Cheese is a product that is produced from an animal source. It provides a complete protein and promotes wound healing.

A nurse is assessing a patient to determine if it is appropriate to administer a prescribed medication via the oral route. Which information indicates that the nurse should ask the primary healthcare provider for a change in route? (Select all that apply.) A. Nausea B. Unconsciousness C. Gastric suctioning D. Emergency situation E. Difficulty swallowing

B.D. Nothing that requires swallowing should ever be placed into the mouth of an unconscious patient because of the risk of aspiration. In an emergency a drug is best administered via IV, rather than orally, because it is faster acting.

A primary healthcare provider prescribes a liquid oral medication for a patient. Which actions should the nurse implement when administering this medication? (Select all that apply.) A. Vigorously shake the liquid before pouring a dose. B. Measure oral liquids in a calibrated medication cup at eye level. C. Pour liquid with the label facing away from the palm of the hand. D. Place an opened top of a container on a surface with the inside lid facing up E. Use a needleless syringe to measure an oral liquid less than 5 mL and transfer it to a medication cup.

B.D.E. Measuring oral liquid in a calibrated medication cup at eye level ensures accuracy. Placing an opened top of a container on a surface with the inside lid facing up prevents contamination of the inside of the lid and subsequent contamination of the bottle when the lid is returned and closed. Using a needless syringe to measure an oral liquid volume less than 5 mL and transferring it to a medication cup are acceptable practices because they ensure accuracy.

A nurse who is administering a piggyback intermittent intravenous infusion of medication to a patient observes that there is a cloudy, white substance forming in the IV tubing. What actions should the nurse take in this situation? (Select all that apply) A. Assess the IV site for signs of infiltration or phlebitis B. Stop the IV from flowing and stop administering the medication C. Prime the secondary tubing by "backfilling" it D. Clamp the IV at the site nearest to the patient E. Replace the tubing on primary and secondary infusions F. Check literature regarding incompatibilities of medications after administering

B.D.E. Stop the IV from flowing and stop administering the medication. Clamp the IV at the site nearest to the patient Replace the tubing on primary and secondary infusions

The nurse inspects the stool of patients admitted to the hospital with abdominal distress. Which statements accurately describe the normal characteristics of stool and special considerations for observation? (Select all that apply) A. Consistently large diarrheal stools suggest a disorder of the left colon or rectum B. The rapid rate of peristalsis in the breastfed infant causes the stool to be yellow C. The absence of bile may cause the stool to appear black D. Antacids in the diet cause the stool to be whitish E. A gastrointestinal obstruction may result in a narrow, pencil shaped stool F. The odor of the stool is influenced by its pH value, which is normally slightly acidic

B.D.E. The rapid rate of peristalsis in the breastfed infant causes the stool to be yellow. Antacids in the diet cause the stool to be whitish. A gastrointestinal obstruction may result in a narrow, pencil shaped stool

A nurse is monitoring a patient's IV infusion. Which data are necessary to determine that the IV is "on time"? (Select all that apply.) A. Drip rate per minute B. Time the bag was hung C. Solution indicated on the IV bag D. Volume of solution in the IV bag E. Milliliters per hour ordered by the primary healthcare provider

B.D.E. The time that the IV bag was hung is essential to know to determine if the IV is "on time". Identify how many min/hr the IV has been running and multiply this number by the mL of solution ordered per min/hr. Deduct this volume from the original volume in the bag. Then compare the actual volume in the bag with what should be in the bag to determine if they match. If they match, it's "on time." And if it's too much in the bag, it's "behind schedule". If there's not less in the bag, it's "ahead of schedule." The nurse must also know the volume of the solution in the IV bag, as well as the number of mL/hr, and do the same calculation.

Which human responses to illness alert the nurse that a patient is at risk for aspiration during meals? (Select all that apply.) A. Bulimia B. Lethargy C. Anorexia D. Stomatitis E. Dysphagia

B.D.E. When a person is sleepy, sluggish, or stuporous (lethargic), there may be a reduced level of consciousness and diminished reflexes, including the gag and swallowing reflexes. This condition can result in aspiration of food or fluids that can compromise the person's airway and respiratory status. An inflammation of the mucous membranes of the mouth (stomatitis) may result in dysphagia (difficulty swallowing) and increase the risk of aspiration. Dysphagia places a patient at risk for aspiration generally because of impaired innervations of the tongue and muscles used for swallowing

A nurse works for a physician who practices the naturopathic system of medicine. What is the focus of nursing actions based on this type of medical practice? (Select all that apply) A. Treating the symptoms of the disease B. Providing patient education C. Focusing on treating individual body systems D. Making appropriate interventions to prevent illness E. Believing in the healing power of nature F. Encouraging patients to take responsibility for their own health

B.D.E.F. Naturopathic medicine is not only a system of medicine, but also a way of life, with emphasis on client responsibility, client education, health maintenance, and disease prevention. its principles include minimizing harmful side effects and avoiding suppression of symptoms, educating patients and encouraging them to take responsibility for their own health, treating the whole person, preventing illness, believing in the healing power of nature, and treating the cause of a disease or condition rather than its symptoms

A nurse is explaining the method of allopathic medicine to a patient. Which statements describe this approach to medicine? (Select all that apply) A. "Curing is accomplished by internal agents." B. "Illness occurs in either the mind or the body, which are separate entities." C. "Illness is a manifestation on imbalance or disharmony and is a process." D. "Curing occurs quickly and seeks to destroy the invading organism or repair the affected part." E. "Healing is done by the patient." F. "Health is the absence of disease."

B.D.F. Illness occurs in either the mind or the body, which are separate entities. Curing occurs quickly and seeks to destroy the invading organism or repair the affected part. Health is the absence of disease

The nurse is performing digital removal of a fecal impaction. Which nursing actions follow guidelines for this procedure? (Select all that apply) A. Have the patient lie on his stomach and pie fold top linens over him B. Place the patient in a side lying position C. Vigorously work the finger around and into the hardened mass to break it up D. Use nonsterile gloves for the procedure because the intestinal tract is not sterile E. Use a cleansing enema if necessary F. Lubricate the index finger generously to reduce irritating the rectum, and insert the finger gently into the anal canal

B.D.F. Place the patient in a side lying position. Use nonsterile gloves for the procedure because the intestinal tract is not sterile. Lubricate the index finger generously to reduce irritating the rectum, and insert the finger gently into the anal canal

Which actions would a nurse perform when instilling eardrops correctly? (Select all that apply) A. Make sure the solution to be instilled is at room temperature B. Clean the external ear with cotton balls moistened with normal saline solution C. Place the patient on the affected side in bed D. Draw up the amount of solution needed in the dropper and return any excess medication to the stock bottle E. Straighten the auditory canal by pulling the cartilaginous portion of the pinna up and back in an adult and down and back in an infant or child under 3 years F. Hold the dropper in the ear with its tip above the auditory canal

B.E.F. Clean the external ear with cotton balls moistened with normal saline solution. Straighten the auditory canal by pulling the cartilaginous portion of the pinna up and back in an adult and down and back in an infant or child under 3 years Hold the dropper in the ear with its tip above the auditory canal

A nurse assesses the stool of patients who are experiencing gastrointestinal problems. In which patient's would diarrhea be a possible finding? (Select all that apply) A. A patient who is taking narcotics for pain B. A patient who is taking laxatives C. A patient who is taking diuretics D. A patient who is dehydrated E. A patient who is taking amoxicillin for an infection F. A patient who is taking over the counter antacids

B.E.F. Diarrhea is a potential adverse effect of treatment with Amoxicillin Clavulanate (Augmentin), laxatives, or over the counter antacids. Narcotics, diuretics, and dehydration may lead to consitpation

The nurse is teaching a patient how to use an insulin pen. Which steps reflect recommended procedure? (Select all that apply) A. After administering injection, keep button depressed; count to 3 before removal B. Hold the pen upright and tap to force any air bubbles to the top C. Check that dose selector is at 2 before dialing units of insulin for the dose D. After administering the injection, push the button on the pen halfway in E. Dial the dose selector to 2 units to perform an "air shot" to get rid of bubbles F. Clean injection site and administer injection by holding pen like a dart

B.E.F. Hold the pen upright and tap to force any air bubbles to the top. Dial the dose selector to 2units to perform an "air shot" to get rid of bubbles Clean the injection site and administer injection by holding pen like a dart

A nurse is removing an NG tube and notes epistaxis. What nursing interventions would the nurse perform in this situation? (Select all that apply) A. Notify primary care provider and anticipate order to reinsert NG tube B. Occlude both nares until bleeding has subsided C. Offer facial tissue to blow nose D. Record the amount of blood in the suction container E. Ensure that patient is in upright position F. Document epistaxis in patient's medical record

B.E.F. Occlude both nares until bleeding has subsided. Ensure that patient is in upright position. Document epistaxis in patient's medical record

Which methods would the nurse use when administering regional anesthesia to surgical patients? A. Inhalation B. Spinal block C. Intravenous D. Oral route E. Nerve block F. Epidural block

B.E.F. Spinal block. Nerve block. Epidural block

Which method of feeding would a nurse normally provide if a patient can attempt eating regular meals during the day and is prepared to ambulate and resume activities? A. Continuous feeding B. Intermittent feeding C. Cyclic feeding D. Ambulatory feeding

C. Cyclic feeding

A nurse is assessing the bowel elimination patterns of hospitalized patients. Which nursing actions related to the assessment process are performed correctly? (Select all that apply) A. The nurse auscultates the abdomen before inspection and palpation are performed B. The nurse places the patient in the supine position with the abdomen exposed C. The nurse drapes the patient's chest and pubic area and extends the patient's legs flat against the bed. D. The nurse encourages the patient to drink fluids before the assessment so that the bladder is full and can be examined E. The nurse uses a warmed stethoscope to listen for bowel sounds in all abdominal quadrants F. The nurse notes the character of bowel sounds, which are normally high pitched, gurgling, and soft

B.E.F. The nurse places the patient in the supine position with the abdomen exposed. The nurse uses a warmed stethoscope to listen for bowel sounds in all abdominal quadrants. The nurse notes the character of bowel sounds, which are normally, high pitched, gurgling, and soft

A nurse is assessing the freshly voided urine of a patient. What characteristics of the urine would indicate a urinary problem? (Select all that apply) A. The urine is amber colored B. The urine smells like ammonia C.The urine pH is 6.0 D. The urine is transluscent E. There is pus in the urine F. The urine is cloudy

B.E.F. The urine smells like ammonia. There is pus in the urine. The urine is cloudy

A nurse has been asked to witness a patient signature on an informed consent form for surgery. For which of these patients would the document be valid? (Select all that apply) A. A 92 year old patient who is severely confused B. A 45 year old patient who is oriented and alert. C. A 10 year old who is oriented and alert D. A 36 year old patient who has had a narcotic premedication E. A 45 year old mentally ill patient who has been ruled incompetent F. A 22 year old patient having an abortion against her partner's wishes

B.F. A consent form is not legal if the patient signing the form is confused, sedated, unconscious, or a minor

Box 36-3 Treatment Options For Urinary Incontinence

Behavioral Techniques: -Pelvic Floor exercises: Kegel exercises to strengthen pelvic floor muscles and sphincter muscles. Kegel exercises can be done alone, with weighted cones, or with biofeedback -Biofeedback: Measuring devices are used to help patient become aware of when pelvic floor muscles are contracting -Electrical stimulation: Electrodes are placed in the vagina or rectum that then stimulate nearby muscles to contract -Timed voiding or bladder training: May be used with biofeedback. Patient keeps track of when voiding and leaking occur to enable oneself to plan when to void, with increasing length of voiding intervals. Bladder training involves biofeedback and muscle training. Urgency control is addressed using distraction and relaxation techniques. Pharmacologic Treatment: -Treatment is dependent on type of incontinence. Some medications inhibit contractions of the bladder, others may relax muscles, and some tighten muscles at the bladder neck and urethra -Topical estrogen may be used in postmenopausal women to relieve atrophy of involved muscles -Collagen may be injected into the tissue around the urethra to add bulk and help close the urethral opening Mechanical Treatment: - Pessaries: A stiff ring that is inserted into the vagina, where it helps to reposition the urethra. The pessary may be placed by the patient or by a nurse -External barriers: Adhere to the urethral opening to stop urine leakage. The barrier is a small foam pad placed over the urethral opening. It seals against the body to keep urine from leaking. It is removed and discarded before the patient voids -Urethral insert: Small device, like a plug, that fits into the urethra. Removed to void, the insert is replaced until the patient needs to void again. -Surgical Intervention: Used as a last resort. Type of surgery depends on cause of incontinence.

Box 35-5 Nutritional Assessment Considerations for Older Adults

Biochemical Data: -Low serum albumin level (<3.5mg/dL) may be a reflection of the aging process rather than a nutritional risk factor. Albumin synthesis declines with age -Hemoglobin levels that are lower than normal may only reflect anemia observed in older people as part of the aging process Anthropometric Data: -Because of age related changes in body composition, skin fold measurements, if used, should be taken from several body sites Dietary Data: -Dietary recall may be inaccurate because of vision and memory problems -Question use of vitamin and mineral supplements -Gather information concerning medication regimen (prescribed and over the counter) to assess for food drug interactions and adverse effects of medications

Box 29-6 Nursing Assessments and Interventions to meet Postoperative Elimination needs

Bowel Elimination: -Assess for the return of peristalsis by auscultating bowel sounds every 4 hours when the patient is awake -Assess abdominal distention, especially if bowel sounds are not audible or are high pitched (indicative of possible paralytic ileus, which is absence of intestinal peristalsis) -Assist with movement in bed and ambulation to relieve gas pains, a common postoperative discomfort -Encourage food and fluid intake when ordered, especially fruit juices and high fiber foods -Maintain privacy when patient is using the bedpan, urinal, commode, or bathroom -Administer suppositories, enemas, or medications, such as stool softeners, as prescribed Urinary Elimination: -Monitor patterns of intake and output -Assist in assuming normal position to void by using an upright position when on a bedpan and using a bedside commode or bathroom when able, or by assisting the male patient to stand upright to void with a urinal -Assess for bladder distention by palpating above the symphysis pubis if the patient has not voided within 8 hours after surgery or if the patient has been voiding frequently in amounts of less then 50mL; notify the physician of abnormal assessment results -Maintain prescribed intravenous fluid infusion rates -Encourage oral fluid intake when prescribed -Provide privacy when the patient is using bedpan, bedside commode, urinal, or bathroom -Initiate urinary catheterization, if prescribed

Table 37-2 Classification of Laxatives

Bulk forming (ex Metamucil): Psyllium, grain, or synthetic product that causes stool to absorb water and swell, thus stimulating peristalsis (Action); Usually acts within 24 hours (Advantages); May interfere with absorption of calcium and iron and certain drugs. Should not be given to bedridden patients or those with intestinal strictures. May be expensive (Caution) Stool softener (ex Colace); Agents with detergent activity that allow water and fat to penetrate and lubricate the stool. (Action); Recommended for those who must avoid straining (Advantages); Has lubricant component of drug that may interfere with the absorption of fat soluble vitamins. (Caution) Emollient (ex lubricant, mineral oil); Lubricates the intestinal tract and retards colonic absorption of water, softening the stool, making it easier to pass. (Action); Usually effective within 8hours (Advantages); May interfere with absorption of fat soluble vitamins, can be aspirated, possibly resulting in a lipid pneumonia (Caution) Stimulant (Dulcolax); Drug promotes peristalsis by irritating the intestinal mucosa or stimulating nerve endings in intestinal wall (Action); Works more quickly than bulking agents (Advantages); Are the most abused laxatives on the market, causes lazy bowel syndrome, may affect absorption of vitamin D and calcium, not recommended for elderly patients because or prolonged action, alters electrolyte transport (caution) Osmotic (ex. Miralax); Drug draws water into intestine, resulting in a softer stool and inducing more frequent bowel movements (action); is used when rapid cleansing desired (advantages); May cause bloating and/or diarrhea, can produce dehydration, not recommended in patients with kidney disease or heart failure (caution)

A nurse performs a presurgical assessments of patients in an ambulatory care center. Which patient would the nurse report to the surgeon as possibly needing surgery to be postponed? A. A 19 year old patient who is a vegan B. an elderly patient who takes daily nutritional drinks C. A 43 year old patient who takes ginkgo biloba and and aspirin daily D. An infant who is breast feeding

C. A patient taking gingko biloba (an herbal), aspirin, and vitamin E (dietary supplement) may have to have surgery postponed due to an increased risk for excessive bleeding, because each of these substances have anticoagulant properties. Being a vegan should not affect surgery unless the patient has serious nutritional deficiencies. Drinking nutritional drinks and breast feeding do not adversely affect the outcomes of surgery.

For which patient might the nurse need to alter the plan of care based on the principles of the patient's chosen medical system? A. A patient who visits a chiropractor B. A patient who believes in a strong mind body connection C. A patient who is being treated by a naturopathic physician D. A patient who is being treated by an allopathic physician

C. A patient who is being treated by a naturopathic physician

A nurse is assessing the nutritional needs of patients. Which criteria indicates that a patient most likely needs TPN? A. Serum albumin level of 2.5g/dL or less B. Residual of more than 100mL C. Absence of bowel sounds D. Presence of dumping syndrome

C. Absence of bowel sounds

A nurse administers a dose of gentamicin, and the patient has an immediate reaction of hypotension, bronchospasms, and rapid, thready pulse. What is the next appropriate action of the nurse? A. Administer antiobiotic, antihistamines, and Isuprel B. Administer bronchodiliators, antihistamines, and vasodilators C. Administer epinephrine, antihistamines, and bronchodiliators D. Administer antihistamines, vasodilators, and bronchoconstrictors

C. Administer epinephrine, antihistamines, and bronchodiliators

A nurse determines that a patient has costovertebral tenderness. What condition is indicated by this physical finding? A. A bladder infection B. A bladder obstruction C. An inflamed kidney D. The presence of a kidney stone

C. An inflamed kidney

Which nursing action should the PACU nurse take to prevent postoperative complications in patients? A. Instruct the patient to avoid coughing to prevent injury to the incision B. Encourage the patient to breathe shallowly to prevent the collapse of alveoli C. Assist the patient to do leg exercises to increase venous return. D. Avoid turning the patient in bed until the incision is no longer painful

C. Assist the patient to do leg exercises to increase venous return

The nurse is changing a patient's ostomy appliance and observes that the peristomal skin is excoriated. What would be the nurse's priority intervention in this situation? A. Notify the primary care provider B. Suspect ischemia and notify the provider immediately C. Clean the outside of bag thoroughly when emptying D. Make sure that the appliance is not cut too large

C. Clean the outside of the bag thoroughly when emptying

A nurse is caring for a client who weighs 80kg (176lb) and is 1.6m (5'3) tall. Calculate her body mass index (BMI) and determine whether this client's BMI indicates that she is of a A. Healthy weight B. Overweight C. Obese

C. Obese. A BMI greater than 30 is considered obese.

When administering a subcutaneous injection to a patient, the needle pulls out of the skin when the skin fold is released. What would be the appropriate next action of the nurse in this situation? A. Pull out and discard the needle B. Discard the equipment and start the procedure from the beginning C. Engage safety shield on needle guard and discard needle appropriately D. Document the incident and inform the primary care provider

C. Engage safety shield on needle guard and discard the needle appropriately

The nurse is caring for a patient whose treatment has been based on the Ayurveda medical system. Which nursing intervention incorporates this patient's beliefs into the nursing plan? A. Basing practice on the yin-yang theory B. Preparing the patient for exercises that help the patient regulate qi C. Helping the patient to balance his dosha D. Including the patient's shaman in the plan of his care

C. Helping the patient to balance his dosha

A nurse in an outpatient surgical care center is admitting a client for a laparoscopic procedure. The client has a prescription for preoperative diazepam. Prior to administering the medication, which of the following actions is the nurse's priority? A. Teaching the client about the purpose of the medication B. Giving the medication at the administration time the provider prescribed C. Identifying the client's medication allergies D. Documenting the client's anxiety level

C. Identifying the client's medication allergies. The greatest risk to this client is injury from an allergic reaction. The priority action is to identify the client's allergies prior to medication administration The nurse should teach the client about the purpose of the medication to make sure the client understands why the provider prescribed it. However, another action is priority. The nurse should administer the medication at the time the provider prescribed that the client receives it to help prepare the client for the surgical procedure. However, another action is the priority. The nurse should document the client's anxiety level to have a baseline against which to measure the effectiveness of the medication. However, another action is the priority

A nurse administers a dose of an oral medication for hypertension to a patient who immediately vomits after swallowing the pill. What would be the appropriate initial action of the nurse in this situation? A. Readminister the medication and notify the primary care provider. B. Readminister the pill in a liquid form if possible C. Assess the vomit, looking for the pill. D. Notify the primary care provider

C. If a patient vomits immediately after swallowing an oral pill, the nurse should assess the vomit for the pill or fragments of it. The nurse should then notify the primary care provider to see if another dosage should be administered

A nurse is performing intermittent closed-catheter irrigation for a patient with an indwelling catheter. After attaching the syringe to the access port on the catheter, the nurse finds that the irrigant will not enter the catheter. What intervention would the nurse appropriately perform next? A. Apply pressure to the catheter to force the solution into the catheter B. Disconnect and reconnect the drainage system quickly C. Notify the primary care provider D. Change the catheter

C. If the irrigation solution will not enter the catheter, the nurse should not force the solution into the catheter; instead the nurse should notify the primary care provider and prepare to change the catheter

A nurse is administering a large volume cleansing enema to a patient prior to surgery. Once the enema solution is introduced, the patient complains of severe cramping. What would be the appropriate nursing intervention in this situation? A. Elevate the head of the bed 30 degrees and reposition the rectal tube B. Place the patient in a supine position and modify the amount of solution C. Lower the solution container and check the temperature and flow rate D. Remove the rectal tub and notify the primary care provider

C. If the patient complains of severe cramping with introduction of an enema solution, the nurse should lower the solution container and check the temperature and flow rate. If the solution is too cold or the flow rate too fast, severe cramping may occur

A nurse is assisting a patient to empty and change an ostomy appliance. When the procedure is finished, the nurse notes that the stoma is protruding into the bag. What would be the nurse's first action in this situation? A. Reassure the patient that this is a normal finding with a new ostomy B. Notify the primary care provider that the stoma is prolapsed C. Have the patient rest for 30 minutes to see if the prolapse resolves D. Remove the appliance and redo the procedure using a larger appliance

C. If the stoma is protruding into the bag after changin the appliance on an ostomy, the nurse should have the patient rest for 30 minutes. If the stoma is not back to normal size within that time, notify the physician. If the stoma is prolapsed, it may twist, resulting in impaired circulation to the stoma

A male patient is being transferred to the hospital from a long term care facility with a diagnosis of dehydration and urinary bladder infection. His skin is also excoriated from urinary incontinence. Which nursing diagnosis is most appropriate for this patient? A. Impaired Skin Integrity related to functional incontinence B. Urinary incontinence related to urinary tract infection C. Impaired Skin Integrity related to urinary bladder infection and dehydration D. Risk for Urinary Tract Infection related to dehydration

C. Impaired Skin Integrity related to urinary bladder infection and dehydration

A nurse is monitoring a patient post cardiac surgery. What action would help to prevent cardiovascular complications for this patient? A.Position the patient in bed with pillows placed under his knees to hasten venous return B. Keep the patient from ambulating until the day after surgery C. Implement leg exercises and turn the patient in bed every 2 hours D. Keep the patient cool and uncovered to prevent elevated temperature

C. Implement leg exercises and turn the patient in bed every 2 hours

Which fact should the nurse keep in mind when obtaining consent forms from patients scheduled to undergo surgery? A. A consent form is legal, even if the patient is confused or sedated B . The form that is signed is not a legal document and would not hold up in court C. In emergency situations, the doctor may obtain consent over the telephone D. The responsibility for securing informed consent from the patient lies with the nurse.

C. In emergency situations, the doctor may obtain consent over the telephone

The nurse is administering magnesium sulfate to a patient with constipation. Which mechanism of action would the nurse expect from this drug? A. Chemical stimulation of peristalsis B. Softening of the fecal material C. Increasing the intestinal bulk to enhance mechanical stimulation of the intestine D. Drawing water into the intestines to stimulate peristalsis

C. Increasing the intestinal bulk to enhance medical stimulation of the intestine

A nurse is caring for an alert, ambulatory, older resident in a long term care facility who voids frequently and has difficulty making it to the bathroom in time. Which nursing intervention would be most helpful for this patient? A. Teach the patient that incontinence is a normal occurrence with aging B. Ask the patient's family to purchase incontinence pads for the patient C. Teach the patient to perform Kegel exercises at regular intervals daily D. Insert an indwelling catheter to prevent skin breakdown

C. Kegel exercises may help a patient regain control o the micturition process. Incontinence is not a normal consequence of aging. Using absorbent products by remove motivation from the patient and caregiver to seek evaluation and treatment of the incontinence; they should be used only after careful evaluation by a health care provider. An indwelling catheter is the last choice of treatment

A nurse is providing a lecture on CAT to a group of patients in a rehabilitation facility. Which teaching point should the nurse include? A. CAT is a safe intervention used to supplement traditional care. B. Many patients use CAT as outpatients, but do not wish to continue as inpatients. C. Many nurses are expanding their clinical practice by incorporating CAT to meet the demands of patients. D. Most complementary and alternative are relatively new and their efficacy has not been established

C. Many nurses are expanding their clinical practice by incorporating CAT. Although CAT may seem totally safe, some therapies have led to harmful and at times lethal outcomes. Many patients use these types of therapies as outpatients and want to continue their use as inpatients. Although the use of most complementary and alternative therapies predates modern medicine, it was not until recently that nursing and medical schools began to teach about their use

A nurse is guiding a patient in the practice of meditation. Which teaching point is most useful in helping the patient to achieve a state of calmness, physical relaxation, and psychological balance? A. Teach the patient to always lie down in a comfortable position during meditation. B. Teach the patient to focus on multiple problems that the patient feels demand attention. C. Teach the patient to let distractions come and go naturally without judging them. D. Teach the patient to suppress distracting or wandering thoughts to maintain focus.

C. Meditators should have an open attitude by letting distractions come and go naturally without judging them. They should also maintain a specific, comfortable posture lying down, sitting, standing, walking, etc.; focus attention on a mantra, object, or breathing.; and not suppress distracting or wandering thoughts, instead they should gently bring attention back to focus.

What consideration based on gender would a nurse make when planning a menu for a male patient with well defined muscle mass? A.Men have a lower need for carbohydrates B. Men have a higher need for minerals C. Men have a higher need for proteins D. Men have a lower need for vitamins

C. Men have a higher need of proteins

A nurse is caring for a client who is 1 day postoperative following a total knee arthroplasty. The client states his pain level is 10 on a scale of 0 to 10. After reviewing the client's medication administration record, which of the following medications should the nurse administer? A. Meperidine 75mg IM B. Fentanyl 50mcg/hr transdermal patch C. Morphine 2mg IV D. Oxycodone 10mg PO

C. Morphine 2mg IV- The nurse should administer IV morphine because the onset is rapid, and the absorption of the medication into the blood is immediate, which provides the optimal response for a client who is reporting pain at a level of 10 Although Meperidine is a strong analgesic, the IM route of administration can allow for slow absorption, delaying the onset of pain relief. the IM route also can cause additional pain from the injection Although Fentanyl is a strong analgesic, the transdermal route of administration can allow for slow absorption, delaying the onset of pain relief Although oxycodone is a strong analgesic, the oral route of administration of this medication can allow for onset of pain relief in 10 to 15min, which can be a long time for a client who is reporting a pain at a level of 10

A nurse is assisting a patient who is experiencing pulmonary embolism. What would be the priority nursing intervention for this patient? A. Attempt to overhydrate the patient with fluids B. Instruct the patient to perform Valsalva's maneuver C. Place the patient in semi-Fowler's position. D. Assist the patient to ambulate every 2 to 3 hours

C. Place the patient in semi-Fowlers position

The nurse is collecting a clean catch specimen from a patient. Which nursing actions is performed correctly in this procedure? A. Clean the area at the meatus with antiseptic solution B. Collect the first 10mL of urine voided in the sterile specimen container C. Position the container near the meatus, and collect at least 10mL of urine D. Continue collecting the urine in the container until the bladder is empty

C. Position the container near the meatus, and collect at least 10mL of urine

The nurse is performing a nutritional assessment of an obese patient who visits a weight control clinic. What information should the nurse take into consideration when planning a weight reduction plan for this patient? A. To lose 1 pound/week, the daily intake should be decreased by 200 calories. B. One pound of body fat equals approximately 5000 calories C. Psychological reasons for overeating should be explored, such as eating as a release for boredom. D. Obesity is very treatable, and 50% of obese people who lose weight maintain the weight loss for 7 years

C. Psychological reasons for overeating should be explored, such as eating as a release for boredom.

A nurse is teaching a client who is lactating about taking medications. Which of the following actions should the nurse recommend to minimize in the entry of medication into breast milk? A. Drink 8oz milk with each dose of medication B. Use medications that have an extended half-life C. Take each dose right after breastfeeding D. Pump breast milk and freeze it prior to feeding the newborn

C. Taking medication immediately after breastfeeding helps minimize medication concentration in the next feeding. The intake of food or fluid with medication does not affect entry of medications intro breast milk. The clients should avoid medications that have an extended half life due to their increased entry into breast milk. Pumping and freezing breast milk does not affect entry of medications into breast milk

A medication order reads: "K-Dur, 20 mEq po b.i.d." When and how does the nurse correctly give this drug? A. Daily at bedtime by subcutaneous route B. Every other day by mouth C. Twice a day by the oral route D. Once a week by transdermal patch

C. The abbreviation b.i.d. refers to twice a day administration, po (by mouth) refers to administration by the oral route

A nurse discovers that she made a medication error. What should be the nurse's first response? A. Record the error on the medication sheet. B. Notify the physician regarding course of action C. Check the patient's condition to note any possible effect of the error D. Complete and incident report, explaining how the mistake was made.

C. The nurse's first responsibility is the patient- careful observation is necessary to assess for any effect of the medication error. The other nursing actions are pertinent, but only after checking the patient's welfare.

A patient has a fecal impaction. The nurse correctly administers an oil retention enema by: A. Administering a large volume of solution (500-1000mL) B. Mixing milk and molasses in equal parts for an enema C. Instructing the patient to retain the enema for at least 30 minutes D. Administering the enema while the patient is sitting on the toilet

C. The patient should be instructed to retain the enema solution for at least 30 minutes or as indicated in the manufacturer's instructions. The usual amount of solution administered with a retention enema is 150-200mL for an adult. The milk and molasses mixture is a carminative enema that helps to expel flatus. The patient should be instructed to lie on the left side of the bed as dictated by the patient condition and comfort

A perioperative nurse is preparing a patient for surgery for treatment of a ruptured spleen as a result of an automobile crash. The nurse knows that this type of surgery belongs in what category? A. Minor, diagnostic B. Minor, elective C. Major, emergency D. Major palliative

C. This surgery would involve a major body organ, has the potential for postoperative complications, requires hospitalization, and must be done immediately to save the patient's life. Elective surgery is a procedure that is preplanned by essentially healthy people. Diagnostic surgery is performed to confirm a diagnosis. Palliative surgery is not curative, rather it is done to relieve or reduce the intensity of an illness

A nurse is caring for a client who requires a low residue diet. The nurse should expect to se which of the following foods on the client's meal tray? A. Cooked barley B. Pureed broccoli C. Vanilla custard D. Lentil soup

C. Vanilla custard. A low residue diet consists of foods that are low in fiber and easy to digest. Dairy products and eggs, such as custard and yogurt, are appropriate for a low residue diet. Whole grains, such as barley and oats, are high in fiber and thus inappropriate components of a low residue diet. Raw and gas producing vegetables, such as broccoli and the cabbage in coleslaw, are high in fiber and thus inappropriate components of a low residue diet. Legumes, such as lentils and black beans, are high in fiber and thus inappropriate components of a low residue diet

Potassium chloride (K-Dur), 20mEq, is ordered. Potassium chloride is available as 10meq per tablet. How many tablets would the nurse administer

Dose on hand = Dose required ---------------- ----------------- Quantity on hand X(quantity desired) 10mEq = 20mEq ------- --------- 1 tablet X 10x= 20 X= 2 tablets

A nurse is performing digital removal of stool on a 74 year old female patient with a fecal impaction. During the procedure the patient tells the nurse she is feeling dizzy and nauseated, and then she vomits. What should be the nurse's next actions? A. Reassure the patient that this is a normal reaction to the procedure B. Stop the procedure, prepare to administer CPR, and notify the physician C. Stop the procedure, assess vital signs, and notify the physician D. Stop the procedure, wait five minutes, and then resume the procedure

C. When a patient complains of dizziness or lightheadedness and has nausea and vomiting during digital stool removal, the nurse should stop the procedure, assess heart rate and blood pressure, and notify the physician. The vagal nerve may have been stimulated

A nurse is administering heparin subcutaneously to a patient. What is the correct technique for this procedure? A. Aspirate before giving and gently massage after the injection. B. Do not aspirate; massage the site for 1 minute. C. Do not aspirate before or massage after the injection. D. Massage the site of the injection; aspiration is not necessary but will do no harm.

C. When giving heparin subcutaneously, the nurse should not aspirate or massage, so as not to cause trauma or bleeding in the tissues.

A nurse is reconstituting powdered medication in a vial. Which action is a recommended step in this process? A. The nurse draws up the proper amount of powered medication into the syringe. B. The nurse inserts the needle through the rubber stopper of the diluent vial. C. The nurse gently agitates the powdered medication vial to mix the powder and diluent completely. D. The nurse draws up the prescribed amount of medication while holding the syringe horizontally at eye level.

C. When reconstituting powdered medication in a vial, the nurse should draw up the appropriate amount of diluent into the syringe, insert the needle through the center of the self sealing stopper on the powdered medication vial, inject the diluent into the powdered medication vial, remove the needle from the vial and replace the cap, and gently agitate the vial to mix the powdered medication and diluent completely. The nurse should then draw up the prescribed amount of medication while holding the syringe vertically and at eye level.

The nurse is providing teaching for a postoperative patient regarding pain management. Which teaching point should the nurse include? A. Be sure to ask for your PRN medication when the pain becomes severe. B. If your pain is not relieved, ask your nurse to order a different medication C. You will receive pain medication by injection as long as you are NPO D. All postoperative pain control methods will be given by injection

C. You will receive pain medication by injection as long as you are NPO

A nurse is preparing a patient for a cesarean section and teaches her the effects of the regional anesthesia she will be receiving. Which effects would the nurse expect? (Select all that apply) A. Loss of consciousness B. Relaxation of skeletal muscles C. Reduction or loss of reflex action D. Localized loss of sensation E. Prolonged pain relief after other anesthesia wears off. F. Infiltrates the underlying tissues in an operative area

C.D. A localized loss of sensation and possible loss of reflexes occurs with a regional anesthetic. Loss of consciousness and relaxation of skeletal muscles occurs with general anesthesia. Prolonged pain relief after other anesthesia wears off and infiltration of the underlying tissues in an operative area occur with topical anesthesia

A nurse is teaching a patient with dysphagia how to eat safely. Which should the nurse encourage the patient to do? (Select all that apply). A. Tilt the head backward when swallowing B. Drink fluids when eating bites of solid food C. Reduce environmental stimuli to a minimum D. Make sure that the mouth is empty after eating E. Keep food in the front of the mouth when chewing

C.D. A person with dysphagia should concentrate on the acts of chewing and swallowing. Environmental stimuli can be distracting and can result in inadequate chewing or premature swallowing, which in turn can result in choking and aspiration. Ensuring that the mouth is empty after eating reduces the risk of aspiration.

While the nurse is applying a transdermal patch, the patient asks the nurse, "Why can't I just take a pill?" Which should the nurse explain are the advantages of administering a medication via a transdermal patch? (Select all that apply.) A. "It limits allergic responses." B. "It prevents drug interactions." C. "It delivers the drug over a period of time." D. "It bypasses the harsh acidic digestive system." E. "It provides a local rather than a systemic effect."

C.D. A transdermal patch placed on the skin gradually releases a predictable amount of medication that is absorbed into the bloodstream for a prescribed period of time. This approach maintains therapeutic blood levels and reduces fluctuations in circulating drug levels. A drug administered via a transdermal patch cannot be inactivated by gastric acidity. Drugs taken orally can cause gastric irritation, which can be avoided if the drugs are administered via a transdermal patch.

The nurse is teaching a patient how to use herbs and supplements as part of an integrated treatment plan. Which teaching points would the nurse include? (Select all that apply) A. Use the internet to buy herbs and supplements B. Whenever possible, buy products with more than one ingredient C. Buy herbs and supplements that are standardized D. Give the product adequate time to work E. Be knowledgeable about the product and its therapeutic actions F. Take a higher than recommended dose of hers to initiate the therapeutic effect.

C.D.E. Buy herbs and supplements that are standardized. Give the product adequate time to work. Be knowledgeable about the product and its therapeutic actions

The nurse researches factors that may alter nutrition. Which statements accurately describe factors that influence nutritional status? (Select all that apply) A. During adulthood, there is an increase in the basal metabolic rate with each decade B. Because of the changes related to aging, the caloric needs of the older adult increase C. During pregnancy and lactation, nutrient requirements increase D. Nutritional needs per unit of body weight are greater in infancy than at any other time in life. E. Men and women differ in their nutrient requirements F. Trauma, surgery, and burns decrease nutrient requirements

C.D.E. During pregnancy and lactation, nutrient requirements increase. Nutritional needs per unit of body weight are greater in infancy than at any other time in life. Men and women differ in their nutrient requirements

The nurse is selecting antidiarrheal medications for patients with diarrhea. Which statements accurately describe the action of specific antidiarrheal medication? (Select all that apply) A. Kaopectate does not interfere with the absorption of other oral medications. B. Imodium is an antimicrobial against bacterial and viral pathogens C. Imodium is a nonaddictive antidiarrheal medication that has a longer duration of action than Lomotil D. Pepto-Bismol contains salicylates; a physician should be consulted before giving it to children or patients taking aspirin E. Paregoric contains morphine and may be addictive F. Lomotil has a longer duration of action than Imodium

C.D.E. Imodium is a nonaddictive antidiarrheal medication that has a longer duration of action than Lomotil. Pepto-Bismol contains salicylates; a physician should be consulted before giving it to children or patients taking aspirin. Paregoric contains morphine and may be addictive

A home health care nurse is teaching a patient and caregivers how to administer an enteral feeding. Which teaching points are appropriate? (Select all that apply) A.When checking residuals, routinely discard residuals to prevent an acid base imbalance B. When cleaning around a gastric tube insertion site, be careful not to rotate the guard after cleaning around it C. Check for leaking of gastric contents around the insertion site (is guard too loose or ballon not filled adequately?) D. Clean around the gastric tube with soap and water, making sure it is adequately rinsed E. Keep the head elevated while delivering a gastric feeding and for approximately an hour after the feeding F. Mark gastrostomy tubes with an indelible marker and check the mark to make sure it is at the level of the abdominal wall.

C.D.E.F. Check for leaking of gastric contents around the insertion site (is guard too loose or balloon not filled adequately?). Clean around the gastric tube with soap and water, making sure it is adequately rinsed. Keep the head elevated while delivering a gastric feeding and for approximately an hour after the feeding. Mark gastrostomy tubes with indelible marker and check the mark to make sure it is at the level of the abdominal wall.

A nurse who is planning a diet for a patient who has anorexia chooses nutrients that supply energy to the body including; (Select all that apply) A. Vitamins B. Minerals C. Carbohydrates D. Protein E. Water F. Lipids

C.D.F. Carbohydrates, Protein, Lipids

Which effects of medications should the nurse consider when assessing patients for surgical risks? (Select all that apply) A. Diuretics may precipitate hemorrhage B. Anticoagulants may cause electrolyte imbalances C. Diuretics may cause respiratory depression from anesthesia D. Tranquilizers may increase the hypotensive effect of anesthetic agents E. Adrenal steroids may cause respiratory paralysis F. Abrupt withdrawal from adrenal steroids may cause cardiovascular collapse in long term users

C.D.F. Diuretics may cause respiratory depression from anesthesia. Tranquilizers may increase the hypotensive effect of anesthetic agents. Abrupt withdrawal from adrenal steroids may cause cardiovascular collapse in long term users

Which actions would the nurse perform when administering a subcutaneous injection correctly? (Select all that apply) A. If using the outer aspect of the upper arm, place the patient's arm over the chest with the outer area exposed. B. Remove the needle cap with the dominant hand, pulling it straight off C. Grasp and bunch the area surrounding the injection site or spread the skin taut at the site D. Inject the needle quickly at an angle of 45 to 90 degrees E. If blood appears when aspirating, withdraw the needle and reinject it at another site F. After removing the needle, do not massage the area to prevent hematoma formation

C.D.F. Grasp and bunch the area surrounding the injection site or spread the skin taut at the site. Inject the needle quickly at an angle of 45 to 90 degrees After removing the needle, do not massage the area to prevent hematoma formation

A nurse who is planning menus for a patient in a long term care facility takes into consideration the effects of foods and fluids on bowel elimination. Which examples correctly describe these effects? (Select all that apply) A. Patients with lactose intolerance may experience diarrhea or gas when consuming starchy foods. B. A patient who is constipated should eat eggs and pasta to relieve the condition. C. Patients who are constipated should eat more fruits and vegetables D. Patients experiencing flatulence should avoid gas producing foods such as cauliflower and onions E. Alcohol and coffee and tend to have a constipating effect on patients F. Patients with food intolerances may experience altered bowel elimination

C.D.F. Patients who are constipated should eat more fruits and vegetables. Patients who are experiencing flatulence should avoid gas producing foods such as cauliflower and onions. Patients with food intolerances may experience altered bowel elimination

The nurse is administering an oil retention enema to a patient. Which nursing actions in this procedure are performed correctly? (Select all that apply) A.The nurse chooses a large rectal tube B. The nurse instills the solution into the rectum by applying gentle pressure on the collapsible solution container. C. The nurse administers the oil retention enema at body temperature D. The nurse instructs the patient to retain the oil for at least 30minutes E. The nurse administers a cleansing enema prior to the oil retention enema F. The nurse administers a cleansing enema after the oil retention enema

C.D.F. The nurse administers the oil retention enema at body temperature. The nurse instructs the patient to retain the oil for at least 30 minutes. The nurse administers a cleansing enema after the oil retention enema

The nurse is administering a medication to a patient via a nasogastric tube. Which are accurate guidelines related to this procedure? (Select all that apply) A. Crush the enteric coated pill for mixing in a liquid B. Flush open the tube with 60mL of very warm water C. Check for proper placement of the nasogastric tube D. Give each medication separately and flush with water between each drug. E. Lower the head of the bed to prevent reflux. F. Adjust the amount of water used if patient's fluid intake is restricted

C.D.F. The nurse should use the recommended procedure for checking tube placement prior to administering medications. The nurse should also give each medication separately and flush with water between each drug and adjust the amount of water used if fluids are restricted. Enteric coated medications should not be crushed, the tube should be flushed with 15 to 30mL of water, and the head of the bed should be elevated to prevent reflux.

A primary healthcare provider prescribes a liquid medication that has an unpleasant taste for a school-aged child. What should the nurse do to facilitate administration of this medication? (Select all that apply.) A. Mix it with the child's favorite food. B. Teach that the taste only lasts a short time. C. Give an ice pop just before giving the medication. D. Have a parent administer the medication if present. E. Offer the child the choice of a spoon, needleless syringe, or dropper.

C.E. An ice pop just before administration may numb the taste buds and minimize the unpleasant taste of the medication. Offering the child a choice supports a sense of control. Involvement in decisions limits resistance.

A patient has a decreased hemoglobin level because of a low intake of dietary iron. Which foods should the nurse teach the patient are excellent sources of iron? (Select all that apply.) A. Eggs B. Fruit C. Meat D. Bread E. Spinach

C.E. Meat, especially liver, is an excellent source of iron. Three ounces of meat contain 1.6 to 5.3 mg of iron depending on the type of meat and whether it is a regular or lean cut. Spinach is an excellent source of iron. A half cup of boiled spinach contains 3.2 mg of iron.

The instructions with a medication states to use the Z-track method. Which actions should the nurse implement that are specific to this procedure? (Select all that apply.) A. Pinch the site throughout the procedure B. Massage the site after the needle is removed C. Add 0.3 to 0.5 ml of air after drawing up the correct dosage D. Remove the needle immediately after the medication is injected E. Change the needle after the medication is drawn into the syringe

C.E. The injection of a small amount of air after the medication is administered instills air into the Z track, and this helps keep the medication deeply seated in the muscle. The Z-track method is used with viscid or caustic solutions. Changing the needle ensures that medication is not on the outside of the needle, which prevents tracking of the medication into subcutaneous tissue during needle insertion.

A nurse is preparing a brochure to teach patients how to prevent urinary tract infections. Which teaching points would the nurse include? (Select all that apply) A. Wear underwear with a synthetic crotch B. Take baths rather than showers C. Drink eight to ten 8oz glasses of water per day D. Drink a glass of water before and after intercourse and void afterwards E. Limit caffeine containing beverages F. Drink 10oz of cranberry or blueberry juice daily

C.E.F. It is recommended that a healthy adult drink eight to ten 8oz glasses of fluid daily, limit caffeine because it is irritating to the bladder mucosa, and drink 10oz of cranberry or blueberry juice daily to help prevent bacteriuria. It is also recommended to wear underwear with a cotton crotch, take showers rather than baths, and drink two glasses of water before and after sexual intercourse and void immediately after intercourse

Which nursing interventions would be appropriate for a patient recovering from a surgical procedure? (Select all that apply) A. Teach the patient to suppress urges to cough in order to protect the incision B. Encourage the patient to take frequent shallow breaths to improve lung expansion and volume C. Place the patient in semi-Fowlers position to perform deep breathing exercises every 1-2 hours for the first 24-48 hours after surgery and as necessary thereafter. D. Encourage the patient to lie still in bed with the incision facing upward to prevent putting pressure on the stitches E. Teach the patient the appropriate leg exercises to increase venous blood return from the legs F. Encourage the patient to use incentive spirometry 10 times each waking hour for the first 5 days after surgery

C.E.F. Place the patient in semi-Fowlers position to perform deep breathing exercises every 1-2 hours for the first 24-48 hours after surgery and as necessary thereafter. Teach the patient the appropriate leg exercises to increase venous blood return from the legs. Encourage the patient to use incentive spirometry 10 times each waking hour for the first 5 days after surgery

Table 28-1 Common Types of Drug Preparations

Capsule- Powder or gel form of an active drug enclosed in a gelatinous container; may also be called a liquigel Elixir- Medication in a clear liquid containing water, alcohol, sweeteners, and flavor Enteric coated- A tablet or pill coated to prevent stomach irritation Extended release- Preparation of a medication that allows for slow and continuous release over a predetermined period; may also be referred to as CR or CRT (controlled release) SR (sustained release) SA (sustained action) LA (long acting) or TR (time released) Liniment- Medication mixed with alcohol, oil, or soap, which is rubbed on the skin Lotion- Drug particles in a solution for topical use Lozenge- small, oval, round, or oblong preparation containing a drug in a flavored or sweetened base, which dissolves in the mouth and releases the medication, also called troche Ointment- Semisolid preparation containing a drug to be applied externally; also called unction Pill- Mixture of a powdered drug with a cohesive material, may be round or oval Powder- Single or mixture of finely ground drugs Solution- A drug dissolved in another substance ( in an aqueous solution) Suppository- An easily melted medication preparation in a firm base such as gelatin that is inserted into the body (rectum, vagina, urethra) Suspension- Finely divided, undissolved particles in a liquid medium; should be shaken before use Syrup- Medication combined in a water and sugar solution Tablet- Small, solid dose of medication, compressed or molded; may be any color, size, or shape; enteric coated tablets are coated with a substance that is insoluble in gastric acids to reduce gastric irritation by the drug Transdermal patch- Unit dose of medication applied directly to skin for diffusion through skin and absorption into the bloodstream

A nurse is administering an injection of insulin to a 5 year old who has juvenile diabetes. Which statement by the nurse would take into consideration this child's developmental level? A. "Don't worry, this won't hurt a bit" B. "If you are brave and don't cry, I will give you a sticker." C. "Try not to move, or this will hurt more." D. "You will feel just a little pinch."

D. "You will feel just a pinch."

A nurse is estimating caloric requirements for a female patient whose healthy weight is 120 pounds and whose activity level is moderate. This patient's recommended total daily calories is: A. 1200 B. 1440 C. 1560 D. 1680

D. 1680

Captopril (Capoten), 12.5mg PO, is ordered. Captopril is available as 25mg tablets. How many tablets would the nurse administer?

Dose on hand = Dose required ---------------- ----------------- Quantity on hand X(quantity desired) 25mg =12.5mg ------ ------- 1 tab X 25X= 12.5mg X= 0.5 or 1/2 tablet

Active Learning Scenario: Chapter 50 Include the definition and an example of each of the five pregnancy risks

Category A: There is no evidence of risk to a fetus from taking the medication during pregnancy, according to adequate and well controlled studies. Ferrous sulfate, an iron supplement, is a Category A medication Category B: There is no evidence of risk to an animal fetus according to studies, but there are no adequate and well controlled studies of pregnant women. Or, there is evidence of risk to an animal fetus, but controlled studies of pregnant women show no evidence of risk to the fetus. Esomeprazole, and anitulcer medication, is in Category B Category C: Studies have demonstrated adverse effects on animal fetuses, but there are no adequate and well controlled studies of pregnant women. Glipizide, and antidiabetes medication, is in Category C Category D: Studies have demonstrated adverse effects on human fetuses according to data from investigational or marketing experience, but potential benefits from the use of the medication during pregnancy might warrant its use. Sorafenib, and antineoplastic medication, is in Category D Category X: Studies have demonstrated adverse effects on animal and human fetuses, according to studies and data from investigational or marketing experience. Pregnancy is a contraindication for the use of the medication because the risks outweigh the potential benefits. Estradiol, an estrogen replacement, is a Category X medication

Table 35-7 Modified Consistency Diets

Clear Liquid Diets: Composed only of clear fluids or foods that become fluid at body temperature. Requires minimal digestion and leaves minimal residue. Includes clear broth, coffee, tea, clear fruit juices (apple, cranberry, grape), gelatin, popsicles. commercially prepared clear liquid supplements; Indications: Preparation for bowel surgery and lower endoscopy; acute gastrointestinal disorders; initial postoperative diet Pureed Diet: Also known as a blenderized liquid diet because the diet is made up of liquids and foods blenderized to liquid form. All foods are allowed; Indications: After oral and facial surgery; chewing and swallowing difficulties Mechanically altered diet: Regular diet with modifications for texture. Excludes most raw fruits and vegetables and foods with seeds, nuts, and dried fruits. Foods are chopped, ground, mashed or soft; Indications: Chewing and swallowing difficulties; after surgery to the head, neck, or mouth

Table 36-1 Characteristics of Urine

Color: A freshly voided specimen is pale yellow, straw colored, or amber, depending on its concentration (normal finding); Urine is darker than normal when it is scanty and concentrated. Urine is lighter than normal when it is excessive and diluted. Certain drugs, such as cascara, l-dopa, and sulfonamides, alter the color of urine. Some foods can alter the color; for example beets can cause the urine to appear red. (special considerations) Odor: Normal urine smell is aromatic. As urine stands, it often develops an ammonia odor because of bacterial action. (normal findings) Some foods cause urine to have a characteristic odor; for example, asparagus causes urine to have a strong musty odor. Urine high in glucose content has a sweet odor. Urine that is heavily infected has a fetid odor. (special considerations) Turbidity: Fresh urine should be clear or translucent; as urine stands and cools; it becomes cloudy. (Normal findings). Cloudiness observed in freshly voided urine is abnormal and may be due to the presence of red blood cells, white blood cells, bacteria, vaginal discharge, sperm, or prostatic fluid (special considerations) pH: The normal pH is about 6.0, with a range of 4.6 to 8. (Urine alkalinity or acidity may be promoted through diet to inhibit bacterial growth or urinary stone development or to facilitate the therapeutic activity of certain medications) Urine becomes alkaline on standing when carbon dioxide diffuses into the air. (Normal findings) A high protein diet causes urine to become excessively acidic. Certain foods tend to produce alkaline urine, such as citrus fruits, dairy products, and vegetables, especially legumes. Certain foods such as meats tend to produce acidic urine. Certain drugs influence the acidity or alkalinity of urine; for example, ammonium chloride produces acidic urine, and potassium citrate and sodium bicarbonate produce alkaline urine. (Special considerations) Specific gravity: This is a measure of the concentration of dissolved solids in the urine. The normal range is 1.015 to 1.025. (Normal findings) Concentrated urine will have a higher than normal specific gravity; diluted urine will have a lower than normal specific gravity. In the absence of kidney disease, a high specific gravity usually indicates dehydration and a low specific gravity indicates over-hydration (Special considerations) Constituents: Organic constituents of urine include urea, uric acid, creatinine, hippuric acid, indican, urene pigments, and undetermined nitrogen. Inorganic constituents are ammonia sodium, chloride, traces of iron, phosphorous, sulfur, potassium, and calcium. (Normal findings) Abnormal constituents of urine include blood, pus, albumin, glucose, ketone bodies, casts, gross bacteria, and bile. (Special considerations)

Table 27-1 Beliefs underlying complementary and alternative therapies and allopathic therapies

Complementary and Alternative therapies: Mind, body, spirit are integrated and together influence health and illness. Health is a balance of body systems: mental, social, and spiritual, as well as physical Symptoms are a sign or reflection of a deeper instability within the person; restoring physical and mental harmony will alleviate the symptoms. Healing is a slow process that involves the whole person. Emphasis is on health. Healing is done by the patient; care is individualized Allopathic Therapies: Illness occurs in either the mind or the body, which are separate entities Health is the absence of disease The main causes of illness are considered to be pathogens (bacteria or viruses) or biochemical imbalances Curing seeks to destroy the invading organism or repair the affected part. Emphasis is on disease and high technology. Drugs, surgery, and radiation are among the key tools for dealing with medical problems

Table 35-6 Selected Therapeutic Diets

Consistent Carbohydrate Diet: Total daily carbohydrate content is consistent; emphasizes general nutritional balance. Calories based on attaining and maintaining healthy weight. High Fiber and heart healthy fats encouraged, sodium and saturated fats are limited; Indications: Type 1 and 2 diabetes, gestational diabetes, impaired glucose tolerance Fat restricted diet: Low fat diets are intended to lower the patient's total intake of fat; Indications: Chronic cholecystitis (inflammation of the gallbladder) to decrease gallbladder stimulation; cardiovascular disease; to help prevent atherosclerosis High Fiber: Emphasis on increased intake of foods high in fiber; Indications: Prevent or treat constipation, irritable bowel syndrome; diverticulosis Low Fiber: Fiber limited to <10g/day; Indications: Before surgery, ulcerative colitis, diverticulitis, Crohn's disease Sodium restricted: Sodium limit may be set at 500-3000mg/day; Indications: Hypertension, heart failure, acute and chronic renal disease, liver disease Renal diet: Reduce workload on kidneys to delay or prevent further damage; control accumulation of uremic toxins. Protein restriction 0.6-1g/kg/day; sodium restriction 1000-3000mg/day; Potassium and fluid restrictions dependent on patient situation; Indications: Nephrotic syndrome; chronic kidney disease; diabetic kidney disease

A nurse is helping an overweight female patient to devise a meal plan to lose 2 pounds per week. How many calories would the patient need to delete per day in order to accomplish this goal? A. 250 calories B. 500 calories C. 750 calories D. 1000 calories

D. 1lb (0.45kg) of body fat equals about 3500cal. Therefore, to gain or lose 1lb (0.45kg) in a week, a daily calorie intake should be increased or decreased, respectively, by 500 cal (3500 cal divided by 7= 500cal/day). Similarly, a weight gain or loss of 2lb (0.9kg) per week would require an adjustment of 1000cal/day

A nurse is scheduling tests for a patient who has been experiencing epigastric pain. The physician ordered the following tests: (a) barium enema, (b) fecal occult blood test, (c) endoscopic studies, (d) upper gastrointestinal series. Which is the correct order in which the tests would normally be performed A. c,b,d,a B. d,c,a,b C. a,b,d,c D. b,a,d,c

D. A fecal occult blood test should be done first to detect gastrointestinal bleeding. Barium studies should be performed next to visualize gastrointestinal structures and reveal any inflammation, ulcers, tumors, strictures, or other lesions. A barium enema and routine radiography should precede an upper gastrointestinal series because retained barium from an upper gastrointestinal series could take several days to pass through the gastrointestinal tract and cloud anatomic detail on the barium enema studies. Noninvasive procedures usually take precedence over invasive procedures, such as endoscopic studies, when sufficient diagnostic data can be obtained from them

Which of the following nursing diagnoses would be most appropriate for a patient with a body mass index (BMI) of 18? A. Risk for Imbalanced Nutrition: More Than Body Requirements B. Imbalanced Nutrition: More Than Body Requirements C. Readiness for Enhanced Nutrition D. Imbalanced Nutrition: Less Than Body Requirements

D. A patient with a body mass index (BMI) of 18 is considered underweight, therefore a diagnosis of Imbalanced Nutrition: Less than Body Requirements is appropriate. The patient is not at risk for imbalanced nutrition because it is already a problem and certqainly is not experiencing nutrition that is more than body requirements. Readiness for Enhanced Nutrition is appropriate when there is a healthy pattern of nutrient intake that is sufficient for meeting metabolic needs and can be strengthened and enhanced.

A nurse is changing the stoma appliance on a patient's ileal conduit. Which characteristic of the stoma would alert the nurse that the patient is experiencing ischemia? A. The stoma is hard and dry B. The stoma is a pale pink color C. The stoma is swollen D. The stoma is a purple-blue color

D. A purple-blue stoma may reflect compromised circulation or ischemia. A pale stoma may indicate anemia. The stoma may be swollen at first, but that condition should subside with time. A normal stoma should be moist and dark pink to red in color

A patient who is moved to a hospital bed following throat surgery is ordered to receive continuous tube feedings through a small bore nasogastric tube. Following the placement of the tube, which nursing action would the nurse initiate to ensure correct placement of the tube? A. Auscultate the bowel sounds B. Measure the gastric aspirate pH C. Measure the amount of residual in the tube. D. Order radiographic examination of the tube

D. Although a radiographic examination exposes the patient to radiation and is costly, it is still the most accurate method to check correct tube placement. Other methods that can be used are aspiration of gastric contents and measurement of the pH of the aspirate. The recommended method for checking placement, other than a radiograph, is measuring the pH of the aspirate. Visual assessment of aspirated gastric contents is also suggested as a tool to check placement. In addition, the length of the exposed tube is measured after insertion and documented. Tube length should be checked and compared with this initial measurement, in conjunction with the previous two methods for checking tube placement. The auscultatory method is considered inaccurate and unreliable. Measurement of residual amount does not confirm placement.

A nurse is preparing a presentation about basic nutrients for a group of high school athletes. She should explain that which of the following nutrients provides the body with the most energy? A. Fat B. Protein C . Glycogen D. Carbohydrates

D. Carbohydrates. Carbohydrates are the body's greatest energy source; providing energy for cells is their primary function. They provide glucose, which burns completely and efficiently without end products to excrete. They are also a ready source of energy, and they spare proteins from depletion. Although the body gets about half of its energy supply from fat, it is an inefficient means of obtaining energy. It produces end product the body has to excrete, and it requires energy from another source to burn the fat. Protein can supply energy, but it has other very essential and specific functions that only it can perform. So it is not the body's priority energy source. Glycogen, which the body stores in the liver, is a backup source of energy, not a primary or priority source

A nurse is administering medication to a 78 year old female patient who experienced symptoms of stroke. When administering the medication prescribed for her, the nurse should be aware that this patient has an increased possibility of drug toxicity due to which of the following age related factors? A. Decreased adipose tissue and increased total body fluid in proportion to total body mass. B. Increased number of protein binding sites C. Increased kidney function, resulting in excessive filtration and excretion D. Decline in liver function and production of enzymes needed for drug metabolism

D. Decline in liver function and production of enzymes needed for drug metabolism

Table 35-4 Summary of Macrominerals and Microminerals

Macrominerals: Calcium: Milk and dairy products, canned fish with bones, greens; Bone and tooth formation, blood clotting, nerve transmission, muscle contraction; Tetany, osteoporosis (Signs of deficiency); Renal calculi in susceptible people Phosphorous: Milk and milk products, soft drinks, processed foods; Bone and tooth formation, acid base balance, energy metabolism; Hypophosphatemia, anorexia, muscle weakness (signs of deficiency), Hyperphosphatemia, symptoms of hypocalcemic tetany (signs of excess) Magnesium: Green leafy vegetables, nuts, beans, grains; Bone and tooth formation, protein synthesis, carbohydrate metabolism; Hypomagnesemia, weakness, muscle pain, poor heart function (signs of deficiency); Hypermagnesemia, CNS depression, coma, hypotension (signs of excess) Sulfur (provided by adequate amounts of protein): Meat, eggs, milk, dried peas and beans, nuts; Promotes certain enzyme reactions and detoxification reactions; None known (signs of deficiency) none known (signs of excess) Sodium: Salt, processed foods; Major ion of extracellular fluid, fluid balance, acid base balance; Hyponatremia, muscle cramps, cold and clammy skin (signs of deficiency); Edema, weight gain, high blood pressure if susceptible Potassium: Whole grains, fruits, leafy vegetables; Major ion of intracellular fluid, fluid balance, acid base balance; Hypokalemia, muscle cramps and weakness, irregular heartbeat (signs of deficiency), Hyperkalemia, irritability, anxiety, cardiac, arrhythmia, heart block (signs of excess) Chloride: salt; Component of HCI in stomach, fluid balance, acid balance; Hypochloremia, muscle spasms, alkalosis, depressed respirations (signs of deficiency); Hypercholemia, acidosis (signs of excess) Microminerals: Iron: Liver, lean meats, enriched and whole grain breads and cereals; Oxygen transport by way of hemoglobim, constituent of enzyme systems; Microcytic anemia, pallor, decreased work capacity, fatigue, weakness (signs of deficiency); Hemosiderosis, acute iron poisoning from accidental overdose leads to GI symptoms and possible shock (signs of excess) Iodine: Iodized salt, seafood, food additives; component of thyroid hormones; goiter (signs of deficiency); acne like lesions (signs of excess) Zinc: Oysters, liver, meats, dried peas and beans, nuts; Tissue growth, sexual maturation, immune system functioning (signs of distress); Anorexia, nausea, vomiting, diarrhea, muscle pain, lethargy (signs of excess) Copper: Liver, shellfish, grains, dried peas and beans; Aids in metabolism and activity of some enzymes; Anemia, altered bone formation, hypercholesterolemia (signs of distress); Nervous system disturbances, vomiting (signs of excess) Manganese: whole grains, nuts, dried peas and beans, fruit; Part of enzymes needed for protein and energy metabolism; Poor reproductive performance, growth retardation (signs of distress); None known (signs of excess) Fluoride: Fluoridated water, fish, tea; Tooth formation and integrity, bone formation and integrity; Tooth decay, may increase risk for osteoporosis (signs of distress); Mottling and discoloration of tooth enamel (signs of excess) Chromium: Whole grains, meats; Cofactor for insulin, proper glucose metabolism; Impaired glucose tolerance, insulin resistance (signs of distress); none known (signs of excess) Selenium: Wheat (if grown in high selenium soil), organ meats; antioxidant; none known (signs of distress); loss of hair, brittle fingernails, fatigue (signs of excess) Molybdenum: Liver, whole grains, dried peas and beans, organ meats; oxidizes sulfur and products of sulfur metabolism; none known (signs of deficiency); Interferes with copper metabolism (signs of excess) Cobalt: Organ meats; Essential component of vitamin B12; none known (signs of deficiency); none known (signs of excess)

A nurse is caring for a hospitalized patient who states: I feel so sick all the time, my aura must be disturbed by all of these bad force fields. What is an appropriate NANDA diagnosis for this patient? A. Social Isolation B. Impaired coping C. Hopelessness D. Disturbed energy field

D. Disturbed energy field

What action should the nurse take when giving an intramuscular injection using the Z track method? A. Use a needle at least 1 inch long B. Apply pressure to the injection site C. Inject the medication quickly and steadily withdraw the needle D. Do not massage the site because it may cause irritation

D. Do not massage the site because it may cause irritation

A nurse is administering an intradermal injection to a patient for a skin allergy test. When the nurse is finished, there is no sign of a wheal or blister at the site on injection. What is the nurse's best action in this situation? A. Choose another site and re-inject the medication B. Prepare another syringe and administer it to the patient at the same site C. Document the administration as correctly administered D. Document the administration and inform the primary care provider

D. Document the administration and inform the primary care provider

The nurse is caring for a patient who is scheduled for an esophagogastroduodenoscopy (EGD). What action would the nurse take to prepare the patient for this procedure? A. Ensure that the patient ingests a gallon of bowel cleanser such as GoLytely, in a short period of time B. Inform the patient that a chalky tasting barium contrast mixture will be given to drink before the test C. Provide a light meal before the test and administer two fleet enemas. D. Ensure that the patient fasts 6-12 hours before the test as per policy

D. Ensure that the patient fasts 6-12 hours before the test as per policy

What intervention would a nurse perform when physically preparing a patient undergoing surgery? A. Shave the area of the incision with a razor B. Empty the patient's bowel of feces C. Do not allow the patient to eat or drink anything for 8 to 12 hours before surgery D. Ensure that the patient is well nourished and hydrated

D. Ensure that the patient is well nourished and hydrated

A nurse is teaching a patient with frequent constipation how to implement a bowel training program. What is a recommended teaching point? A. Using a diet that is low in bulk B. Decreasing fluid intake to 1000mL C. Administering an enema once a day to stimulate peristalsis D. Allowing ample time for evacuation

D. For a bowel training program to be effective, the patient must have ample time for evacuation (usually 20-30 minutes). Fluid intake is increased to 2500-3000mL, food high in bulk is recommended as part of the program, and a daily enema is not administered in a bowel training program. A cathartic suppository may be used 30 minutes before the patient's usual defecation time to stimulate peristalsis

A nurse is teaching a client about taking multiple oral medications at home to include time-release capsules, liquid medications, enteric-coated pills, and opioids. which of the following statements should the nurse identify as an indication that the client understands the instructions? A. I can open the capsule with the beads in it and sprinkle them on my oatmeal B. If I am having difficulty swallowing, I will add the liquid medication to a batch of pudding C. I can crush the pills with the coating on them D. I will eat two crackers with the pain pills

D. I will eat two crackers with the pain pills- The clients should take irritating medications, such as analgesics, with small amounts of food. It can help prevent nausea and vomiting Although this might help a client who has swallowing issues, it is essential for the client to swallow enteric-coated or time-release medications whole. Although adding a liquid medication to food is helpful if the client is having difficulty swallowing, he should not mix the medication with large amounts of food or beverages in case he cannot consume the entire quantity. The client must not crush enteric coated or time release preparations. He must swallow them whole

A nurse is caring for a postoperative patient who is experiencing pain. Which CAT involves active participation by the patient and is effective for pre or postoperative pain control? A. Acupuncture B. Therapeutic touch C. Botanical supplemenets D. Guided imagery

D. Imagery involves using all five senses to imagine an event or body process unfolding according to plan. A patient can be encouraged to "go to a favorite place." With the other modalities, the patient is more passive.

Data must be collected to evaluate the effectiveness of a plan to reduce urinary incontinence in an older adult patient. Which information is the least important for the evaluation process? A. The incontinence pattern B. State of physical mobility C. Medications being taken D. Age of patient

D. Incontinence is not a natural consequence of the aging process. All the other factors are necessary information for the plan of care

While assessing a patient in the PACU, a nurse notes increased wound drainage, restlessness, a decreasing blood pressure, and an increase in the pulse rate. The nurse interprets these findings as most likely indicating: A. Thrombophlebitis B. Atelectasis C. Infection D. Hemorrhage

D. Increased wound drainage, restlessness, decreasing blood pressure, and increasing pulse rate are assessment findings that indicate hemorrhage. Manifestations of thrombophlebitis are pain and cramping in the calf or thigh of the involved extremity, redness and swelling in the affected area, elevated temperature, and an increase in the diameter of the involved extremity. Manifestations of atelectasis include decreased lung sounds over the affected area, dyspnea, cyanosis, crackles, restlessness, and apprehension. Signs of infection include elevated white blood count and fever.

The nurse is caring for a male patient who has a urinary obstruction and is not a candidate for surgery. What intervention would the nurse expect the health care provider to perform? A.Insertion of an indwelling urethral catheter B. Insertion of a suprapubic catheter C. Insertion of a straight catheter D. Insertion of a urologic stent

D. Insertion of a urologic stent

A nurse is explaining the rationale for performing leg exercises after surgery. Which reason would the nurse include in the explanation? A. Promote respiratory function B. Maintain functional abilities C. Provide diversional activities D. Increase venous return

D. Leg exercises in the postoperative period increase venous return. As a result, the patient has a decreased risk for thrombophlebitis and emboli

A nurse preparing medication for a patient is called away to do an emergency. What should the nurse do? A. Have another nurse guard the preparations B. Put the medications back in the containers C. Have another nurse finish preparing and administering the medications D. Lock the medications in a room and finish them upon return

D. Lock the medications in a room and finish them upon return

While a nurse is administering a cleansing enema, the client reports abdominal cramping. Which of the following actions should the nurse take? A. Have the client hold his breath briefly and bear down B. Discontinue the fluid instillation C. Remind the client that cramping is common at this time. D. Lower the enema fluid container

D. Lower the enema container. To relieve the client's discomfort, the nurse should slow the rate of instillation by reducing the height of the enema solution container. The nurse should have the client take slow, deep breaths to relax and ease discomfort. The nurse should stop the instillation if the client's abdomen becomes rigid or distended or if the nurse notes bleeding from the rectum. Reminding the client that cramping is common is nontherapeutic because it implies that the client must tolerate the discomfort and that the nurse cannon or will not do anything to ease it

A medication order reads: "Hydromorphone, 2mg IV every 3 to 4 hours PRN pain." The prefilled cartridge is available with a label reading Hydromorphone 2mg/ 1 mL. The cartridge contains 1.2 mL fo hydromorphone. Which nursing action is correct? A. Give all the medication in the cartridge because it expanded when it was mixed B. Call the pharmacy and request the proper dose C. Refuse to give the medication D. Dispose of 0.2 mL correctly before administering the drug.

D. Many cartridges are overfilled, and some of the medication needs to be discarded. Always check the volume needed to provide the correct dose with the volume in the syringe. Giving the excess medication in the cartridge may result in adverse effects for the patient. For this dose, it is not necessary to call the pharmacy or refuse to give the medication, provided the order is written correctly

A nurse is explaining pain control methods to a patient undergoing bowel resection. The patient is interested in the PCA pump and asks the nurse to explain how it works. What would be the nurse's correct response? A. The pump allows the patient to be completely free of pain during the postoperative period B. The pump allows the patient to take unlimited amounts of medication as needed C. The pump allows the patient to choose the type of medication given postoperatively. D. The pump allows the patient to self administer limited doses of pain medication

D. PCA infusion pumps allow patients to self administer doses of pain relieving medication within physician prescribed time and dose limits. Patients activate the delivery of the medication by pressing a button on a cord connected to the pump or a button directly on the pump.

A patient is scheduled for elective hernia surgery. While taking a medical history, the nurse learns that the patient is taking antibiotics for an infection. Which surgical risk should the nurse monitor based on this antibiotic use? A. Hemorrhage B. Electrolyte imbalance C. Cardiovascular collapse D. Respiratory paralysis

D. Respiratory paralysis

What catheter would the nurse use to drain a patient's bladder for short periods (5 to 10 minutes)? A. Foley catheter B. Suprapubic catheter C. Indwelling urethral catheter D. Straight catheter

D. Straight catheter

Vitamin K, 10mg given IM is ordered. Vitamin K is available as 5mg/5mL. How much would the nurse administer?

Dose on hand = . Dose desired --------------- --------------- Quantity on hand . Quantity desired 5mg = 10mg ------ ------ 1mL X 5X=10 X=2mL

As part of admission process, nurse at long-term care facility is gathering nutrition history for client with dementia. Which component is priority to determine from their family? A: BMI B: Usual times for meals and snacks C: Favorite foods D: Any difficulty swallowing

D. The greatest risk to this client is related to a nutrition related evaluation is from difficulty swallowing or dysphagia. It puts the client at risk for aspiration, which can be life threatening. It is important to calculate body mass index to help determine appropriateness of the client's weight status and related risks; it is important to know and try to follow the meal schedule the client follows at home; it is important to know which foods are the client's favorites in case it becomes difficult to get the client to consume adequate nutrients; however any difficulty swallowing is the priority

A nurse is caring for a client who will perform fecal occult blood testing at home. Which of the following information should the nurse include when explaining the procedure to the client? A. Eating more protein is optimal prior to testing B. One stool specimen is sufficient for testing C. A red color change indicates a positive test D. The specimen cannot be contaminated with urine

D. The specimen cannot be contaminated with urine. For fecal occult blood testing, the nurse should warn the client not to contaminate the stool specimens with water or urine. Some proteins can alter the test results. The nurse should instruct the client not to consume red meat, fish, and poultry prior to testing. The nurse should instruct the client to obtain 3 specimens from 3 different bowel movements. The nurse should inform the client to look for a blue color on the card to indicate positive blood in the stool.

A nurse is assessing a patient who has been NPO (nothing by mouth) prior to abdominal surgery. The patient is ordered as a clear liquid diet for breakfast, to advance to a house diet as tolerated. Which assessments would indicate to the nurse that the patient's diet should not be advanced? A. The patient consumed 75% of the liquids on her breakfast tray. B. The patient tells you she is hungry C. The patient's abdomen is soft, non distended, with bowel sounds D. The patient reports fullness and diarrhea after breakfast

D. Tolerance of diet can be assessed by the following: absence of nausea, vomiting, and diarrhea; absence of feelings of fullness; absence of abdominal pain and distention; feelings of hunger; and the ability to consume at least 50% to 75% of the food on the meal tray

The nurse is choosing a collection device to collect urine from a nonambulatory male patient? What would be the nurse's best choice? A. Specimen B. Large urine collection bag C. Bedpan D. Urinal

D. Urinal

A nurse is inserting a catheter into a female urinary bladder. Which nursing action is performed correctly? A. Clean the perineal area with a gauze pad and alcohol using a different corner of the gauze with each stroke B. Assist the patient to a prone position with knees flexed, feet about 2 feet apart, with legs abducted C. Using dominant hand, hold the catheter 12 inches from the tip and insert slowly into the urethra D. Use dominant hand to inflate the catheter balloon, and inject entire volume of sterile water supplied in prefilled syringe

D. Use dominant hand to inflate the catheter balloon, and inject the entire volume of sterile water supplied in prefilled syringe

A nurse is feeding a patient who is experiencing dysphagia. Which nursing intervention would the nurse initiate for this patient? A. Feed the patient solids first and then liquids last B. Place the head of the bed at a 30 degree angle during feeding. C. Puree all foods to a liquid consistency D. Provide a 30 minute rest period prior to mealtime

D. When feeding a patient who has dysphagia, the nurse should provide a 30minute rest period to mealtime to promote swallowing; alternate solids and liquids when feeding the patient; sit the patient upright or, if on bedrest, elevate the head of the bed at a 90 degree angle; and initiate a nutrition consult for diet modification and food size and/or consistency

A nurse is teaching a man scheduled to have same day surgery. Which teaching method would be most effective in preoperative teaching for ambulatory surgery? A. Lecture B. Discussion C. Audiovisuals D. Written instrustions

D. Written instructions are most effective in providing information for same day surgery

A nurse in an outpatient clinic is teaching a client who is in her first trimester of pregnancy. Which of the following statements should the nurse make? A. You will need to get a rubella immunization if you haven't had one prior to pregnancy B. You can safely take over the counter medications C. You should avoid any vitamin preparations containing iron D. Your provider can prescribe medication for nausea if you need it

D. Your provider can prescribe medication for nausea if you need it. Providers can prescribe medications to treat nausea and other discomforts of pregnancy. Pregnancy is a contraindication for live virus vaccines, including rubella, due to possible teratogenic effects. Most medications, including over the counter, are potentially harmful to the fetus. The client should avoid any medications unless her provider prescribes them. Nutritional supplements that include iron are common recommendations during pregnancy to support the health of the mother and fetus

Which equipment technique should the nurse use to administer most intramuscular injections? (Select all that apply.) A. Use a 1-inch needle B. Use a 25-gauge needle C. Insert the needle at a 45-degree angle D. Aspirate before instilling the medication (Textbook and EBP says not to do this anymore, but some things still say to do this!!) E. Massage the insertion site after needle removal

D.E. Aspiration is done before instilling the medication to ensure that a blood return does not occur, which indicates that the needle is in a blood vessel. Massage promotes dispersion of the medication. Heparin is one exception to this rule!

A primary healthcare provider orders a clear liquid diet for a patient. Which foods should the nurse teach the patient to avoid when following this diet? (Select all that apply.) A. Strawberry gelatin B. Decaffeinated tea C. Strong coffee D. Pureed soup E. Ice cream

D.E. Pureed soups are permitted on a full-liquid diet, not clear liquid, diet. Puree soups have a high-solute load, including fats and proteins, which stimulates the digestive process. Milk and milk products are not included on a clear liquid diet. Ice cream contains a high-solute load, including fats and proteins, which stimulates the digestive process.

Which routes are associated with the administration of a suppository? (Select all that apply.) A. Ear B. Nose C. Mouth D. Vagina E. Rectum

D.E. Semisolid cone-shaped or oval suppositories that melt at body temperature can be inserted into the vagina or rectum.

A nurse is caring for patients in a long term care facility is often required to collect urine specimens from patients for laboratory testing. Which techniques for urine collection are performed correctly? (Select all that apply) A. The nurse catheterizes a patient to collect a sterile urine sample for routine urinalysis B. The nurse collects a clean catch urine specimen in the morning from a patient and stores it at room temperature until an afternoon pickup C. The nurse collects a sterile urine specimen from the collection receptacle of a patient's indwelling catheter. D. The nurse collects about 3mL of urine from a patient's indwelling catheter to send for a urine culture E.The nurse collects a urine specimen from a patient with a urinary diversion by catheterizing the stoma F. The nurse discards the first urine of the day when performing a 24 hour urine specimen collection on a patient

D.E.F. A urine culture requires about 3mL of urine, whereas routine urinalysis requires at least 10mL of urine. The preferred method of collecting a urine specimen from a urinary diversion is to catheterize the stoma. For a 24hour urine specimen, the nurse should discard the first voiding, then collect all urine voided for the next 24 hours. A sterile urine specimen is not required for a routine urinalysis. Urine chemistry is altered after urine stands at room temperature for a long period of time. A specimen from the collecting receptacle (drainage bag) may not be fresh urine and could result in an inaccurate analysis

A nurse prepares to assist a patient with her newly created ileostomy. Which recommended patient teaching points would the nurse stress? (Select all that apply) A. When you inspect the stoma, it should be dark purple-blue B. The size of the stoma will stabilize within 2 weeks C. Keep the skin around the stoma site clean and moist D. The stool from an ileostomy is normally liquid E. You should eat dark green vegetables to control the odor of the stool F. You may have a tendency to develop food blockages

D.E.F. Ileostomies normally have liquid, foul smelling stool. The nurse should encourage the intake of dark green vegetables because they contain chlorophyll, which helps to deodorize the feces. Patients with ileostomies need to be aware they may experience a tendency to develop food blockages, especially when high fiber foods are consumed. The stoma should be dark pink to red and moist. Stoma size usually stabilizes within 4 to 6 weeks, and the skin around the stoma site (peristomal area) should be kept clean and dry

Table 35-9 Common Components of TPN

Dextrose (carbohydrate)- calories for metabolism Amino acids- protein for tissue repair Lipids- essential fatty acids and calories for metabolism, wound healing, red blood cell production Acetate- Prevents metabolic acidosis Calcium-Development of bones and teeth; aids in blood clotting Chloride- Regulates acid-base balance; maintains osmotic pressure Folic acid- DNA formation; promotes growth and development Magnesium- Carbohydrate and protein absorption Phosphorous- Cell energy and calcium balance Potassium- Cellular activity and cardiac function Sodium- Controls water distribution; maintains normal fluid balance Vitamin B complex- Carbohydrate absorption Vitamin C- Wound healing Vitamin D- Bone metabolism, maintains serum calcium levels Vitamin K- Prevents bleeding disorders Micronutrients, such as Zinc, cobalt, and manganese- Wound healing; red blood cell synthesis

Glycopurrolate (Robinul), 0.4mg IM, is ordered. Glycopyrrolate is available as 0.2mg/mL. How much would the nurse administer?

Dose desired x quantity on hand = X ---------------- Dose on hand 0.4mg x 1mL = X -------- 0.2mg (2)(1)= x x= 2mL

Theophylline elixir, 100mg PO, is ordered by way of a percutaneous endoscopic gastrostomy tube. Theophylline elixir is available as 80mg/15mL. How much would the nurse administer?

Dose desired x quantity on hand = X ---------------- Dose on hand 100mg x 15mL= X -------- 80mg 1500 =X ------ 80 X=18.75 or 19mL

Propantheline (Pro-Banthine), 15mg, is ordered. Propantheline bromide is available as 7.5mg tablets. How many tablets would the nurse administer?

Dose desired x quantity on hand = X ---------------- Dose on hand 15mg x 1 tablet = X ------ 7.5mg (2)(1)= 2 tablets

Octreotide acetate (Sandostatin), 50mcg SC, is ordered. Octreotide is available as 100mcg/mL. How much would the nurse administer?

Dose desired x quantity on hand= X(desired quantity) --------------- Dose on hand 50mcg x 1mL= X -------- 100mcg (1/2)(1)= X X= 1/2mL

Epoeitin alfa (Epogen), 2000units SC, is ordered. Epoetin alfa is available as 4000units/mL. How much would the nurse administer?

Dose on hand = . Dose desired --------------- --------------- Quantity on hand . Quantity desired 4000U = 2000U ------- --------- 1mL . X 4000X= 2000 X= 1/2mL

Morphine sulfate 10mg IV is ordered. Morphine is available as 4mg/mL. How much would the nurse administer?

Dose on hand = . Dose desired --------------- --------------- Quantity on hand . Quantity desired 4mg = 10mg ----- ------ 1mL . X 4X= 10 X= 2.5mL

Phenytoin (Dilantin), 100mg PO, is ordered to be given through a nasogastric tube. Phenytoin is available as 30mg/5mL. How much would the nurse administer?

Dose on hand = Dose required ---------------- ----------------- Quantity on hand X(quantity desired) 30mg = 100mg ------ -------- 5mL . X Cross multiply: 30X= 500 X= 16.66 or 17mL

Metoprolol (Lopressor), 25mg PO, is ordered. Metoprolol is available as 50mg tablets. How many tablets would the nurse administer

Dose on hand = Dose required ---------------- ----------------- Quantity on hand X(quantity desired) 50mg =25mg ------ ------ 1 tab. X Cross multiply: 50X=25 X= 0.5 or 1/2 tablet

Active Learning Scenario: Chapter 44 List at least 10 common risk factors for urinary incontinence

Female gender History of multiple pregnancies and vaginal births, chronic urinary retention, urinary bladder spasm, renal disease, chronic bladder infection Neurological disorders: Parkinson's disease, cerebrovascular accident, spinal cord injury, multiple sclerosis Medications: diuretics, opioids, anticholinergics, calcium channel blockers, sedative/hypnotics, adrenergic antagonists Obesity Confusion, dementia, immobility, depression Physiological changes of aging Decreased estrogen levels, decreased pelvic-muscle tone Immobility, chronic degenerative diseases, dementia, diabetes mellitus, cerebrovascular accident Urinary incontinence increasing the risk for falls, fractures, pressure ulcers, and depression

Table 35-5 Clinical Observations for Nutritional Assessment

General Appearance- Alert, responsive (signs of good nutritional status); listless, apathetic, and cachetic (signs of poor nutritional status) General vitality- Endurance, energetic, sleeps well, vigorous (signs of good nutritional status); Easily fatigued, no energy, falls asleep easily, looks tired, apathetic, depressed mood (signs of poor nutritional status) Weight- Normal for height, age, body build (signs of good nutritional status); Overweight or underweight (signs of poor nutritional status) Hair- Shiny, lustrous, firm, not easily plucked, healthy scalp (signs of good nutritional status); Dull and dry, brittle, loss of color, easily plucked, thin and sparse (signs of poor nutritional status) Face- Uniform skin color; healthy appearance, not swollen (signs of good nutritional status); Dark skin over cheeks and under eyes, flaky skin, facial edema (moon face), pale skin color (signs of poor nutritional status) Eyes- Bright, clear, moist, no sores at corners of eyelids, membranes, moist and healthy pink color, no prominent blood vessels (signs of good nutritional status); Pale eye membranes, dry eyes (xerophthalmia), Bitot's spots, increased vascularity, cornea soft (keratomalacia), small yellowish lumps around eyes (Xanthelasma), dull or scarred cornea (signs of poor nutritional status) Lips- Good pink color, smooth, moist, not chapped or swollen (signs of good nutritional status); swollen and puffy (cheilosis) angular lesion at corners of mouth or fissures or scars (stomatitis) (signs of poor nutritional status) Tongue- Deep red, surface papillae present (signs of good nutritional status); Smooth appearance, beefy red or magenta colored, swollen hypertrophy or atrophy (signs of poor nutritional status) Teeth- Straight, no crowding, no cavities, no pain, bright, no discoloration, well shaped (signs of good nutritional status); Cavities, mottled appearance (Fluorosis), malpositioned, missing teeth (signs of poor nutritional status Gums- Firm, good pink color, no swelling or bleeding (signs of good nutritional status); Spongy, bleed easily, marginal redness, recessed, swollen and inflamed (signs of poor nutritional status) Glands- No enlargement of the thyroid, face not swollen (signs of good nutritional status); Enlargement of the thyroid (goiter), enlargement of the parotid (swollen cheeks) (signs of poor nutritional status) Skin- Smooth, good color, slightly moist, no signs of rashes, swelling, or color irregularities (Signs of good nutritional status); Rough, dry, flaky, swollen, pale, pigmented, lack of fat under the skin, fat deposits around the joints (xanthomas), bruises, petechiae (signs of poor nutritional status) Nails- firm, pink (signs of good nutritional status); Spoon shaped (koilonychia), brittle, pale, ridged (signs of poor nutritional status) Skeleton- Good posture, no malformations (signs of good nutritional status); poor posture, beading of the ribs, bowed legs, or knock knees, prominent scapulas, chest deformity at diaphragm (signs of poor nutritional status) Muscles- Well developed, firm, good tone, some fat under the skin (signs of good nutritional status); Flaccid, poor tone, wasted, underdeveloped, difficulty walking (signs of poor nutritional status) Extremities- No tenderness (signs of good nutritional status); weak and tender, edema of lower extremities (signs of poor nutritional status) Abdomen- Flat (sign of good nutritional status); Swollen (sign of poor nutritional status) Nervous system- Normal reflexes, psychological stability (sign of good nutritional status); Decrease in or loss of ankle and knee reflexes, psychomotor changes, mental confusion, depression, sensory loss, motor weakness, loss of sense of position, loss of vibration, burning and tingling of the hands and feet (paresthesia) (signs of poor nutritional status) Cardiovascular status- Normal heart rate and rhythm, no murmurs, normal blood pressure for age (signs of good nutritional status); Cardiac enlargement, tachycardia, abnormal blood pressure (signs of poor nutritional status) GI system: No palpable organs or masses (liver edge may be palpable in children) (signs of good nutritional status); Enlarged liver or spleen (signs of poor nutritional status)

Active Learning Scenario: Chapter 9 List at least 3 aspects of the health history the nurse must gather and document, as well as at least 3 aspects of the psychosocial evaluation the nurse must gather and document

Health History -Current Illness -Current medications (prescription, herbal supplements, and over the counter) -Prior illnesses, chronic diseasees -Surgeries -Previous hospitalizations Psychosocial assessment: -Alcohol, tobacco, drug, and caffeine use -History of mental illness -History of abuse or homelessness -Home situation/ significant others

Box 35-6 Biochemical Data With Nutritional Implications

Hemoglobin (normal= 12-18g/dL) -Decreased: anemia Hematocrit (normal 40-50%) -Decreased: anemia -Increased: Dehydration Serum albumin (normal= 3.5-5.5g/dL) -Decreased: malnutrition (prolonged protein depletion) Prealbumin (normal= 23-43 mg/dL) -Decreased: protein depletion, malnutrition Transferrin (normal= 240-480mg/dL) -Decreased: anemia, protein deficiency Blood urea nitrogen (normal= 17-18mg/dL) -Increased: starvation, high protein intake, severe dehydration -Decreased: malnutrition, overhydration Creatinine (normal= 0.4-1.5mg/dL) -Increased: dehydration -Decreased: reduction in total muscle mass, severe malnutrition

Table 28-2 Routes for Administering Drugs

Oral route- having the patient swallow the drug Enteral route- Administering the drug through an enteral tube Sublingual administration- Placing a drug under the tongue Buccal route- Placing drug between cheek and gum Parenteral route- Injecting the drug into Subcutaneous injection- subcutaneous tissue Intramuscular injection- Muscle tissue Intradermal injection- Corium (under epidermis) Intravenous injection- Vein Intra-arterial injection- Artery Intracardial injection- Heart tissue Intraperitoneal injection- peritoneal cavity Inraspinal injection- spinal cavity Intraosseous injection- Bone Topical Route- applying drug onto skin or mucous membrane Vaginal administration- Vagina Rectal administration- Retum Inunction- Rubbing drug into skin Instillation- Placing drug into direct contact with mucous membrane Irrigation- Flushing mucous membrane with drug in solution Skin application- Applying transdermal patch Pulmonary route- having patient inhale drug

Active Learning Scenario: Chapter 51 Identify at least four physiologic changes with aging that affect pharmacokinetics

Physiologic changes: -Increased gastric pH (alkaline0 - Decreased gastrointestinal motility and gastric emptying time, resulting in a slower rate of absorption -Decreased blood flow through the cardiovascular system, liver, and kidneys. -Decreased hepatic enzyme function -Decreased kidney function and glomerular filtration rate -Decreased protein binding sites, resulting in lower serum albumin levels -Decreased body water, increased body fat, and decreased lean body mass. -Impaired memory or altered mental state -Multiple or severe illnesses -Changes in vision and hearing -Decreased mobility and dexterity

Box 28-3 Typical Preparations to the skin areas

Powder- Promote drying of the skin; Prevent friction on the skin; Use caution when applying to prevent inhalation of the powder. Apply powder to gauze square, then apply to the desired site to minimize inhalation of the airborne particles Ointment- Provide prolonged contact of a medication with the skin; soften the skin. Massage thoroughly into intact skin Creams and oils- Lubricate and soften skin; prevent drying of skin. When applying to large parts of the body, warm preparation in the hand or fingers to prevent the patient from experiencing chilling Lotions- Protect and soothe the skin. Shake thoroughly before using. Apply with cotton balls or gauze Transdermal: reservoirs, micro-reservoirs, adhesives, matrices- These systems are a "sandwich" of layers, each with a specific job. An impermeable backing prevents drug diffusion from the exposed portion. The drug layer system contains drug, with a rate-controlling layer to slow the release of the drug over time, ending with an adhesive layer to enhance the attachment of the system to the patient's skin. Wear gloves. Handle by edges to avoid touching drug when handling system. Rotate application sites to avoid skin irritation

Table 28-3 Ensuring Correct Administration of Medications "Right to ensure" Questions to ask Actions to take

Right Drug- Has the patient been given this medication before? Given the patient's symptoms and diagnosis, does it make sense for the patient to have this medication. Determine if the patient has any known drug allergies or sensitivities. Assess the patient's other medications to detect possible contraindications. Make sure it is the right medication, packaging, labeling, and spelling of some drugs look alike. Have another person double check al medications and mathematical calculations Right reason- Do the patient's condition, symptoms, and health status warrant receiving this medication. Determine if the patient has the condition the medication is used for. Right dose, route, and preparation- Is the correct dose being administered? How is the medication being administered? Ensure that labeling is legible and clearly understood. Right patient- Is this the right patient to receive this medication? Verify the identity of the patient using at least two identifiers: Name and date of birth (check wristband, ask patient to state name) Right time- Is this the correct time for the medication to be administered? Check when the medication was last administered. If the drug is new document when it is first given. Right assessment data-Is it appropriate to administer the medication, based on the specific data collected? Collect appropriate assessment data related to mechanism of action and/or therapeutic effect. Right route- Is it appropriate to administer the medication orally, intravenously, by injection, or other route? Check the original orders to verify the route of administration Right education- Is the patient familiar with the medication? Does the patient understand the purpose, dosing, and administration information, as well as other information specific to medication? Assess patient's level of knowledge. Provide patient education as necessary Right documentation- Has the correct documentation been completed according to facility policy? Complete documentation according to facility policy immediately after administering any medication. Document and communicate to the appropriate health care provider any signs and symptoms indicative of any adverse effects. Right response- How is the patient responding to the medication? Monitor the patient to: Determine the efficacy of the drug; detect and prevent complication; evaluate and document changes in health status. When applicable, assess the patient's laboratory values to detect changes. Provide patient education, when possible, so patient is alert to adverse effects and changes in how he feels.

Active Learning Scenario: Chapter 47 List the rights of safe medication administration

Right client: Verify clients' identification before each medication administration. The Joint Commission requires two client identifiers. Acceptable identifiers include the clients name, an assigned identification number, telephone number, DOB, or other person specific identifier, such as photo identification card. Nurses also use bar code scanners to identify clients. Check for allergies by asking clients, checking for an allergy bracelet or medal, and checking the MAR Right medication: Correctly interpret medication prescriptions, verifying completeness and clarity. Read medication labels and compare them with the MAR three times: before removing the container, when removing the amount of medication from the container, and in the presence of the client before administering the medication. Leave unit dose medication in its package until administration Right dose: Use a unit dose system to decrease errors. If not available, calculate the correct medication dose; check a drug reference to make sure the dose is within the usual range. Ask another nurse to verify the dose is uncertain of the calculation. Prepare medication dosages using standard measurement devices, such as graduated cups or syringes. Some medication dosages, such as some cytotoxic medications, require a second verifier or witness. Automated medication dispensing systems use a machine to control the dispensing of medications Right time: Administer medication on time to maintain a consistent therapeutic blood level. It is generally acceptable to administer the medication 30 minutes before or after the scheduled time. Refer to the drug reference or the facility's policy for exceptions Right route: The most common routes of administration are oral, topical, subcutaneous, intramuscular, and intravenous. Additional administration routes include sublingual, buccal, intradermal, epidural, inhalation, nasal. ophthalmic, otic, rectal, vaginal, intraosseous, and via enteral tubes. Select the correct preparation for the route the provider prescribed (otic vs. ophthalmic topical ointment or drops) Right Documentation: Immediately record pertinent information, including the client's response to the medication. Document the medication after administration, not before. Right client education: Inform clients about the medication: its purpose, what to expect, how to take it, and what to report. To individualize the teaching, determine what the clients already know, need to know, and want to know about the medication Right to refuse: Respect clients right to refuse any medication. Explain the consequences, inform the provider, and document the refusal. Right assessment: Collect any essential data before and after administering any medication. For example, measure apical heart rate before giving digoxin

Focus on the Older Adult: Nursing Strategies to Address Age Related Changes Effecting Bowel Elimination

Slowing of gastrointestinal motility with increased stomach emptying time. -Encourage small, frequent meals. -Discourage heavy activity after eating -Encourage a high fiber-low fat diet -Encourage adequate fluid intake -Discourage regular use of laxatives -Evaluate medication regimen for possible adverse effects Decreased muscle tone/ incontinence -Provide easy access to the bathroom -Use assistive devices when necessary (raised toilet seat, grab bars, walker) -Ensure safety when ambulating (skid proof slippers) -Encourage participation in bowel retraining program Weakening of intestinal walls with greater incidence of diverticulitis -Encourage a high fiber diet and adequate fluid intake. -Teach patients not to ignore the urge to have a bowel movement -Encourage regular exercise

Table 37-4 Commonly used enema solutions

Tap water (hypotonic): 500-1000mL (amount); distends intestine, increases peristalsis, softens stools (action), 15min (time to take effect); Fluid and electrolyte imbalance, water intoxication (adverse effects) Normal saline (isotonic): 500-1000mL (amount); Distends intestine, increases peristalsis, softens stools (Action); 15min (time to take effect); Fluid and electrolyte imbalance, sodium retention (adverse effects) Soap: 500-1000mL(concentrate at 3-5mL/ 1000mL) (amount); Distends intestine, irritates intestinal mucosa, softens stool (action); 10-15min (time to take effect); Rectal mucosa irritation or damage (adverse effects) Hypertonic: 70-130mL(amount); distends intestine, irritates intestinal mucosa (action) 5-10min (time to take effect); sodium retention (adverse effect) Oil (mineral, olive, or cottonseed): 150-200mL (amount); Lubricates stool and intestinal mucosa (action) 30min (time to take effect)

Table 35-1 Risk of obesity associated diseases and condition by BMI and waist circumference relative to normal weight and waist circumference

Underweight- less than 18.5 (BMI) Waist circumference (Men less than or equal to 40 inches and women 35 inches) NA Waist circumference (Men greater than 40inches and women 35inches) NA Normal- 18.5-24.9 (BMI) Waist circumference (Men less than or equal to 40 inches and women 35 inches) NA Waist circumference (Men greater than 40inches and women 35inches) NA Overweight- 25.0-29.9 (BMI) Waist circumference (Men less than or equal to 40 inches and women 35 inches) -Increased Waist circumference (Men greater than 40inches and women 35inches) - High Obesity, Class I- 30.0-34.9 (BMI) Waist circumference (Men less than or equal to 40 inches and women 35 inches)- High Waist circumference (Men greater than 40inches and women 35inches)- Very High Obesity, Class II- 35.0-39.9 Waist circumference (Men less than or equal to 40 inches and women 35 inches)- Very high Waist circumference (Men greater than 40inches and women 35inches)- Very high Obesity, Class III- 40.0+ Waist circumference (Men less than or equal to 40 inches and women 35 inches)- Extremely high Waist circumference (Men greater than 40inches and women 35inches)- Extremely high

Table 29-2 Postoperative assessments and interventions on return to the unit Factors to assess/ assessments and interventions

Vital signs and oxygen saturation: -Temperature, blood pressure, pulse and respiratory rates, oxygen saturation -Note, report, and document deviations from preoperative and PACU data as well as symptoms of complications Color and temperature of skin: -Skin color (pallor, cyanosis), skin temperature, and diaphoresis Level of consciousness: -Orientation to time, place, and person -Reaction to stimuli and ability to move all four extremities Intravenous fluids: -Type and amount of solution, flow rate, security and patency of tubing -Infusion site Surgical site: -Dressing and dependent areas for drainage (color, amount, consistency) -Drains and tubes, be sure they are intact, patent, and properly connected to drainage systems Other tubes: -Assess indwelling urinary catheter, gastrointestinal suction, and others for drainage, patency, and amount of output -Ensure that dependent drainage bags are hanging properly and suction draining is attached and functioning -If oxygen is ordered, ensure placement of ordered application and flow rate Comfort: -Assess pain (location, duration, intensity) and determine whether analgesics were given in the PACU -Assess for nausea and vomiting -Cover the patient with a blanket -Reorient to the room as necessary -Allow family members to remain with the patient after the initial assessment is completed Position and safety: -Place the patient in an ordered position or -If the patient is not fully conscious, place in a side lying position -Elevate the side rails and place the bed in low position

Table 37-1 The Stool: Normal Characteristics and Special Considerations for Observation

Volume; Variable (Normal finding); The volume of the stool depends on the amount the person eats and the nature of the diet. For example, a diet high in roughage produces more feces than a soft, bland diet. Consistently large diarrheal stools suggest a disorder in the small bowel or proximal colon; small, frequent stools with urgency to pass them suggest a disorder of the left colon or rectum (Special Considerations for Observation) Color: Infant-Yellow to brown; Adult: brown (Normal finding); The brown color of stool is due to stercobilin, a bile pigment derivative. The rapid rate of peristalsis in the breastfed infant causes the stool to be yellow. The color of the stool is influenced by diet. For example, the stool will be almost black if the person eats red meat and dark green vegetables, such as spinach. The stool will be light brown if the diet is high in milk and milk products and low in red meat. The absence of bile may cause the stool to appear white or gray colored. Certain drugs influence the color of the stool. For example, iron salts cause the stool to be black. Antacids cause it to be whitish. Bleeding high in the intestinal tract causes a stool to be black owing to the digestion of blood. Bleeding low in the intestinal tract results in fresh blood in the stool. The stool darkens with standing (Special considerations for observation) Odor: Pungent, may be affected by the foods ingested. (Normal finding); The characteristic odor of the stool is due to indole and skatole, caused by putrefaction and fermentation in the lower intestinal tract. The odor of the stool is influenced by it pH value, which normally is neutral or slightly alkaline. Excessive putrefaction causes strong odor. The presence of blood in the stool causes a unique odor. (Special Consideration for observation) Consistency: Soft, semisolid, and formed (normal finding); The consistency of the stool is influenced by fluid and food intake and gastric motility. The less time stool spends in the intestine (or the shorter the intestine), the more liquid the stool. Many pathologic conditions influence consistency. (Special considerations for observation) Shape: Formed stool is usually about 1inch (2.5cm) in diameter and has the tubular shape of the colon, but may be larger or smaller depending on the condition of the colon. (normal finding); A gastrointestinal obstruction may result in a narrow, pencil shaped stool. Rapid peristalsis thins the stool. Increased time spent in the large intestine may result in a hard, marble like fecal mass. (Special considerations for observation) Constituents: Waste residues of digestion: bile, intestinal secretions, shed epithelial cells, bacteria, and inorganic material (chiefly calcium, and phosphates); seeds, meats, fibers, and fat may be present in small amounts (normal finding); Internal bleeding, infection, inflammation, and other pathologic conditions may result in abnormal constituents. These include blood, pus, excessive fat, parasites, ova, and mucus. Foreign bodies also may be found in the stool. (Special considerations for observation)

Table 35-3 Summary of vitamins

Water soluble vitamins: Vitamin C (ascorbic acid); citrus fruits, broccoli, green pepper, strawberries, greens; collagen formation, antioxidant, enhances iron absorption; Scurvy, hemorrhaging, delayed wound healing (signs of deficiency); hot flashes, headache, nausea, diarrhea (signs of excess) Vitamin B Complex- Thiamine; Pork, liver, whole and enriched grains, legumes; Conezyme in key reactions that produce energy from glucose; Beriberi, mental confusion, fatigue (signs of deficiency); none known (signs of excess) Riboflavin: milk, organ meats, enriched grains, greens; carbohydrate, protein, and fat metabolism; Dermatitis, glossitis, photophobia (signs of deficiency), none entered (signs of excess) Niacin: Kidney, grains, lean meat, nuts; carbohydrate, protein, and fat metabolism; Pellagra, dermatitis (signs of deficiency); Flushing and itching, nausea and vomiting, liver damage (signs of excess) B6 (Pyridoxine); Yeast, banana, cantaloupe, broccoli, spinach; Coenzyme in protein, fat, carbohydrate metabolism; Microcytic anemia, CNS problems (signs of deficiency); Difficulty walking, numbness of feet and hands (signs of excess) Folate: Green leafy vegetables, liver; RNA and DNA synthesis, formation and maturation of RBC; Macrocytic anemia, fatigue, weakness, pallor (signs of deficiency) None known (signs of excess) B12 (Cyanocobalamin): Animal products, organ meats, seafood; Coenzyme in protein metabolism and formation of heme portion of hemoglobin; Pernicious anemia (B12 deficiency related to impaired absorption due to lack of intrinsic factor) (signs of deficiency) none known (signs of excess) Pantothenic acid: Wide spread in foods, meat, poultry, fish, whole grain cereals; carbohydrate, protein, and fat metabolism; None known (signs of deficiency) none known (signs of excess) Biotin; Liver, egg yolk; carbohydrate, protein, and fat metabolism; Rare; dry skin, anorexia, fatigue (signs of deficiency) none known (signs of excess) Fat soluble vitamins: Vitamin A (retinol, retinal, retinoic acid): liver, carrots, egg yolk, fortified milk; Visual acuity in dim light, formation and maintenance of skin and mucuos membranes, immune function; Night blindness, rough skin, bone growth ceases (signs of deficiency); Anorexia, loss of hair, dry skin, bone pain, vomiting, birth defects during pregnancy (signs of excess) Vitamin D (Cholecalciferol, ergocalciferol); Sunlight, fortified milk, fish liver oils; Calcium and phosphorous metabolism, stimulates calcium absorption; Retarded bone growth, bone malformation (signs of deficiency); Excessive calcification of bones, renal calculi, nausea and headache (signs of excess) Vitamin E (tocopherol); vegetable oils, wheat germ, whole grain products; antioxidant, protects vitamin A, heme synthesis; Increased RBC hemolysis and macrocytic anemia in premature infants (signs of deficiency); Relatively nontoxic, although large doses can cause fatigue, diarrhea, and enhance action of anticoagulant medications (signs of excess) Vitamin K: Dark green leafy vegetables, synthesized in intestines from gut bacteria; Synthesis of certain proteins necessary for blood clotting; hemorrhagic disease of newborn, delayed blood clotting (signs of deficiency), Hemolytic anemia and liver damage with synthetic vitamin K (signs of excess)

Box 27-2 CAT Domains

Whole (or alternative) medical systems: similar to the Western, allopathic model in that they are complete systems of theory and practice. They consist of a set of beliefs about the origin of diseases, ways to promote health, and types of treatment. Mind body medicine: uses a variety of techniques designed to enhance the mind's ability to affect bodily function and symptoms Energy medicine: Involves the use of energy fields, such as magnetic fields or biofields (energy fields that some believe surround and penetrate the body.) Manipulative and body based practices; Work with the structures and systems of the body, including bones and joints, soft tissues, and circulatory, and lymphatic systems. Biologically based practices: include the use of botanicals (herbs), animal derived extracts, vitamins, minerals, fatty acids, amino acids, proteins, prebiotics and probiotics, whole diets, and functional foods

Lisinopril (Zestril) 40mg once a day is ordered. Zestril is available as an oral solution 8mg/mL. How much should the nurse administer via the enteral feeding tube?

dose on hand = dose desired --------------- --------------- quantity on hand . quantity desired 8mg = 40mg ------ ------ 1mL . X 8X= 40 X= 5mL via enteral feeding tube


Kaugnay na mga set ng pag-aaral

Intro to Coding - Conditional Statements

View Set

第一課漢字好好玩 廖淑儀Mint

View Set

Atomic Structure and Periodic Trend

View Set

Airfield Managment: Driver Training

View Set

IFRS-Chapter 3-Fair Value Measurement

View Set